Scored an 88 on the series 7 just now. Passing is 72. My score on the 5 areas on the actual exam was:

Seeks business:86-90

Evalautes customer’s:86-90

opens accounts:76-85

provides informaiton:86-90

obtains and verifies: 86-90

Investopedia has a 602 question thing that I only scored a 78.9 on. Score below. Also, Below that is all of my notes. I took a few practice exams and never scored above a 78 really… anyway. below is a copy pasta of all of my note. i did both sections and submitted them with 60 minutes to spare on each so the exam with the 30 minute forced intermission took however long that is. not sure.

so i got 221 out of the 250 questions that counted. i guess there are 10 questions in there that are for there research purposes that don’t actually count. if 72 is passing that means a 180 is passing so i got 41 questions right that were in excess of me passing. so, i could have passed without a calculator, without using a pen… and i probably over studied. i put in like 2-3 weeks of full days.. read through 2 books on the exam and the whole investopedia, took all the practice problems, 1000+.

Practice Exam History

Statistics

Keep up the great work! Your overall score is 78.9%.

Breakdown Per Topic Category: Skipped Correct Incorrect % Correct
Provides customers with information on investments and makes suitable recommendations.
 Drill Down
0 327 60 84.5%
Skipped Correct Incorrect % Correct
Seeks business for the broker-dealer through customers and potential customers
 Drill Down
0 52 23 69.3%
Skipped Correct Incorrect % Correct
Obtains and Verifies Customer’s Purchase and Sales Instructions, Enters Orders, and Follows Up.
 Drill Down
1 35 26 56.5%
Skipped Correct Incorrect % Correct
Opens Accounts, Transfers Assets, and Maintains Appropriate Account Records.
 Drill Down
0 38 13 74.5%
Skipped Correct Incorrect % Correct
Evaluates Customers’ Other Security Holdings, Financial Situation and Needs, Financial Status, Tax Status, and Investment Objectives.
 Drill Down
0 23 4 85.2%

Total Questions Attempted: 602

 Answered Correctly: 475

 Answered Incorrectly: 126

 Skipped Answers:

 

Ex-dividend dates

calculator

Review overview of exam strategy

Review exam alerts

If you absolutely do not know the answer to a definitional type question, select

the choice that has the largest number of words.

 

Which of the following information is found in the official notice of sale in relation to a competitive offering of a municipal bond?

  1. The priority of execution of orders when released to the public
  2. II.    The allotment to each syndicate member
  3. III.   The method of interest-cost calculation used to determine the winner
  4. IV.  The reoffering yields of the bonds with different maturities

Sorry, that is not the correct answer. The correct answer is: A.   III only

The official notice of sale, which you find in The Bond Buyer, contains bidding details for municipal bonds. Included in the bidding details are Roman numeral III — the method of interest-cost calculation used to determine the winner (net interest cost or true interest cost) — and the amount of the good faith deposit, which isn’t one of your choices. Roman numerals I and II are present in the syndicate agreement, and reoffering yields aren’t found anywhere. The answer is A.

 

The point in a limited partnership where revenues exceed deductions is called

Sorry, that is not the correct answer. The correct answer is: B.    the crossover point

Because you’re looking for a point in time where revenues exceed deductions, it makes sense that the word point would be in the answer. The answer is unlikely to be a time period (C) or a type of income (D). When a partnership is set up, it usually takes a little while (or maybe several years) to start making enough revenue to exceed the deductions. The point at which the revenues exceed deductions is called the crossover point.

 

Which TWO of the following are TRUE regarding Treasury STRIPS?            I.             The security’s principal and interest are paid at maturity.        II.            The security’s interest is paid semiannually, and the principal is paid at maturity.                III.                Investors must pay tax on interest earned annually.        IV.          Investors do not pay tax on interest earned until maturity.

Sorry, that is not the correct answer. The correct answer is: A.   I and III

Series 7 test designers are sly, and they often slip in two questions in one, as I’ve done here. Treasury STRIPS (T-STRIPS) are purchased at a discount and mature at par value. Therefore, investors don’t receive the principal and interest until maturity, so I is correct. However, the investor has to pay tax on the accretion (growth) each year, which means III is also true. The answer is A.

 

All of the following are considered regressive taxes EXCEPT         I.             sales taxes          II.            gift taxes             III.                excise taxes       IV.          estate taxes

Sorry, that is not the correct answer. The correct answer is: B.    II and IV only

Regressive taxes are ones in which all individuals (regardless of their tax bracket) are taxed at the same rate. Examples of regressive taxes are the sales tax that you pay when purchasing goods and excise taxes, which are taxes on goods such as gasoline, alcohol, and tobacco. Because this question is an exception question, you have to remove those two types of taxes, which leaves you with answer B. Gift taxes and estate taxes are progressive taxes; therefore, individuals in a higher income bracket pay a higher percentage.

 

Which of the following CMO tranches are backed by a companion tranche?         I.             PO          II.            PAC        III.           Z                IV.          TAC

You are correct. The answer is: B.             II and IV only

Companion (support) tranches absorb the prepayment risk associated with CMOs (collateralized mortgage obligations). All PAC (planned amortization class) and TAC (targeted amortization class) tranches are backed by companion tranches. PAC (planned amortization class) tranches have the most certain prepayment dates and are considered the safest of the CMOs. TAC (targeted amortization class) tranches are subject to more extension and prepayment risk than PAC tranches, so they receive a higher interest rate. As for the others, PO (principal only) tranches receive income from principal payments on the underlying mortgages. Z tranches do not receive payments until all other tranches are retired and are considered the most volatile of the CMO tranches.

 

8.1 TRANSFER AGENT

The transfer agent for a corporation is responsible for the following:

Cancels old and issues new stock certificates

Ensures the issuing of securities in the correct owner’s name

Maintains records of ownership

Resolves problems relating to lost, stolen or destroyed certificates

Decides what securities are acceptable and what is good delivery

– Correct number of shares sold

– Certificates in good condition

– Must be delivered in round lots of 100 or certificates divisible into

“100”. (e.g. 400 share delivery can be one certificate for 400 shares,

Four 100 share certificates, Eight 50 share certificates but not five 80

share certificates as 80 share certificates cannot be put together to

form 100 share lots

Delivers additional shares in the event of stock splits

Acts as the “Rights” agent during a rights offering

Acts as the “Warrant” agent for any warrant exercises

8.2 REGISTRAR

Ensures that a corporation does not have more shares outstanding than

have been authorized

Responsible for certifying that a bond represents a legal debt of the issuing

corporation

Unlike the transfer agent, must be independent of the issuing corporation

and is usually a bank or trust company

Safekeeping of securities

 

 

3. Which of the following values is assigned to a share of common stock at

the time of public issuance?

A. Par

B. Stated

C. CMV

D. A or B

3. (D) A par value or stated value, are both defined as an arbitrary amount

assigned to a share of stock prior to the initial public offering. The CMV or current

market value represents the price a share of stock is being bought or sold at in the

secondary market.

 

 

8. A right is a;

A. Long-term security

B. Short-term security

C. Is not marketable

D. Has no value

 

8. (B) A Right is a short-term security (usually lasts 30 days), has a value and certainly

is marketable.

 

 

 

9. Which of the following requires shareholder approval?

A. Stock splits

B. Management decisions

C. Officer appointment

D. Review of annual audits

 

9. (A) Shareholder approval is needed for stock splits, additional issues, board

vacancies, acceptance of a buy-out and a stock dividend.

 

10. PTS.com decided to pay a 5% common stock dividend to its shareholders.

The Board of Directors decided to distribute treasury stock to pay the

dividend. Which of the following concerning the stock dividend is correct?

A. This stock is not entitled to receive future dividends.

B. This stock is not entitled to vote.

C. This stock will be entitled to vote and to receive future dividends.

D. This stock is entitled to voting rights but not to future dividends

11. Preferred stock looks like which of the following securities?

A. Common stock

B. Bonds

C. Options

D. Warrants

Module 9: Corporate Review Questions

Page 45 NASD Series 7

12. During the first quarter the Board of Directors of PTS.com suspended

the dividend payable to preferred shareholders. Which of the following

statements would be true?

A. Common shareholders will still get their dividend.

B. Common shareholders will receive any missed dividends during the next

quarter.

C. Common shareholders will not receive any dividends until preferred shareholders

receive theirs.

D. Both A & B

13. Which of the following are benefits of owning preferred stock?

I. Fixed dividend

II. Dividend paid prior to common stock dividends

III. Preference over common shareholders in case of a company liquidation

A. I only

B. II only

C. II and III

D. I, II and III

14. Which of the following are types of preferred stock issues?

A. Convertible

B. Participating

C. Callable

D. All of the above

14. (D) Convertible, participating, callable and cumulative are all types of preferred

issues.

Definition of ‘Participating Preferred Stock’

 

A type of preferred stock that gives the holder the right to receive dividends equal to the normally specified rate that preferred dividends receive as well as an additional dividend based on some predetermined condition.

 

 

16. Corporations need shareholder approval for which of the following?

A. Declare a 20% stock dividend

B. Declare a cash dividend

C. Issue additional bonds for capital improvements

D. Repurchase outstanding stock for the employee stock purchase plan

 

16. (C) Shareholders are entitled to vote on whether to issue bonds because

bonds have a priority claim to corporate resources over common stock. The other

choices are made by the Board of Directors and would not require shareholder

approval.

 

20. Which of the following securities provides cash flow without any tax

benefits?

A. Limited partnerships

B. REITS

C. Municipal bonds

D. Tax free bonds

 

20. (B) REITs are cash producing securities without any tax benefit and can possibly

be obtained with limited partnerships.

 

22. In which of the following ways can a corporation declare dividends?

I. Cash

II. Stock

III. Product

IV. Stock of another corporation

A. I and II

B. III and IV

C. II and IV

D. I, II, III, IV

 

22. (D) A corporation can declare dividends in cash, stock, product and stock of

other companies. Stock of other companies is considered a property dividend.

 

 

23. Put the following securities in order from the shortest in duration to the

longest in duration.

I. Warrants

II. Rights

III. Options

A. III, I, II

B. II, III, I

C. I, II, III

D. II, I, III

 

23. (B) Rights have the shortest duration and warrants the longest duration.

Therefore, options would fall in between both.

 

 

24. The current market price of PTS Inc. is $50. PTS Inc. is about to issue

additional common stock. It’s going to take 4 rights to purchase one share

of common stock. The subscription price of the new issue is $35. What is

the value of a Right?

A. $15.00

B. $ 3.75

C. $ 3.00

D. None of the above

 

24. (C) Using the formula, M – S divided by N + 1 equals; M (market price) equals

$50 minus S (subscription price) $35 equals $15. Divide the $15 by N (number of

rights it takes to purchase one share of stock) 4 and add 1 for a total of 5. $15

divided by 5 equals $3.00.

 

 

Section 10: Primary Market

10.1 REGULATORY ISSUES FOR NEW SECU R I T IES

The following Acts were passed which help regulate the issuance of new securities.

They are as follows;

10.1.1 Securities Act of 1933

The Securities Act of 1933 regulates new issues of corporate securities sold

to the public. The Act requires issuers of new securities to provide enough

information for investors to make fully informed buying decisions. This information

must be registered with the federal government and published in a prospectus.

Obviously the Act prohibits any fraudulent activity in connection with

the underwriting and issuing of all securities.

This Act is also referred to as the Paper Act, Full Disclosure Act, New Issues

Act and Prospectus Act. This Act protects investors who buy new issues by

requiring the following;

Registration of new issues that are to be distributed interstate. Intrastate

(distribution within one state are exempt from registration)

Provide full and fair disclosure about itself and the offering

Regulating the underwriting and distribution of primary and secondary

issues

Providing criminal penalties for fraud in the issuance of new securities

10.1.1.1 Registration Statement

Issuers must file with the SEC under the Act of 1933 a registration statement

disclosing material information about the issue. Part of this registration

document becomes the Prospectus.

This registration statement must contain;

Names and addresses of company officers and directors.

Salaries and a five-year business history of the officers and directors.

Description of the issuer’s business.

Company’s capitalization including both equity and debt.

Amount of corporate securities that company officers and directors own.

Identification of investors who own 10% or more of the company.

Module 10: Primary Market

NASD Series 7 Page 52

Description of how the proceeds will be used.

Any legal proceedings in which the company is involved.

The issuer is responsible for the accuracy and adequacy of the required

information no matter who assists the issuer in preparing and filing of the

registration statement.

 

10.1.1.2 State Registration

State securities laws, known as Blue Sky Laws, require state registration

of securities, broker-dealers and registered representatives. Details of

state registrations are tested on the Series 63 and generally not on the

Series 7.

However, the following are the three basic ways of registering securities

with the states;

Filing (Notification): Some states allow some issues to register with

them by having the issuer notify the state of the issuer’s SEC registration.

However, this registration is only good for seasoned companies, not

new companies.

Coordination: This type of registration is typically used by IPOs. The

issuer registers simultaneously with the state and the SEC. Both registrations

become effective on the same date.

Qualification: This is the most difficult of all registrations. It is typically

used for intrastate offerings (exempt transaction) and for new companies

who don’t qualify under Filing or Coordination.

 

10.19 NEW ISSUES REVIEW QUESTIONS

1. The cooling off period for a new issues last approximately how long?

A. 10 days

B. 20 days

C. 30 days

D. 90 days

2. All of the following securities are exempt, EXCEPT;

A. Debentures

B. Private placements

C. US Government Bonds

D. Fixed Annuities

3. All of the following are types of underwritings, EXCEPT;

A. Standby

B. AON

C. Best efforts

D. Mini-max

4. Which of the following statements about a red herring is false?

A. The final offering price does not appear

B. Used to obtain indications of interest from prospective investors

C. A registered representative may send a copy of the company’s research report

with the red herring.

D. Additional information may be added to a red herring at a later date.

5. If the SEC has cleared an issue for sale, which of the following statements

is TRUE?

A. The SEC has endorsed the issue

B. The SEC has guaranteed the issue

C. The underwriter has filed a standard registration statement

D. The SEC has guaranteed the accuracy of information in the prospectus

6. The principal functions of an investment banker are to;

I. Provide a secondary market

II. Advise the issuer about alternatives in raising capital

III. Distribute securities to the public

IV Provide financing for an individual

A. I and II only

B. II and IV only

C. II and III only

D. I, II and IV only

 

6. (C): Choice A is incorrect as an investment banker is found in the primary market,

not the secondary market. The job of an investment banker is to advise the

issuer and help distribute shares to the public. It is also not the job of an investment

banker to provide financing for the issuer.

 

Each bond certificate contains the following information:

Name of the issuer

Type of bond

Principal amount

Issue date

Date of maturity

Call features, if any

Coupon rate (interest rate)

Interest payment dates

Reference to the trust indenture

 

4. How do you calculate the current yield on a bond?

A. Annual interest payments divided by the par value.

B. Annual interest payments divided by the cost of the bond.

C. Yield to the call divided by the par value.

D. Yields to the maturity divided by the par value.

 

4. (B): The current yield (CY) is calcualted by taking the annual interest or dividend

paid by the security and dividing by the investor’s cost.

 

8. A trust indenture spells out the covenants between;

A. The trustee and the issuer for the issuer’s benefit

B. The Issuer and the broker-dealer selling the bond

C. The trustee and the issuer for the benefit of the bondholder

D. The bondholder and the issuer

 

8. (C): The Trust Indenture Act of 1939 is meant to protect the investor. The covenants

are between the issuer and the trustee with the trustee looking out for the

benefit and rights of investors.

 

15. Which of the following Acts provide protection for the corporate investor

when buying bonds?

A. Securities Act of 1933

B. Securities Act of 1934

C. Trust Indenture Act of 1939

D. Investment Act of 1940

 

15. (C): The Trust Indenture Act of 1939 requires corporations to hire independent

trustees to protect investors.

 

17. All of the following are examples of secured bonds EXCEPT;

A. Morgage bonds

B. Collateral trust bonds

C. Income bonds

D. Equipment trust bonds

 

17. (C): Choices A, B and D are all examples of secured bonds. Income bonds

are high risk bonds that hardly ever pay interest.

 

18. Which uses actual days in the calculation of accrued interest?

A. Corporate bonds

B. U.S. Government bonds

C. Municipal bonds

D. Income bonds

 

18. (B): The U.S. Government uses actual days when computing days of accrued

interest. Corporate and municipal bonds use 30 day months when computing

days of accrued interest.

 

19. Treasury bills have a maximum maturity of;

A. 3 months

B. 6 months

C. 9 months

D. 12 months

 

23. A 5% bond maturing in five years and selling at 102 has a current yield of

4.9%. The approximate yield-to-maturity for this bond is:

A. 3.50%

B. 5.00%

C. 5.65%

D. 6.00%

 

23. (A): When a bond is purchased at a premium, the yield-to-maturity will always

be less than the nominal yield (coupon rate) and the current yield. The nominal

yield is given as 5.0% and the current yield is given as 4.9%. Therefore, of the

choices given, only 3.5% could possibly be correct. The other choices are greater

than the current yield and the nominal yield and are therefore incorrect.

 

25. Which of the following is more volatile?

A. 30 year par bonds

B. 25 year premium bonds

C. 25 year discount bonds

D. 20 year premium bonds

 

25. (C): Discount bonds are always more volatile than par or premium bonds.

 

20.3 MONEY MARKET SECURITIES

Examples of money market securities are:

Repurchase Agreements

Reverse Repurchase Agreements

Bankers’ Acceptances

Commercial Paper

Negotiable Certificates of Deposit (CDs)

Money Market Funds

 

two types of financial market involving financing. First, the

capital market, which serves as a source of intermediate-term to long-term financing

which is usually in the form if equity or debt securities with maturities of more

than one year.

The second, money markets, which provide short-term funds to corporations,

municipalities and the U.S. Government. Money market securities are debt issues

with maturities of one year or less.

20.2 L IQUDITY AND SAFETY

Fixed income securities with short-term maturities, typically one year or less

HIghly liquid

Relatively high degree of safety because they are short-term

Have very little chance of default

20.3 MONEY MARKET SECURITIES

Examples of money market securities are:

Repurchase Agreements

Reverse Repurchase Agreements

Bankers’ Acceptances

Commercial Paper

Negotiable Certificates of Deposit (CDs)

Money Market Funds

20.3.1 Repurchase Agreements

In a repurchase agreement (known as repo), a dealer raises cash by temporarily

selling some of the securities it holds with an agreement to buy back the

securities at a later date. So, a repo is simply an agreement between a buyer

and a seller to conduct a transaction (purchase), then to reverse that transaction

(repurchase) in the future.

Module 20: Money Markets

NASD Series 7 Page 126

20.3.1.1 Primary Uses of repos

U.S. Government and municipal securities dealers financing their inventories.

Commercial banks raising short-term funds

Federal Reserve adjusting member bank reserves

20.3.2 Reverse Repurchase Agreement

In a repo, a dealer agrees to sell its securities to a lender and buy them back at

a higher price in the future. In a reverse repurchase agreement a dealer agrees

to buy securities from an investor and sell them back later at a higher price.

The difference between repos and reverse repos is that the dealer initiates the

sale in a repo and an investor initiates the sale in a reverse repo.

20.3.3 Bankers’ Acceptances

A Bankers’ Acceptance (BA) is a short-term term draft with a specified payment

date drawn on a bank used to facilitate foreign trade. It is essentially a postdated

check or line of credit. The maximum payment date of a BA is 270 days

(nine months).

BAs are used extensively to finance international trade in that they pay for

goods and services in a foreign country.

They are actively traded in the money market and are considered high quality

since they are secured by a guarantee from a large bank and also by the

goods originally purchased by the importer.

20.3.4 Commerical Paper

Commercial paper is short-term unsecured corporate date with a maximum

maturity of 270 days (nine months).They are primarily used to raise cash to

finance accounts receivable and seasonal inventiry gluts. Commercial paper

interest rates are lower than bank loan rates.

Primary buyers of commercial paper are money-market funds, commercial

banks, pension funds, insurance companies, corporations and nongovernmental

agencies.

Normally issued in book entry form.

Usually issued by companies with excellent credit ratings.

Moody’s and standard and Poor’s issue ratings for commercial paper.

Module 20: Money Markets

Page 127 NASD Series 7

20.3.5 Negotiable Certificates of Deposit

Large commercial banks can borrow money by issuing CDs. These are shortterm,

interest bearing securities backed by the general credit of the issuing

bank.

Known as jumbo CDs with minimum denominations of $100,000

Typically trade in denominations of $1,000,000 or more

A normal negotiable CD would not be insured by the FDIC if its size is

greater than $100,000 as it would exceed the FDIC’s insurance limit

Accrued interest is included in the price of a negotiable CD

20.3.6 Money Market Funds

A money market is a type of open-end investment company (known as a

mutual fund) that invests in short-term debt securities and money market securities.

Must be registered under the Securities Act of 1933 and the Investment

Company Act of 1940.

Must be sold with a prospectus.

Minimum initial investments will be stated in the prospectus.

Section 22: Tax Advantaged Products

Municipal Securities

22.1 MU N I C I PAL SECURITIES OVERVI EW

Test Clue: There are a large number of Series 7 questions asked about

municipal securities and the MSRB rules.

Municipal securities are debt instruments that are for the most part exempt from

federal taxation while some issues are also exempt from state and local taxation.

Municipal bonds are issued by state or local government of U.S. Territories,

authorities and special districts.

Investors purchasing these securities are actually lending money to the issuers for

the purpose of public works and construction projects such as;

Hospitals

Roads

Civic Centers

Sewer Systems

Airports

Let’s take a look at some general features of Municipal Securities;

Interest is exempt from Federal taxation. This is based on the Doctrine of

Reciprocal Immunity which was established by a Supreme Court Decision in

1895. The doctrine specifies that a level of government can only tax the interest

of its own issues. So, municipal securities may be taxed by the municipal

level such as state and local governments, but not the federal government.

Capital gains are always taxable even though the interest may not be

Because of the inherent tax reduction of municipal securities, they tend to

pay lower interest rates than do corporate issuers or U.S. Government

issues.

Municipal securities can be issued as fully registered in certificate form or by

Book Entry, similar to U.S. Government Treasuries

All municipal bonds must have a Legal Opinion printed on the face of the

bond. This opinion is issued by legal counsel hired by the issuer to make sure

that the bond issued is legally binding on the issuer and the interest is exempt

from Federal tax.

Module 22: Tax Advantaged Products

NASD Series 7 Page 132

– A Qualified Opinion by bond counsel is not desirable as the opinion

is indicating there may be a problem. An example could be that interest

from the bond may be taxable under new Federal tax rulings.

– Most long-term bonds are issued as serial issues with interest paid

semi-annually.

Broad categories of municipal securities include general obligation bonds

(GO), revenue bonds, special types of bonds and short-term notes.

22.2 LEGAL OP I N I ON

The issuer hires a bond counsel, also known as a bond attorney to write a legal

opinion on the bond issue. The responsibilities of the bond counsel are as follows;

Examine the indenture and securities laws

Determine that the issue is valid and binding on the issuer.

Determine that the issue is tax exempt.

Issues an unqualified or qualified legal opinion.

Unqualified: This is the good one. It means that the issuer meets all

conditions without any restrictions. This is sometimes called a clean

legal opinion as there are no contingencies that would keep the issue

from being valid or tax exempt.

Qualified: This means that the issuer meets conditions with certain

restrictions. Restrictions can result because of a lien on the property

or a pending lawsuit, as examples.

22.3 BOND INSURANCE

Municipal bonds that are insured guarantee the timely payment of principal and

interest by a number of different insurance agencies. If a municipality was unable

to pay its debt, the insurance company would.

As an insured bond is an additional benefit for investors, coupon rates on insured

bonds tend to be lower than non-insured bonds.

Some example of companies that insure municipal bonds are as follows;

AMBAC – American Municipal Bond Assurance Corporation

MBIA – Municipal Bond Insurance Association Corporation

FGIC – Financial Guaranty Insurance Corporation

FSA – Financial Security Assurance Corporation

 

Test Point: The exam usually asks for time limits

on advertising (3 years) and customer complaints (6 years)

 

Test Taking Note: The exam question must tell you if they are talking about an

OID municipal bond. If OID is not mentioned in the test question then assume it

is a regular Secondary Municipal Bond and is taxable.

 

Section 26: Tax Advantaged Review

Questions

1. All of the following are general features of municipal securities EXCEPT;

A. Capital gains are never taxable.

B. Tend to pay lower interest rates as compared to corporate debt securities.

C. Interest is exempt from federal taxation.

D. Municipal bonds must have a legal opinion attached.

2. Most municipal bonds pay interest;

A. Annually

B. Semi-annually.

C. Quarterly.

D. Monthly.

3. All of the following are examples of municipal bond insurers EXCEPT;

A. AMBAC

B. MBIA

C. FGIC

D. NASD committee on tax-free stuff

4. Which type of municipal bond relies on ad valorem taxes?

A. GOs

B. Revenue

C. IDR

D. Notes

5. A double barreled bond is first a;

A. GOs

B. Revenue

C. IDR

D. Notes

6. Which of the following municipal bonds are considered the “safest”?

A. GO

B. Revenue

C. IDR

D. Income

7. A feasibility study is generally required with which of the following?

A. GOs

B. Revenue

Module 26: Tax Advantaged Review Questions

NASD Series 7 Page 180

C. IDR

D. Notes

8. Which covenant promises to maintain the equipment and facilities?

A. Rate Covenant

B. Sinking Fund Covenant.

C. Maintenance Covenant.

D. Catastrophic Clause.

9. Which type of special bond is backed by U.S. Government subsidiaries?

A. GO

B. Revenue

C. Public Housing Authority

D. Special Notes

10. Municipal notes are rated by;

A. Standard & Poor’s

B. Diana’s Rating Service

C. Moody’s Ratings

D. Munifacts

11. All of the following are correct regarding Official Statements, EXCEPT;

A. Must be sent with advertisements

B. Creditworthiness will be indicated

C. Must be filed with the SEC

D. Prepared by the issuer

12. All of the following indexes are compiled in the Bond Buyer, EXCEPT;

A. 40 Bond Index

B. 20 Bond Index

C. 11 Bond Index

D. 9 Bond Index

13. Which of the following are the types of municipal securities offerings?

I. Negotiated

II.Competitive

III.Sharing

IV.Fungible

A. I only

B. I and II only

C. II and III only

D. I, II, III and IV

Module 26: Tax Advantaged Review Questions

Page 181 NASD Series 7

14. Which of the following is the underwriting syndicate that is known as the

undivided account?

A. Eastern

B. Northern

C. Southern

D. Western

15. Which of the following is the correct allocation of orders for a new

municipal offering?

A. PSDM

B. PGDM

C. GDMP

D. SGPM

16. When a broker-dealer acts as an “Agent”, compensation would be a;

A. Commission

B. Markup

C. Markdown

D. Cross trade sum

17. An apprenticeship of how many days is required by persons entering the

securities industry when selling municipals?

A. 30 days

B. 45 days

C. 60 days

D. 90 days

18. Municipal bonds issued by Puerto Rico are generally;

A. Taxable

B. Federally tax-free

C. Double tax-free

D. Triple tax-free

19. When a bond is sold to generate a tax loss and the same bond is purchased

back in 10 days, what has been violated?

A. Back tax rule

B. Wash sale rule

C. Clean sale rule

D. Capital gain rule

Module 26: Tax Advantaged Review Questions

NASD Series 7 Page 182

20. If a bond is purchased at 110, what is the tax basis according to the IRS

after 5 years?

A. 115

B. 110

C. 108

D. 105

21. Which of the following tax characteristics are impossible to avoid?

I. Continuity of life

II. Free transferability of shares

III.Limited liability

IV.Centralization of Management

A. I and II only

B. II and III only

C. III and IV only

D. I and IV only

22. What is the Form number that reports all tax consequences when investing

in limited partnerships?

A. 8K

B. 10K

C. K-1

D. 10Q

23. All of the following items increase the basis, EXCEPT:

A. Contributions of cash

B. Contributions of property

C. Distributions of cash

D. Recourse debt

24. Which of the following is NOT a feature of an investment in Raw Land?

A. Substantial cash flow

B. No depreciation of land

C. Risk of swamp land

D. Risk of zoning changes

25. New Construction is;

A. Low risk

B. High risk

C. No risk

D. Risk-free

Module 26: Tax Advantaged Review Questions

Page 183 NASD Series 7

Rules Review Answers

1. (A) Capital gains are always taxable, even for tax-free bonds. The tax-free

refers to the income received from the coupon only. Choices B, C and D are features.

2. (B) Like most bonds, interest is paid twice a year or semi-annually.

3. (D) Choices A, B and C are companies that insure municipal bonds. FSA is also

an insurer of municipal bonds.

4. (A) Ad valorem taxes are real estate taxes that support payment of interest for

General Obligation (GO) bonds.

5. (A) While a double barreled bond is a combination of a revenue and a general

obligation bond, for test purposes they are GO bonds first.

6. (A) GO bonds are generally considered the safest because of the ability to raise

the ad valorem taxes.

7. (B) Feasibility studies are generally required with revenue bonds.

8. (C) The Maintenance Covenant promises to maintain the equipment and facilities.

9. (C) Public Housing Authority bonds are backed by the U.S. Government.

10. (C) Moody’s rate municipal notes.

11. (C) On the contrary, the Official Statement does NOT have to be filed with the

SEC.

12. (D) The Bond Buyer compiles all the choices except Choice D. The Bond

Buyer also compiles the Revdex 25 Index.

13. (B) There are two types of municipal offerings, Negotiated and Competitive.

14. (A) The Eastern syndicate is known as the undivided account.

15. (B) The correct allocation for orders is P G D M. In corporate bond issues the

acronym changes to P S D M. For exam purposes, just learn these acronyms.

16. (A) Agents charge a commission while dealers charge a markup or markdown.

17. (D) 90 days by rule.

Module 26: Tax Advantaged Review Questions

NASD Series 7 Page 184

18. (D) Generally, municipal bonds are only exempt from federal taxation. Some

states offer double tax free bonds (free from both federal and state) while bonds

issued by U.S. possessions are all triple tax free (free from federal, state and

local).

19. (B) This is clearly a violation of the Wash Sale Rule.

20. (B) A bond bought at a premium of 100 dollars would amortorize over its life

until maturity. With a bond amoritizing over a 10 year period, the annual change to

the tax basis would be $10 per bond. So, after five years the bond has decreased

in tax basis by $50 (5 years times $10 per bond) to 105.

21. (C) Limited liability and centralization of management are impossible for a

partnership to avoid.

22. (C) The K-1 reports all tax consequences for limited partnerships.

23. (C) Most distributions of dividends or tax benefits reduces the current tax

basis.

24. (A) Not hardly! There generally is NO cash flow.

25. (B) New construction is considered high risk because of its potential inherent

problems.

 

Exam Alert: One of the major differences between UITs and open-end

management companies is that the UIT is NOT actively managed and

therefore does not have fund managers or a management fee.

 

28.3.1 Diversified Investment Company

Invest at least 75% of its assets in such a way;

That no more than 5% of their total assets can be used to purchase any

one stock and;

When purchasing stock no more than 10% of the outstanding voting

stock can be purchased

 

Exam Alert: Remember for the exam, the Act of 1940 requires at least

40% of the directors to be independent or noninterested persons.

 

Section 32: Review Questions

1. Investment companies are regulated by which of the following?

A. Investment Advisors Act of 1970

B. Investment Company Act of 1940

C. SIPC

D. Securities Act of 1934

2. All of the following are investment companies EXCEPT:

A. Face amount certificates

B. Unit investment trusts

C. Mutual funds

D. Management companies

3. Which of the following is one of the major differences between an openend

company and a unit investment trust?

A. Regulated by the Investment Company Act of 1940

B. Redemption of shares

C. One of the investment companies

D. Portfolio not actively managed

4. Which of the following are management companies?

I. Face amount certificates

II. Open-end companies

III. Unit investment trusts

IV. Closed-end companies

A. II and IV

B. I and III

C. II and III

D. I, II, III, IV

5. Selling closed-end shares can be accomplished by;

A. Redemption through the fund company

B. Redemption through a broker-dealer

C. Trading in the stock markets

D. Trading with the mutual fund company

Module 32: Review Questions

NASD Series 7 Page 240

6. The major difference between an open-end company and a closed-end

company is the;

A. Net Asset Value

B. Capitalization

C. Public Offering Price

D. Investment Company Act of 1940

7. A diversified investment company cannot own more than what percent of

the voting stock of a corporation?

A. 5%

B. 10%

C. 50%

D. 75%

8. Detailed information about a fund can be found in the;

A. Prospectus

B. Company guidelines

C. Broker-dealer’s charter

D. Corporate charter

9. Open-end fund shares can be purchased by investors with what type of

shares?

A. Fractional only

B. Full only

C. Full and fractional

D. Margined

10. Which of the following are examples of fund investment objectives?

I. Safety of principal

II. Income

III. Capital gains

IV. Preservation of capital

A. I and II

B. I and IV

C. II, III, IV

D. I, II, III, IV

11. Which type of analyst looks inside the company before making any recommendations?

A. Fundamentalist

B. Technical analyst

C. Fund specialist

D. Chartist

Module 32: Review Questions

Page 241 NASD Series 7

12. The principal underwriter of an open-end investment company is also

known as the;

A. Registrar

B. Transfer agent

C. Custodian

D. Sponsor

13. Under the Investment Company Act of 1940, shareholders must receive

financial reports;

A. Quarterly

B. Annually

C. Semi-annually

D. Monthly

14. The fees earned by the investment manager depends on the;

A. Net profit of the fund

B. Salary earned

C. Net assets of the fund

D. Volume of new shares purchased during a 30 day period

15. Who in a corporation establishes dividend and capital gain policies?

A. Corporate officers

B. Board of Directors

C. Shareholders

D. NASD

16. Who is responsible for issuing, redeeming and canceling share

certificates?

A. Custodian bank

B. Board of Directors

C. Transfer agent

D. Corporate officers

17. The holder of money for a mutual fund company is usually a;

A. Savings and loan company

B. Another mutual fund company

C. Commercial bank

D. Broker-dealer

Module 32: Review Questions

NASD Series 7 Page 242

18. Shareholders get to vote through the use of a;

A. Proxy

B. Certificate of election

C. Broker-dealer voting form

D. Corporate certification

19. Which of the following is NOT an issue that shareholders get to vote

for?

A. Changes in the fund’s investment objectives

B. Appointment of corporate officers

C. Board of Directors

D. Eliminating 12b-1 fees

20. When selecting a board of directors what percent can be interested

persons?

A. 20%

B. 40%

C. 50%

D. 60%

21. When calculating net investment income per share, the investment company

includes;

A. Both gross dividends and gross interest

B. Capital gains

C. Both gross dividends and gross interest minus operating expenses

D. Only dividends

22. A return of capital;

A. Is not a taxable event

B. Is a taxable event

C. Is not available with mutual fund distributions

D. Is the same as a capital gains distribution

23. When share prices increase, but shares are not sold yet, the investor

has;

A. Realized appreciation

B. Realized depreciation

C. Unrealized appreciation

D. Unrealized depreciation

Module 32: Review Questions

Page 243 NASD Series 7

24. Investment companies distribute capital gains;

A. Monthly

B. Quarterly

C. Semi-annually

D. Annually

25. Which of the following is NOT an advantage of a mutual fund

investment?

A. Exchange privilege between the different funds of a fund family

B. Fund company safekeeping of shares

C. Use of fund shares as collateral for a bank loan

D. Investor decides which of the underlying securities are to be purchased with his

invested dollars.

26. A mutual fund’s expense ratio is its expenses divided by;

A. Income

B. Average net assets

C. Net asset value

D. Public offering price

27. Which of the following is NOT an example of an income fund?

A. Municipal bond fund

B. International bond fund

C. Equity fund

D. GNMA fund

28. Which of the following equity based funds also have income as their primary

investment objective?

A. Utility companies

B. Aggressive equity

C. Equity income

D. A & C

Go to the next page!

Module 32: Review Questions

NASD Series 7 Page 244

29. Which of the following is (are) examples of a mutual fund concentrating

on one industry?

I. Energy

II. Biotech

III. Blue chip

IV. Banking

V. S&P Index

A. I and II

B. I, II, III

C. I, II, IV

D. II, III, IV, V

30. Which of the following is NOT an example of a security found in money

markets?

A. Bankers Acceptances

B. CDs

C. Commercial paper with a maximum maturity of 310 days

D. Repos

31. Municipal bond funds are generally:

A. Double tax free

B. Federal tax free

C. Triple tax free

D. Taxable

32. Which type of foreign securities fund can invest in companies residing

in the United States?

A. International fund

B. Global fund

C. Korea fund

D. No foreign fund can have U.S. securities

33. Precious metals funds can include which of the following?

I. Gold

II. Brass

III. Silver

IV. Platinum

V. Copper

A. I, II, III

B. II, IV, V

C. III, IV, V

D. I, III, IV

Module 32: Review Questions

Page 245 NASD Series 7

34. Tax exempt bond funds may invest in which of the following?

A. Common stock

B. Preferred stock

C. Municipal bonds

D. Commercial paper

35. Which of the following fund delivery methods represents sales by brokerage

firms?

A. Fund to underwriter to dealer to investor

B. Fund to underwriter to investor

C. Fund to investor

D. Fund to fund

36. The NAV is also known as the;

A. Offer price

B. Bid price

C. Public Offering Price

D. Sales price

37. Peggy Sue signed an LOI stating that she would purchase $20,000 worth

of the PTS Growth Fund. After 13 months she had invested only $17,000.

What effect will her actions have?

A. She must pay the additional dollars or risk a law suit

B. She will be charged the maximum possible of 8 ½%

C. She will be charged whatever sales charge she is entitled to for the actual

amount she invested

D. Her account will immediately be closed and subject to federal penalty

38. The maximum sales charge allowed for regular mutual funds is;

A. 5%

B. 8%

C. 8½%

D. 9%

39. Letters of intent can be backdated up to how many days?

A. 30

B. 45

C. 60

D. 90

Module 32: Review Questions

NASD Series 7 Page 246

40. With the NAV at $11.00 and the POP at $11.50, what will an investor

receive when redeeming 1000 shares?

A. $11,000

B. $11,500

C. $11,250

D. $11,000 less a redemption fee

41. As a registered representative wanting to help your clients make the

most money possible you call 10 of your best clients to tell them about a

dividend being paid in two weeks by a mutual fund. You suggest strongly

that they purchase the fund now to take advantage of the dividend being

paid. Which of the following statements best describes this situation?

A. Your suggestion is great as it’s in the best interest of your clients

B. You are violating the selling dividends rule

C. You must limit the sales to only 5 clients a month

D. It is not against NASD rules to “sell dividends”

42. The ex-dividend day for mutual funds is set by the;

A. NASD

B. SEC

C. Mutual fund

D. Investment Company Act of 1940

43. Sales charges are stated as a percentage of the;

A. Bid price

B. Offering price

C. NAV

D. Average of the bid and offering price

44. Contingent deferred sales charges are found in what class shares?

A. A

B. B

C. C

D. D

45. Which share class is known as level load?

A. A

B. B

C. C

D. D

Module 32: Review Questions

Page 247 NASD Series 7

46. With rights of accumulation, any quantity of securities an investor owns

could be based on which of the following?

I. Total purchase of shares of the actual public offering prices

II. Current net asset value of the securities

III. Current value of all redeemable securities with the same fund family

IV. Current public offering price of the securities

A. I and II only

B. II and III only

C. II only

D. I, II, III and IV

47. Billy Bob wishes to redeem 1000 shares of the PTS Aggressive Growth

Fund. The closing NAV is $12 while the POP is $12.38. The fund has a

redemption fee of 1%. What is the net amount Billy Bob will receive from

the fund company?

A. $12,000

B. $12,380

C. $11,880

D. $12,256

48. Which of the following newspaper symbols indicates a 12b-1 marketing

fee?

A. nl

B. r

C. p

D. t

49. Which of the following would not qualify for breakpoints under the definition

of a “person”?

A. Corporations

B. Pension fund

C. Investment club

D. Employee benefit plan

50. Class A shares of a mutual fund have a;

A. Asset based fee

B. Front-end fee

C. Level load fee

D. Back-end fee

Module 32: Review Questions

NASD Series 7 Page 248

Module 32: Review Questions

Page 249 NASD Series 7

32.1 REVIEW ANSWERS

1. (B) Investment Company of 1940 by definition.

2. (C) Use caution on the exam! Investment companies consist of Face Amount

Certificates, Unit Investment Trusts and Management Companies. Management

companies are made up of open-end and closed-end funds. Mutual funds are not

mentioned anywhere in the Investment Company Act of 1940. Open-end companies

have been given the name mutual funds many years ago by marketing firms.

3. (D) One of the major differences between open-end companies and unit

investment trusts is that UITs are NOT managed. A portfolio of investments are

selected but not managed.

4. (A) All four are investment companies but only two are management companies,

the open-end and closed-end companies as they are actively managed.

5. (C) While open-end shares are redeemed through the mutual fund company,

closed-end shares are bought and sold in the secondary stock markets after the

initial public offering.

6. (B) The major difference between open-end and closed-end companies is the

capitalization structure. Open-end always have shares to offer but closed-end

funds have a limited number of shares available, just like stocks.

7. (B) To be diversified a mutual fund company must invest at least 75% of its

assets in such as way that no more than 5% of its assets can purchase the stock

of any one company and when it does buy stock of a company it cannot own more

than 10% of its voting stock.

8. (A) All information about a fund can be found in the prospectus.

9. (C) One of the flexible benefits of owning mutual fund shares is being allowed

to purchase with a specific dollar amount versus a number of shares. By purchasing

with a specific dollar amount full and fractional shares can be purchased.

10. (D) All are examples of fund investment objectives.

11. (A) The Fundamentalist looks inside a company to gather information while

the Technical Analyst (Chartist) uses outside factors to make their recommendations.

12. (D) The principal underwriter is also known as the sponsor or distributor.

13. (C) Semi-annually reports with the annual report needing to be audited.

14. (C) Fees earned by the investment manager are based on the assets of the

fund.

Module 32: Review Questions

NASD Series 7 Page 250

15. (B) The Board of Directors is responsible for establishing dividend and capital

gain policies.

16. (C) The Transfer Agent is responsible for issuing, redeeming and canceling

share certificates.

17. (C) The custodian of the fund’s assets is usually a commercial bank.

18. (A) Proxies are used by all corporate shareholders.

19. (B) Appointment of officers is a function for the Board of Directors.

20. (D) The Investment Company Act of 1940 requires that 40% of the board of

directors be noninterested leaving 60% as interested members.

21. (C) To arrive at the net investment income a fund company takes the gross

dividends and gross interest and subtracts the operating expenses.

22. (A) A return of capital is NOT a taxable event as it is neither a dividend or capital

gain.

23. (C) If share prices go up but are not sold yet the investor has unrealized

appreciation. If share prices are lower but not sold yet the investor has unrealized

depreciation. Realized appreciation or depreciation can only be determined once

the shares are sold.

24. (D) Fund companies are required to distribute long-term capital gains annually.

Short-term capital gains (capital gains representing less than a year), taxed

like dividends (at the investor’s tax bracket), can be distributed monthly, quarterly,

semi-annually or annually.

25. (D) Investors cannot make investment decisions regarding individual securities

purchased within a fund as this is the job of the fund manager.

26. (B) By definition, the expense ratio is calculated by taking its expenses and

dividing them by the average net assets.

27. (C) Choices A, B and D are examples of bond funds while choice C is an

example of an equity fund.

28. (D) Utility companies and equity income funds are made up of stock as the

underlying security. However, they also have income as their primary investment

objective. An aggressive equity fund has growth and not income as its primary

objective.

29. (C) Energy, biotech and banking are examples of sector funds, also known as

specialized funds which concentrate on one industry. Blue chip and the S&P 500

are well diversified funds with no concentration in only one industry.

Module 32: Review Questions

Page 251 NASD Series 7

30. (C) To be in a money market as a short-term debt instrument, commercial

paper must have a maximum maturity of 270 days, not 310 days. All the others

can be found in a money market.

31. (B) Municipal bond funds are typically only federally tax free. Some states

allow a double tax free status but the investor must be a resident of that state and

the bonds issued there also. Triple tax free bonds are issued by the possessions

of the United States such as Guam, Puerto Rico and the Virgin Islands.

32. (B) Global funds can invest in any country in the world, including the United

States, while International funds must invest in companies outside the United

States. The Korea fund is an example of a Single Country fund which will invest in

countries outside the United States.

33. (D) Gold, silver and platinum and palladium are examples of precious metals

that can be found in precious metal funds. Brass and copper are not considered

precious metals.

34. (C) Municipal bonds are usually federally tax free while the others are not.

35. (A) The fund sells shares to their underwriters who in turn sell the shares to a

brokerage firm and the brokerage firm sells shares to retail customers. Choice B

indicates a sale to the fund’s underwriter and then the in-house brokers sell

shares directly to the customers without using a brokerage firm. Choice C indicates

a no-load situation where the fund company sells shares directly to the customers.

Choice D is not valid.

36. (B) The NAV is also known as the bid price while the POP is also known as

the ask price.

37. (C) This is a non-binding contract. Therefore, Peggy Sue will get a discount

if she has reached any of the breakpoint levels. If not, she will pay the full sales

charge.

38. (C) 8 ½% is the maximum allowed as long as rights of accumulation, reinvesting

at the NAV and LOIs are allowed. If not, the maximum sales charge is

6½%. Contractual plans allow a maximum of 9%.

39. (D) 90 days by definition.

40. (A) Funds are redeemed at the NAV. The NAV is $11 times 1,000 shares

which equals $11,000.

41. (B) Selling dividends is prohibited by the NASD. On ex-dividend day the

NAV of the fund will fall the amount of the dividend and your client will have a

potential tax consequence.

42. (C) The ex-dividend for mutual funds is set by the fund company while the

NASD establishes the ex-dividend day for stocks.

Module 32: Review Questions

NASD Series 7 Page 252

43. (B) By definition

44. (B) Contingent deferred sales charges are found in Class B shares.

45. (C) Level load shares are Class C

46. (D) Actually any of the choices would be okay as long as the mutual fund

company approves the method.

47. (C) Shares are redeemed at the NAV. The NAV of $12 times 1,000 shares

equals $12,000. Next the redemption fee of 1% is deducted before the investor

receives the distribution of $12,000 minus 1% or $11,880.

48. (C) P indicates a 12b-1 fee while nl is a no-load, r is a redemption fee and t

represents a redemption fee plus a 12b-1 plan in effect.

49. (C) Investment clubs never qualify for mutual fund breakpoints.

50. (B): Class A shares have a front-end load feature. This means the sales

charge is deduted directly from the investment at the time of purchase.

 

34.19 OPTION REVIEW ONE

This would be a good place to review and reinforce the various option points

you’ve learned by completing the following quiz questions. The answers will follow

at the end of the last question. Your goal is to answer all the questions without

looking at the answers.

1. A customer is long 1 XYZ Feb 40 call @ 5. The CMV of XYZ is 43. This

contract is;

A. Out of the money

B. In the money

C. At the money

D. No idea

2. The breakeven for the option contract in Question #1 is;

A. 35

B. 48

C. 45

D. 38

3. What is the Time Value for the option contract in Question #1?

A. 2

B. 3

C. 4

D. 5

A customer buys 1 Dell Jun 30 put @ 4 when Dell closes at 27. Use this

option contract to answer the next 2 questions.

4. The intrinsic value of the contract is;

A. 1 point

B. 2 points

C. 3 points

D. 5 points

5. The time value of the contract is;

A. 1 point

B. 2 points

C. 3 points

D. 5 points

Go to the next page for answers and rationale

Module 34: Options

Page 267 NASD Series 7

34.19.1 Option Review One Answers

1. (B) As the market price is at 43 and the strike price at 40, this option contract

is in the money by 3 points, the difference between the strike price and current

market value of the underlying security.

2. (C) When calculating a breakeven for a call, take the strike price and add

(call up) the premium. The strike price is 40 and the premium is 5 giving a total

breakeven of 45.

3. (A) Use the acronym P = I + T. This option is in-the-money by 3 points (strike

price of 40 and a current market value of 43). Intrinsic value is the same as inthe-

money. So, with a premium of 5, P = I + T, 5 = 3 + 2 (time value).

4. (C) A put with a strike price of 30 and a current market value of 27 would be

in the money (intrinsic value) by 3 points (30 – 27).

5. (A) Once again use P = I + T (4 = 3 + 1 (time value).

 

guarantees and standardizes option contracts?

A. NASD

B. OCC

C. SEC

D. FRB

2. Options settle in;

A. 1 day

B. 2 days

C. 3 days

D. 5 days

3. Equity options expire on the;

A. Third Friday

B. Third Friday after the third Saturday

C. Saturday following the third Friday

D. Third Saturday

4. Equity options have a technical maximum contract life of;

A. 8 months

B. 9 months

C. 11 months

D. 12 months

5. Rocky Road Inc. declares a 3:1 split. Jo Bob owns 1 Rocky Road Mar 75

Call. The new position would be which of the following?

A. 1 Rocky Road Mar 75 call with 300 underlying shares.

B. 3 Rocky Road Mar 75 calls

C. 4 Rocky Road Mar 75 calls

D. 2 Rocky Road Mar 75 call with 200 underlying shares each

6. Which of the following cannot trade for their own accounts?

A. OBO

B. Market makers

C. ROPs

D. Floor specialists

Module 35: Options Rules and Regulations

NASD Series 7 Page 274

35.6.1 Option Review Two Answers

1. (B) The Options Clearing Corporation guarantees and standardizes option

contracts.

2. (A) Equity option contracts settle in one business day.

3. (C) You must memorize this! Equity options expire on the Saturday following

the third Friday.

4. (B) Equity options have a maximum contract life of 9 months.

5. (B) With an even stock split the number of contracts with 100 shares of

underlying shares would increase.

6. (A) Order Book Officials cannot trade for their own account.

 

36.5 OPTION REVIEW THREE

The purpose of the following questions is to review all the aspects of equity

options which were provided in the Equity Options Section.

1. An investor buys 1 Dell May 50 call @ 4.50. The investor’s maximum

potential gain is;

A. $450

B. $4,550

C. $5,000

D. Unlimited

2. Using the option contract in Question 1, what is the investor’s potential

maximum loss?

A. $450

B. $4,550

C. $5,000

D. Unlimited

3. Using the option contract in Question 1, what is the breakeven?

A. 50

B. 54.50

C. 4.50

D. Unlimited

4. In buying call options as compared to buying the underlying stock, which

of the following is NOT an advantage?

A. Buying a call allows greater leverage than buying the underlying stock.

B. Buying a call would require a smaller capital commitment.

C. Buying a call has a lower dollar loss potential than buying the stock.

D. The call has a time value beyond an intrinsic value that gradually dissipates.

5. An investor buys 1 PTS Dec 40 put @ 3. What is the investor’s maximum

potential loss?

A. $300

B. $5,000

C. $4,700

D. $5,300

Continued on the next page

Module 36: Equity Options

NASD Series 7 Page 284

6. The holder of a long put will realize a profit upon exercise of the option if

the price of the underlying stock;

A. Falls below the exercise price

B. Falls below the exercise price minus the premium paid

C. Exceeds the exercise price

D. Exceeds the exercise price plus the premium paid

7. All of the following ways can be used to close an option position EXCEPT;

A. Exercise

B. Expire worthless

C. Trade the option contract

D. Attaching the contract

8. Which of the following ways to close an option position is usually based

on intrinsic and time value?

A. Exercise

B. Expire worthless

C. Trade the option contract

D. Attaching the contract

9. Billy Bob sells 1 GM Mar 35 put @ 3. What is Billy Bob’s maximum potential

gain?

A. $3,500

B. Unlimited

C. $300

D. $3,200

10. Mary Jane, one of your best clients, wants to purchase Dell stock, currently

trading at 28. She expects that the price of Dell stock will rise slowly

in the near future. But, she doesn’t want to pay the current market value.

Instead, she wants to own the stock below its market value. Which of the following

strategies would you recommend?

A. Buy a put and exercise the option

B. Write a put at 25 and buy the stock

C. Buy a call and exercise the option

D. Write a call at 25

Module 36: Equity Options

Page 285 NASD Series 7

36.6 OPTION REVIEW THREE ANSWERS

1. (D) Anytime a call is purchased the maximum gain is unlimited as shown in the

T-chart below.

450 Unlimited

2. (A) The most any invest can lose when purchasing an option is the premium

paid. This is displayed in the above T-chart in the “out” column.

3. (B) The breakeven for call options is found by taking the strike price (50) and

adding (call-up) to it the premium of 4.50 for a total of 4.50. A breakeven is always

found on a one share basis, not times 100.

4. (D) Statements A, B and C are all true statements as to why buying a call would

be advantageous over buying the underlying stock. However, choice D is not an

advantageous as options are wasting assets and the life of a call or put will end at

a set date.

5. (A) Once again, the most an investor can lose when buying an option is the premium.

In this question the premium is 3 so the maximum that can be lost is $300.

6. (A) When buying a put the investor would be happy if the price of the underlying

security would fall. The more it falls below the strike price the happier the investor

will be as the put premium increases.

7. (D) The three ways of closing an option contract is to let the contract expire

worthless, trade the option or exercise the option. There is no such thing as

attaching the contract.

8. (C) The value of an option contract for trading purposes is based on the premium.

The premium is made up of intrinsic value (amount the contract is in the

money) and any time value (time for something to happen). Volatility also contributes

to the value of the premium but cannot be measured for exam purposes.

9. (C) The maximum potential gain for any option writer is the premium.

10. (B) This is the classic example of a reason to write puts, hoping to buy the

stock below market price. Writing the put gives the investor a chance of buying the

stock at the strike price of 25 if the stock falls in value and the put is exercised by

the holder of the put. In addition the writer gets the premium so if the contract is

exercised the writer will own the stock less than the strike price of 25.

 

37.1.5 Straddle/Combination Review Questions

1. Your customer buys a PTS Apr 30 call. To establish a straddle your

customer would;

A. Buy a PTS Apr 30 put

B. Sell a PTS Apr 30 call

C. Buy a PTS Apr 30 call

D. Sell a PTS Apr 30 put

2. A long straddle is profitable in a:

I. Rising market

II. Falling market

III. Stable market

A. I only

B. II only

C. III only

D. I and II

Use the following information to answer the next 3 questions.

Your best customer, Diana Lynne, buys 1 GE Mar 30 call @ 5 and

buys 1 GE Mar 30 put @ 4 when the market price of GE is 31.

3. The maximum gain for this position is;

A. $900

B. $300

C. $3,100

D. Unlimited

Go to the next page!

Straddles and Combinations Summary

Calculation Long Straddle or

Combination

Short Straddle or

Combination

Max Gain Unlimited Total Premiums Paid

Max Loss Total premiums received Unlimited

Breakevens Call: strike + both premiums

Put: strike – both premiums

Call: strike + both premiums

Put: strike – both premiums

Module 37: Multiple Option Strategies

NASD Series 7 Page 292

4. The maximum loss for this position is;

A $900

B. $300

C. $3,100

D. Unlimited

5. Using the same facts, what are the breakeven points?

A. 21, 39

B. 35, 26

C. 39, 21

D. 26, 35

37.1.5.1 Straddle/Combination Review Answers

1. (A) A long straddle is made up of a long call and a long put with the

same strike price and same expiration month.

2. (D) For a long straddle to be profitable the investor would want the market

to go up for the call and go down for the put.

3. (D) Use the T-chart. As you can see the premiums

cost $900 (Out) which would be the maximum

loss while the maximum gain would be

unlimited as indicated by the blank side (In).

4. (A). As explained in Answer #3 above, the

most an investor can lose in a long position are

the premiums paid.

5. (C) The breakeven for the call is 30 (strike

price of the call) plus 9 (both premiums) for a total of 39. The breakeven

for the put is 30 (strike price of the put) minus 9 (both premiums) for a total

of 21.

You should now be ready to move on to another multiple option strategy

known as Spreads. Please turn to the next page!

 

call

Write 1 PTS Jan 50 call

II. Long 1 PTS Jan 50 call

Short 1 PTS Jan 60 call

III. Long 1 PTS Jan 50 call

Short 1 PTS May 50 call

IV. Long 1 PTS Jan 50 call

Short 1 PTS May 60 call

A. I only

B. II only

C. III and IV only

D. II and IV only

Use the following set of facts to answer the following 5 questions.

On the same day a customer buys 1 Dell Mar 70 call @ 5 and sells 1 Dell Mar

80 call @2 when the market price of Dell is 71.

2. The maximum potential loss is;

A. $200

B. $300

C. $500

D. $700

3. The maximum potential gain is;

A. $200

B. $300

C. $500

D. $700

4. The breakeven point is;

A. 70

B. 73

C. 75

D. 77

Module 37: Multiple Option Strategies

NASD Series 7 Page 302

5. The position will be profitable if;

I. Both contracts expire

II. Both contracts are exercised

III. The spread widens

IV. The spread narrows

A. I and III

B. I and IV

C. II and III

D. II and IV

6. A customer buys 1 IBM Sep 40 call and sells 1 IBM Feb 40 call. This is a;

A. Calendar debit spread

B. Calendar credit spread

C. Vertical debit spread

D. Vertical debit spread

7. A customer buys 1 PTS Mar 40 call and sells 1 PTS Aug 50 call. This is a;

A. Horizontal spread

B. Diagonal spread

C. Vertical spread

D. Horizontal credit spread

Use the following facts to answer the next 2 questions.

A customer on the same day buys 1 Stuff Feb 80 put @ 6 and sells 1 Stuff

Feb 90 put @10 when the market price of Stuff is 85.

8. The maximum potential gain is;

A. $400

B. $600

C. $1,000

D. $1,600

9. The maximum potential loss is;

A. $400

B. $600

C. $1,000

D. $1,600

Go to the next page for Review Answers

Module 37: Multiple Option Strategies

Page 303 NASD Series 7

37.5 MULTIPLE OPTIONS REVIEW ANSWERS

1. (C) Horizontal spreads have different expiration months. This would include

spreads III and IV only. Options I is not a spread as both the expiration months

and strike prices are the same. In order to be a spread the months and/or expiration

months must be different. Option II is a Vertical Spread as the strike prices are

different but the months the same. If both the months and strike prices were different

it would be a Diagonal Spread.

2. (B) Set up your T-chart.

Look at the T-chart to see which column is larger. In this case the “out” column is

larger, therefore reflecting the maximum loss. Just take the difference of the numbers

in both columns which gives you the maximum loss of $300.

3. (D) To find the other maximum, in this case the maximum gain, you must exercise

both option contracts. The key is to place the exercise dollars into the correct

column. Remember, use “Calls Match” and “Puts Cross” to determine which column

to place the exercise dollars into.

This is a call spread so you will use “Calls Match” which means place the exercise

dollars into the column that has the premium for that option contract.

4. (B) To calculate the breakeven point for a call spread use the acronym C.A.L.S.

In other words, if you have a call spread (C) add (A) to the lower strike (L.S.) price

the difference of the spread premiums. So take the lower strike price of 70 and

add (call up) the difference of the two premiums (5 – 2 = 3) 3.

5. (C). This is a debit spread (look at the diagram in answer #2) because the “out”

column is larger than the “in column” (You spent more than you brought in). Now

use the acronym WED (D for debit) and the answer would be choice C as it would

be more profitable for a client if the contracts are exercised (E) and the spread

widens (W).

Out In

Out In

500 200

Step 1: Exercise the Mar 70 call (70 x 100) = 7,000. As calls

match place the 7,000 into the Out column as this is the column

with this contract’s premium.

Step 2: Exercise the Mar 80 call (80 x 100) = 8,000. As calls

match place the 8,000 into the In column as this is the column

with this contract’s premium.

Step 3: Take the difference between the Out and In columns,

which is 700

500

7000

7500

200

8000

8200

Module 37: Multiple Option Strategies

NASD Series 7 Page 304

6. (B). Since the September call expires prior to the February call (first option

contract listed in the question is the first in time), the investor is buying the less

expensive contract and selling the more expensive contract. Therefore this would

be a credit spread (more in than out). As the months are different this would also

be a calendar spread. The strike prices are the same so this spread cannot be a

vertical spread.

7 (B) When both the strike prices and expiration months are different the spread is

called a Diagonal Spread.

8. (A). Set up your T-chart! As you can see below the column containing the larger

number would be the “in” column. Take the difference (1000 – 600 = 400) and this

would be the maximum potential gain for this option spread.

9. (B) To calculate the maximum loss, in this example, you would have to exercise

both contracts. Remember “Puts Cross” so take the exercise dollars and

place them in the column across from their contract premium.

Add the columns and take the difference for the answer.

Out In

600 1000

Out In

600

9000

1000

8000

9600 9000

The key to whether the premium should widen or narrow or if the contracts

should or should not be exercised is with the following two acronyms;

WED

NUC

The last letter of WED and NUC represents whether the spread is a Debit or

a Credit spread.

The first letter stands for widen (W) or narrow (N).

The middle letter stands for exercise (E) the options or let the contracts

remain unexercised (U).

If the spread is a Debit Spread then it would be in the client’s best interest for

the premiums to widen and the option contracts exercised.

If the spread is a Credit Spread then it would be in the client’s best interest

for the premiums to narrow and the option contracts to remain unexercised.

 

43.12 MARGIN REVIEW QUESTIONS

1. John D buys 500 shares of PTS Inc. at $50 in a margin account with Reg T

at 50%. After making the deposit, what is the investor’s debit balance?

A. $25,000

B. $12,500

C. $13,000

D. $50,000

2. Credit from bank to broker is controlled under;

A. Regulation A

B. Regulation T

C. Regulation U

D. Regulation Z

3. Generally, new issues cannot be margined for;

A. 10 days

B. 20 days

C. 30 days

D. 60 days

4. What are initial and maintenance margins for stock positions in a long

margin account?

A. 50 / 50

B. 50 / 35

C. 50 / 30

D. 50 / 25

5. An investor opens a new margin account and buys 400 shares of PTS @

25. What is the investor’s initial margin requirement?

A. $5,000

B. $7,500

C. $8,000

D. $10,000

6. An investor has an established margin account with a CMV of $5,000 and

a debit balance of $2,000. How much equity does the investor have in the

account?

A. $500

B. $2,500

C. $1,000

D. $3,000

Margin Accounts

NASD Series 7 Page 350

7. A customer sells short 2000 shares of PTS stock at $10, as the initial

transaction in a new margin account. The customer must deposit;

A. $5,000

B. $2,000

C. $10,000

D. $20,000

8. An investor buys 1,000 shares of PTS at $100 in a margin account. If PTS

drops to $66 what is the maintenance call?

A. $0

B. $500

C. $1,000

D. $2,000

9. A customer has a restricted margin account with a LMV of $33,500. If the

customer wishes to sell $15,000 of securities which of the following is true;

A. Do nothing special

B. Limit the sale to 50% of the LMV

C. Broker-dealer retains 7,500 of the proceeds

D. Pays a special fee of 25%

10. The approval to open a new margin account is given by the;

A. Customer

B. Registered representative

C. General securities principal

D. NASD

Margin Accounts

Page 351 NASD Series 7

43.13 MARGIN REVIEW ANSWERS

1. (B): The CMV is $25,000. Therefore the required Reg T deposit of 50% or

$12,500. The debit balance (loan) is also $12,500.

2. (C): Reg U regulates credit between banks and broker-dealers while Reg T regulates

credit between broker-dealers and customers.

3. (C): New issues cannot be margined for a period of 30 days.

4. (D): For long accounts the initial margin is Reg T of 50% while the NASD/NYSE

minimum maintenance rule requires maintaining 25% of the CMV as equity at all

times. For short accounts the numbers are 50 / 30 (not 25).

5. (A): The Reg T requirement is 50% of the purchase value of $10,000 or $5,000.

6. (D): With a CMV of 5,000 and a Debit of $2,000 the Equity would be $3,000.

7. (C): The short market value is $20,000. Therefore the investor must place into

the account the Reg T requirement of 50% or $10,000.

8. (B): The long market value initially was $100,000. Therefore, both the debit and

equity was $50,000 each. With the LMV dropping to $66,000, the debit stays the

same so the equity decreases to $16,000. The NASD/NYSE minimum maintenance

rule states that there must be a minimum of 25% of the LMV as equity.

Well, 25% of $66,000 is $16,500 and the equity is only $16,000. Therefore there

will be a margin (maintenance) call of $500.

9. (C): When an account is restricted the broker-dealer must retain 50% of the

proceeds to apply to the debit balance.

10. (C): Initially, the general securities principal of an NASD firm or branch manager

of NYSE firm must sign the margin account along with the registered representative.

 

44.7 REVIEW QUESTIONS

1. Which department in a brokerage firm would handle applications for

credit?

A. Margin

B. Purchases and sales

C. Sales order

D. Reorganization

2. According to current regulations, customer statements should be sent

how often to clients with inactive accounts?

A. Weekly

B. Monthly

C. Quarterly

D. Annually

3. Trade confirmations must be sent to the client;

A. Daily

B. By the trade date

C. By the settlement date

D. Quarterly

4. Who reports the trade execution to the customer?

A. Supervisor

B. Registered representative

C. Margin department

D. Sales department

5. Which department sends confirmations to customers?

A. Order department

B. Margin department

C. Purchases and sales department

D. Wire room

Brokerage Support Services

NASD Series 7 Page 362

44.8 REVIEW ANSWERS

1. (A): The margin department will review customer applications for credit when

buying or selling short securities.

2. (C): If there’s no activity then statements should be mailed at least quarterly.

With activity, statements are mailed monthly.

3. (C): Trade confirmations must be sent to the client by the end of the trade which

is the settlement date.

4. (B): The registered representative reports the trade execution to his/her client.

5. (C): That’s the job of the purchases and sales department.

 

47.5 REVIEW QUES T IONS

1. A customer asks you to sell 100 shares of PTS if the market falls to 40, but

she does not want to sell for less than 35. The proper order is;

A. Sell 100 PTS @ 40 Stop Limit

B. Sell 100 PTS @ 35 Stop Limit 40

C. Sell 100 PTS @ 40 Stop Limit 35

D. Sell 100 PTS @ 35 Stop Limit

2. The above order is known as a;

A. Limit order

B. Stop order

C. Split order

D. Stop limit order

3. In a falling market which of the following orders will be executed?

I. Open Buy Limits

II. Open Sell Limits

III. Open Buy Stops

IV. Open Sell Stops

A. I and IV

B. I and III

C. II and IV

D. II and III

4. Billy Bob has a gain on a long stock position which he wishes to protect.

The appropriate order is;

A. Market order

B. Buy stop order

C. Sell Limit order

D. Sell stop order

5. An order to sell at 38.65 Stop Limit is entered before the opening of the

market. The subsequent trades are 39, 38.75, 38.12, 38, 39, 39.12. Which of

the following is the order triggered and executed?

A. 38.75, 39.12

B. 38.12, 39

C. 38.75, 39

D. 39, 39.12

Module 47: Types of Orders

NASD Series 7 Page 382

47.6 REVIEW ANSWERS

1. (C): The customer wants to sell if the price falls to 40 or less but is not willing to

accept anything less than 35. So the proper order would be 40 Stop Limit 35.

2. (D): This is known as a Stop Limit Order.

3. (A): With a falling market Open Buy Limits and Open Sell Stops may be executed.

Open Buy Limits indicate that the order was already triggered but didn’t fill

because the market didn’t fall to the limit price. A falling market could in fact execute

this type of order. With Open Sell Stops the order hasn’t been triggered yet. A

falling market could result in a trigger of the stop order and then it turns into a market

order with immediate execution.

4. (D) Sell stop orders would be appropriate to protect a gain on a long stock position.

The stock price would have to fall to the stop price for a trigger and then the

order turns into a market order for immediate execution.

5. (B) 38.12, 39: First the market price has to fall at or below the stop of 38.65.

The trade tape indicates that 38.12 is the first trade which meets this criteria. The

Stop order is now triggered and becomes a limit order of 38.65. This means the

trade will not be executed unless the price rises to 38.65 or higher (better). The

trading tape indicates a price of 39 which qualifies for the execution.

 

You need to know what type of orders are adjusted when a stock is going

to pay a dividend. Remember BLiSS – Buy Limit and Sell Stop orders get

adjusted by the exchange specialist.

 

Stock symbols appear in the trading tapes. A round lot is 100 shares.

SLD = Trade is being reported out of sequence. Make sure you remember

that if you see the symbol SLD in a trade tape for a stop order question, do

NOT use that trade for purposes of a trigger or execution.

OPD = There was a delayed opening with this stock. This can occur when

good or bad news is released prior to the opening of the new trade day.

48.3.3.1 Round Lots

Round lots of 100s are indicated on a trade tape using the symbol “s”. If

the trading tape has a 3s a trade took place for 300 shares.

Round lots of 10s is represented by s/s

 

49.3.2 Yellow Sheets

The Yellow Sheets is a daily printing of wholesale quotes for OTC Corporate

Bonds.

49.3.3 Blue List

The Blue List is a daily printing of quotes for Municipal Bonds. They are primarily

municipal bonds that are available in the secondary market, but can

also include other types of secondary bonds.

 

49.7 REVIEW QUES T IONS

1. Which of the following is a “firm quote”?

A. 10 – 10.12 workout

B. 10 – 10.12 subject

C. 10 – 10.12 nominal

D. 10 – 10.12

2. Which of the following persons trades securities over-the-counter?

A. Market maker

B. Specialist

C. Registered Representative

D. Two Dollar Broker

3. Who needs to report trades in NASDAQ stocks?

A. The buy side of the trade

B. The sell side of the trade

C. The registered representative

D. The supervising Series 24

4. What is the inside quote as based on the following Level II service?

A. 18.00 – 18.32

B. 18.12 – 18.21

C. 17.87 – 18.32

D. 18.01 – 12.25

5. What is the name of the NASD automated order execution system?

A. SuperDot

B. Brokerage support system

C. SOES

D. Big Board System

Dell Computers Level II NASDAQ Service

Market Maker Bid Ask

Prudential 18.000 18.250

Wachovia 18.120 18.250

Sal SmithBarney 17.875 18.210

Merrill 18.010 18.320

Module 49: Over the Counter Market (OTC)

NASD Series 7 Page 396

49.8 REVIEW ANSWERS

1. (D): A firm quote is one without anything attached or qualified. All the others are

not firm.

2. (A) Market makers make markets and trade securities in the over-the-counter

market.

3. (B) The sell side of a trade must report the trade within 90 seconds.

4. (B) A firm quote is taken from all the market makers making a market which is

indicated on the Level II system. Take the best bid (price sell at) of $18.12 and the

best offer (price buying at) of $18.21 and voila, the inside quote.

 

529 savings plan:

• Municipal bonds are not suitable in this plan.

T-bills and conventional corporate bonds are not great choices.

Zero coupon bonds, STRIPS and corporate stocks are a really good choice.

 

to you for help. Which of the following would you recommend?

A. Roth IRA

B. Traditional IRA

C. Keogh (HR-10)

D. 401(k) plan

2. Which of the following investors are eligible to establish a traditional

IRA?

I. Nursing student who earned $1,300 in a part-time job

II. A day trader whose sole source of income is $115,000 per year in investment

income

III. Paco, who earned almost $4,000 selling home made vases and whose

spouse is covered by a company qualified plan

A. I and II

B. I and III

C. II and III

D. I, II, and III

3. If a mandatory distribution is not taken from an IRA what is the IRS

penalty?

A. 6%

B. 10%

C. 15%

D. 50%

4. IRA rollovers can take place how many times a year?

A. 1

B. 2

C. 6

D. Unlimited

5. To remain tax-free, how many years must an investor keep assets in a

Roth IRA?

A. 5

B. 10

C. Until age 70 ½

D. No minimum number of years

Module 52: Retirement Plans

NASD Series 7 Page 414

6. Who would not be eligible for a TSA account?

A. Student at a private college

B. Professor at the University of Illinois

C. Custodian at the Barrington High School

D. Clerk at the Carpentersville Elementary School

7. Qualified profit-sharing plans have all of the following features, EXCEPT;

A. Contributions are taxable when received

B. Contributions are deductible for the employees

C. Employer contributions are flexible

D. Investment earnings accumulate on a tax-deferred basis

8. Excessive contributions to an IRA could result in penalty of;

A. 5%

B. 6%

C. 10%

D. 50%

9. Which of the following plans benefit the older corporate employee?

A. IRA

B. Roth IRA

C. Defined Benefit plan

D. Defined Contribution plan

10. Doctor Joyce earned $110,000 last year. What is the maximum contribution

that can be made to a Keogh Plan?

A. $27,500

B. $30,000

C. $40,000

D. $25,000

Module 52: Retirement Plans

Page 415 NASD Series 7

52.14 REVIEW EXPLANATIONS

1. (C) Keogh plans are for self-employed persons like Doris L. She could have an

IRA but that would not maximize her potential contribution. 401(k) plans are for

corporate employees.

2. (B) Choice II, the day trader, would not be eligible to have an IRA as he does

not have any earned income. Investment income does not count. Both of the others

have some sort of earned income. The nursing student will not be able to contribute

the maximum in an IRA but could still contribute her earnings.

3. (D) Mandatory distributions must be taken by the IRA holder by April 1st following

the year he/she turned 70½.

4. (A) Rollovers can only take place once a year while transfers can be made for

an unlimited number of times.

5. (A) To remain tax-free, contributions in a Roth IRA must remain there for at

least five years.

6. (A) The student is not working so no contribution to a plan. All the others can

make contributions.

7. (B) Contributions into a profit sharing plan can be made by the employer, not

the employee. All the other choices are features of profit sharing plans.

8. (B) Excessive contributions into an IRA may result in a 6% penalty if not taken

out by tax filing day.

9. (C) Defined benefit plans benefit the older employee while defined contribution

plans benefit the younger employees.

10. (C) With $110,000 gross income you can contribute 100% of earnings or

$40,000, whichever is less under the TRA 2001 tax laws.

 

from a variable annuity are subject to which of the following

forms of taxation?

A. Not taxed because of the special investment

B. Short-term capital gains

C. Ordinary income

D. Long-term capital gains

2.Variable annuity salespersons must register with the;

I. NYSE

II. NASD

III. State insurance department

IV.State real estate department

A. I and II only

B. III and IV only

C. II and III only

D. II and IV

3. A variable annuity contract guarantees a;

I. Fixed administrative expense

II. Fixed mortality expense

III.Rate of return

A. I only

B. II only

C. III only

D. I and II only

4. A variable contract has an AIR of 7%. The current month’s actual performance

was 7.2%. The annuitant’s monthly check as compared to the prior

month’s check will;

A. Go down in value

B. Stay the same

C. Go up in value

D. Not enough information

5. Which of the following payout options provides the highest monthly payout?

A. Life annuity

B. Life annuity with a period certain

C. Joint survivorship

D. Life annuity with a 20 year period certain

Module 53: Annuities

NASD Series 7 Page 426

53.14 VARIABLE ANNUI T Y REVIEW ANSWERS

1. (C): Distributions from all variable annuities, whether qualified or non-qualified

are taxed as ordinary income. They do not have favorable capital gain treatment.

2. (C): As a variable annuity is both a securities and an insurance product, the

salesperson must have both a securities and insurance license.

3. (D): A variable annuity guarantees both expenses and mortality risks. This

means that the amount of these costs are limited and if they go higher they will be

paid by the insurance company, not the annuitant. A rate of return, however, is

never guaranteed in a variable annuity as the investor makes the investment

choices, not the insurance company.

4. (C). Remember, if the actual return is higher than the AIR the next check will go

up in value. If the actual return was less than AIR the next check would go down in

value. If the actual return was the same as the AIR the next check would stay the

same.

5. (A): The life annuity option provides the highest monthly income.

 

55.10.3 Disintermediation

Disintermediation is defined as the flow of money out of low yielding savings

accounts and investing in higher yielding investments.

Disintermediation occurs during periods of tight money.

 

55.11.2 Balance of Payments Review

When debits exceed credits, a deficit in the balance of payments occurs.

When credits exceed debits surplus in the balance of payments occurs.

Go to the next page!

Debit (Deficit) vs. Credit (Surplus)

Debit Items Credit Items

Imports Exports

U.S. spending abroad Foreign spending in the U.S.

U.S. investments abroad Foreign investments in the U.S.

U.S. bank loans abroad

U.S. foreign aid

Customer Accounts 55: Economics

Page 445 NASD Series 7

55.12 ECONOMICS REVI EW QUESTIONS

1. Which is considered to be a coincident economic indicator?

A. Corporate profits

B. Stock market prices

C. Orders for machinery

D. Index of industrial production

2. An “easing” of money and credit in the economy will probably be the

result of;

I. Increasing the discount rate

II. Increasing reserve requirements

III.Decreasing reserve requirements

IV.Decreasing the discount rate

A. I and II only

B. II and III only

C. III and IV only

D. I and IV only

3. All of the following tools are used by the Federal Reserve to control the

money supply EXCEPT;

A. Setting reserve requirements

B. Open market operations

C. Setting the discount rate

D. Setting the prime rate

4. Cyclical changes in the business cycle would most likely have the greatest

affect on a;

A. Utility company

B. Machine tool company

C. Tobacco company

D. Food company

5. A six-month mild decline in economic activity is a(an);

A. Expansion

B. Upsurge

C. Recession

D. Depression

Customer Accounts 55: Economics

NASD Series 7 Page 446

55.13 ECONOMIC REVI EW ANSWERS

1. (D): Corporate profits show what happened in the preceding quarter so they are

a lagging indicator. Stock market prices are a leading indicator. Orders for machinery

show production to come and are a leading indicator. The index of industrial

production shows current production levels and is a coincident indicator.

2. (C): An easing of money and credit in the economy (more money would be

available) would be the result of a decrease in the discount rate and a discount in

reserve requirements. Both would cause an increase in the free reserves of banks

that could be loaned out.

3. (D): Monetary policy tools of the Fed include setting reserve requirements, the

discount rate and margin rates. The prime rate is set by Fed member banks which

is offered to their best customers.

4. (B): The stock of a machine tool company is cyclical and fluctuates with the

business cycle. The other choices are necessities or staples and are considered

defensive due to their resistance to recession.

5. (C): A recession is a decline in the GDP for 2 consecutive quarters or more.

 

58.1.1 Liquidity Formulas

(+) add (-) subtract (/) divide (=) equals

Net Working Capital = Total current assets – Total current liabilities

Current Ratio = Total current assets / Total current liabilities

Quick Asset Ratio = Total current assets – Inventory / Total current liabilities

Cash Flow = Net Income (or loss) + Annual Depreciation

58.1.2 Capitalization Ratios

Bond Ratio = Par value of bonds / Total long-term capital

Preferred Stock Ratio = Par value of preferred stock / Total long-term capital

Common Stock Ratio = Common stock at par + Capital surplus + Retained

earnings / Total long-term capital

Debt to Equity Ratio = Bonds + Preferred stock / Common stock at par + Capital

surplus + Retained earnings

58.1.3 Coverage Ratios

Bond Interest Coverage = Earnings before interest & taxes / Annual bond

interest expense

Preferred Dividend Coverage = Net income / Preferred dividends

58.1.4 Use of Assets

Inventory Turnover Rate = Cost of goods sold / Inventory

Book Value per Common Share = Total assets – Intangibles – Preferred stock

/ Number of outstanding shares

Customer Accounts 58: Financial Formulas Recap

NASD Series 7 Page 466

58.1.5 Profitability

Operating Profit Margin = Operating income / Net sales

Net Profit Margin = Net income / Net sales

Return on Common Equity = Net income – Preferred dividends / Common

stock at par + Capital surplus + Retained earnings

58.1.6 Evaluation of Earnings

Earnings per share = Net income – Preferred dividends / Number of outstanding

shares

Price/Earnings Ratio = Market price / Earnings per share

Dividend Payout Ratio = Annual dividend paid on common stock / Earnings

per share

Current Yield = Annual dividend per common share / Current market price

Customer Accounts 59: Fundamental Analysis Review Questions

Page 467 NASD Series 7

Section 59: Fundamental Analysis Review

Questions

1. When a corporation declares a dividend:

I. Current assets decrease

II. Current liabilities decrease

III. Stock holders equity decreases

IV. Current liabilities increase

A. I and II only

B. II and III only

C. III and IV only

D. II, III and IV only

2. Which of the following ratios would be used by an analyst examining the

capital structure of a wine producing corporation?

A. The acid-test ratio

B. The current ratio

C. The debt-to-equity ratio

D. The price/earnings ratio

3. A corporation issues new common stock. Which of the following are

affected?

I. Current Assets

II. Total liabilities

III. Net worth

IV. Retained earnings

A. I and II only

B. III and IV only

C. I, III, IV only

D. I and III only

4. A corporation buys some office furniture and fixtures, paying cash. Which

of the following are affected?

I. Current Assets

II. Total liabilities

III. Net worth

IV. Working capital

A. I and III only

B. II and III only

C. I and IV only

D. I, II, III and IV

Customer Accounts 59: Fundamental Analysis Review Questions

NASD Series 7 Page 468

5. A customer reading the newspaper notes that a corporation has a PE

ratio of 9 and a market price of $27. The earnings per share of the company

is approximately.

A. $1.50

B. $3.00

C. $18.00

D. $36.00

6. An investor may find all of the following financial information in a corporation’s

balance sheet EXCEPT the:

A. Expense ratios of the corporation

B. Assets of the corporation

C. Book value of the corporation

D. Long-term funds received from all sources

7. If total assets remain the same, but stockholder’s equity decreases, this

means that;

A. Retained earnings increased

B. Total liabilities increased

C. Capital in excess of par increased

D. Accrued expenses decreased

8. All of the following are examined by a fundamental analyst EXCEPT;

A. Balance sheets

B. EPS

C. Industry

D. Timing

9. The interest rate that banks charge each other for overnight loans is the;

A. Discount rate

B. Fed Funds rate

C. Prime rate

D. Call loan rate

10. The interest rate charged in margin accounts is the;

A. Discount rate

B. Fed Funds rate

C. Prime rate

D. Call loan rate

Customer Accounts 59: Fundamental Analysis Review Questions

Page 469 NASD Series 7

59.1 REVIEW ANSWERS

1. (C): When a corporation declares a dividend, stockholder’s equity (retained

earnings) is decreased and current liabilities are increased. When the corporation

pays the dividend, current assets are decreased and current liabilities are

decreased.

2. (C): The capital structure of a corporation is the dollar amount of the corporation’s

capitalization (equity and debt securities). An analyst would therefore be

interested in the debt to equity ratio. This is actually the ratio of those securities

creating fixed charges (bonds plus preferred stock) to common stock.

3. (A): If new common stock is issued, cash increases (from the proceeds of the

sale) as well as common stock at par and capital in excess of par. There is no

effect on retained earnings – which change due to profits, losses, and dividend

payouts.

4. (C): If furniture is bought with cash, then cash goes down (current asset) and

property, plant and equipment increases (long-term asset). If current assets drop,

then working capital drops. There is no effect on current liabilities because the furniture

is fully paid. Net worth is only affected by a profit, loss, dividend payout, or

capital structure change.

5. (B): The price/earnings ratio (PE) is found by dividing the earnings per share

into the market price of the stock. When given the market price and the P/E ratio,

the approximate earnings per share may be found by dividing the market price by

the P/E ratio ($27 market price divided by 9 = $3.00)

6. (A): Expense ratios cannot be determined from the balance sheet. All expenses

are found in the income (profit and loss) statement. All of the other items mentioned

can be determined from the balance sheet.

7. (B): The basic balance sheet equation is: total assets = total liabilities + stockholder’s

equity. If total assets remain the same and stockholder’s equity

decreases, total liabilities must have increased.

8. (D): A fundamental analyst determines what to buy by examining the income

statement, balance sheet, industry, and management of the company. A technical

analyst determines when to buy (timing).

9. (B): The Fed Funds rate is the interest rate that commercial banks charge each

other for overnight loans and is the most volatile of all interest rates.

10. (D): The call loan rate is the interest rate charged in margin accounts by brokerage

firms and is the highest of all loan rates.

 

Section 62: Technical Analysis Review

Questions

1. When analyzing a corporation’s stock, a technical analyst would be

LEAST concerned with its;

A. Chart pattern

B. Short interest

C. Trading volume

D. Debt-to-equity ratio

2. A technical analyst who monitors stock advances against declines subscribes

to the;

A. Dow Theory

B. Efficient Market Theory

C. Odd Lot Theory

D. Breath of Market Theory

3. A head and shoulder “top” formation is:

I. Bullish

II. Bearish

III. Reverse Upward Trend

IV. Reverse Downward Trend

A. I and III

B. II and III

C. I and IV

D. II and IV

4. An extended period where there is no discernible trend in prices is

termed;

A. Consolidation

B. Resistance

C. Support

D. Overbought

5. The market price average is increasing daily. However, the level of

advances relative to declines is falling. The market is reaching a(an);

A. Breakout on the upside

B. Overbought condition.

C. Oversold condition.

D. Breakout on the downside.

Customer Accounts 62: Technical Analysis Review Questions

NASD Series 7 Page 482

62.1 TECHNICAL ANALYS I S REVIEW ANSWERS

1. (B): A technical analyst would not be concerned with a corporation’s debt-toequity

ratio. A fundamental analyst would examine the debt-to-equity ratio to analyze

a company’s capitalization.

2. (D): An analyst who charts advances relative to declines is measuring the

“breadth” of the market movement as an indicator of future market direction.

3. (B): A head and shoulder top formation is bearish since the market has topped

out and is trending down. It is an uptrend that has reversed itself.

4. (A): The market is said to be “consolidating” when prices are not moving in any

direction for a long period after a price rise or fall.

5. (B): The market averages are rising, but the strength of the market is weakening

because advancing issues are declining relative to falling issues. The market

is reaching an “overbought” condition, and is approaching a peak.

 

Selection risk

B. Credit risk

C. Systematic risk

D. Non-systematic risk

2. The measure of “stock specific” risk is;

A. Delta

B. Alpha

C. Beta

D. Pita

3. A portfolio with a beta of +1 has;

A. No risk

B. Unsystematic risk

C. Systematic risk

D. Both systematic and unsystematic risk

4. Passive asset management is

A. Using index funds as the investments for each asset class.

B. Buying securities positions and holding them to the liquidation date of the portfolio.

C. Selecting securities to be purchased for each asset class based upon fundamental

analysis.

D. Buying securities positions and holding them until pre-established prices are

reached.

5. The use of multiple asset classes when constructing a portfolio reduces;

A. Purchasing power risk

B. Regulatory risk

C. Market risk

D. Interest rate risk

Customer Accounts 63: Portfolio Analysis

Page 493 NASD Series 7

63.9 PORTFOLIO ANALYS I S REVIEW ANSWERS

1. (C): Market risk is the same as systematic risk. This risk cannot be diversified

away.

2. (B): Alpha measures the risk peculiar to an individual security. Beta measures a

security’s volatility relative to the market.

3. (C): A portfolio with a beta of +1 is one that moves in the same direction and at

the same rate as the market. Thus, this portfolio only has market risk which is also

known as systematic risk. This is the risk that cannot be diversified away.

4. (A): Passive asset management does not mean that there is no management.

Passive asset management is the use of index funds (which are managed to mirror

a chosen index benchmark) as the security selections within an asset class.

Thus, the actual specific security selection and management is embedded within

the index fund chosen for investment.

5. (C): The use of multiple asset classes when constructing a portfolio reduces

market risk.

 

A prospectus is NOT considered advertising under the NASD advertising

rules because the prospectus is regulated by the Securities Act of 1933.

Therefore, they are not subject to the NASD advertising rules. Remember

this point as it does appear on the Series 7 exam.

 

65.12 REVIEW QUESTIONS

1. Which of the following securities transactions are subject to the antifraud

provisions of the Securities and Exchange Act of 1934?

I. Corporate Stock sales

II. U.S. Treasury Notes

III. Municipal Bonds

A. I only

B. I and II

C. II and III

D. I, II and III

2. Freeriding and withholding applies to which of the following?

A. New issues that sell at a premium

B. New issues that sell at a discount

C. All securities no matter how they sell

D. None of the above

3. The time period during which the SEC reviews new registration packages

is known as the;

A. Due Diligence Meeting

B. 20-day cooling off period

C. 90 day cooling off period

D. Blue-sky timer period

4. To which security market does the Securities Act of 1933 apply?

A. Primary

B. Secondary

C. Third

D. Fourth

5. Maximum protection for SIPC accounts is;

A. $500,000 in securities plus $100,000 in cash

B. $500,000 with any combination of securities and cash

C. $500,000 total which includes a maximum of $100,000 in cash

D. $500,000 in securities and no cash protection

Module 65: Federal Securities Acts

NASD Series 7 Page 508

65.13 REVIEW ANSWERS

1. (D) All securities and all transactions are subject to the anti-fraud provision of

the Securities Act of 1934.

2. (A) New issues selling at a premium are known as hot issues. Freeriding and

withholding applies to hot issues.

3. (B) All new registration packages are reviewed by the SEC for a minimum of 20

days. This is known as the 20-day cooling off period.

4. (A) The Securities Act of 1933 applies to the registration of new nonexempt

issues.

5. (C) SIPC provides protection up to $500,000 which includes a maximum of

$100,000 in cash.

 

66.7 REVI EW QUESTIONS

1. The NASD is known as a(an);

A. Federal agency

B. State agency

C. SRO

D. A legal arm of the SEC

2. Annual NASD fees would include;

I. Branch office charges

II. Basic membership fee

III. Assessments for NYSE stock transactions based on gross income

IV. Fees for each registered representative

A. I and II

B. II and III

C. III and IV

D. I, II and IV

3. Which of the following is also known as the Conduct Rules?

A. Rules of Fair Practice

B. Code of Procedure

C. Uniform Practice Code

D. Code of Arbitration

4. When filing for a position with a broker-dealer you must file which of the

following forms?

A. U-5

B. U-4

C. U-3

D. U-2

5. A registered representative’s termination is normally effective after;

A. 15 days

B. 30 days

C. 45 days

D. 60 days

Module 66: Registration and Regulation of Broker-Dealers

NASD Series 7 Page 518

66.8 REVIEW ANSWERS

1. (C) The NASD is a Self Regulatory Organization (SRO) for the securities

industry.

2. (D) All are correct except assessments for NYSE stock transactions. The NASD

does not supervise the NYSE as the New York Stock Exchange is regulated by

their own rules. Therefore, the NASD is not allowed to assess trades which take

place on the NYSE.

3. (A) The Rules of Fair Practice are also known as the Conduct Rules which sets

out fair and ethical trade practices that member firms and their registered representatives

must follow when dealing with the general public.

4. (B) The U-4 is the initial regulatory form that must be filed with the NASD. Upon

leaving a brokerage firm a U-5 must be filed.

5. (B) All terminations are effective 30 days after the date of notice.

67.4 REVIEW QUESTIONS

1. The SEC has been given authority to make advertising rules under which

of the following Acts?

I. Securities Act of 1933

II. Securities Exchange Act of 1934

III. Telephone Solicitation Act of 1998

IV. Brokerage Firm Act of 1939

A. I only

B. I and II

C. III and IV

D. I, II, III and IV

2. Which of the following is NOT an example of Advertising?

A. Motion pictures

B. Television

C. Form letters

D. Seminars

3. New brokerage firms must file advertisements with the NASD;

A. Within 10 days after first use

B. After 10 days from the first use

C. Prior to use

D. After use

4. Mutual fund advertising by an existing brokerage firm must be filed

within;

A. Ten days after first use.

B. Within 10 days prior to first use

C. Prior to use

D. Never because the brokerage firm is experienced

5. All of the following are true regarding advertising issues EXCEPT?

A. Recommendations must be suitable

B. Generic ads must be accompanied by a prospectus

C. NASD name must be smaller than the firm’s name when printed on business

cards

D. Advertising must disclose maximum amount of a loaded fee fund

Module 67: Advertising and Sales Rules

NASD Series 7 Page 524

67.5 REVIEW ANSWERS

1. (B) By definition

2. (C) Form letters are an example of sales literature, not advertising.

3. (C) For new firms in business for under one year all advertising must be submitted

for pre-approval by the NASD prior to first use.

4. (A) Mutual fund, variable contracts and unit investment trust advertising for a

seasoned brokerage firm needs to be filed within 10 days after first use.

5. (B) Generic advertising need NOT be accompanied by a prospectus because it

is generic and not a specific product.

 

68.7 REVIEW QUESTIONS

1. Churning refers to which of the following?

A. Freeriding in at least five customer accounts

B. Excessive trading in a customer’s account for purposes of generating commissions

C. Mixing one pound of butter and one quart of milk

D. Manipulation

2. An OSJ does all of the following EXCEPT:

A. Execute customer orders

B. Review all trades prior to the order being placed

C. Approve new accounts

D. Approve advertising

3. Which of the following is NOT required as a suitability inquiry?

A. Investment objectives

B. Tax status

C. Date of Birth

D. Current investments

4. Which of the following is NOT a violation?

A. Misrepresentation

B. Breakpoint sales (Recommending a purchase just shy of reaching a breakpoint)

C. Short-term buying and selling of mutual funds (Trading mutual funds)

D. Not selling dividends

5. Registered principals must review all trade tickets;

A. As the trade is made

B. Each morning

C. By the end of the day

D. Once a week

Module 68: Conduct Rules

NASD Series 7 Page 530

68.8 REVIEW ANSWERS

1. (B) By definition

2. (B) Trades must be reviewed by a principal by the end of the day, not prior to

each trade. All other choices are correct.

3. (C) The date of birth is never required. Inquiry into age of majority is required.

Other choices should be part of all suitability inquiries.

4. (D) Selling dividends is a violation, so not selling dividends is not a violation. All

the other choices are violations.

5. (C) The requirement is for registered principals to review all trade tickets by the

end of the day.

 

70.8 REVIEW QUESTIONS

1. Who can make a complaint against a registered representative?

A. A client

B. The NASD

C. Brokerage firm

D. Any of the above

2. Binding arbitration is required against all of the following EXCEPT:

A. Registered rep against a brokerage firm

B. Brokerage firm against another brokerage firm

C. Client against a registered representative

D. Brokerage firm against a client

3. Arbitration decisions are;

A. Subject to court review within 90 days

B. Final

C. Subject to review by the SEC

D. Subject to review by the county courts

4. Summary complaint procedure can result in which of the following?

A. Maximum fine of $25,000

B. Maximum fine of $2,500

C. Censure

D. B & C

5. Initial complaints are filed with the;

A. National Adjudicatory Council

B. District

C. SEC

D. Federal Courts

Module 70: Uniform Code of Arbitration

NASD Series 7 Page 536

70.9 REVIEW ANSWERS

1. (D) Any of them

2. (D) Anyone can bring anyone to arbitration with one exception. A customer cannot

be brought to arbitration unless they agreed in a prior written agreement.

3. (B) They are final.

4. (D) Maximum punishment is a $2,500 fine and or censure.

5. (B) Initial complaints are filed with the District which was formerly the DBCC.

 

71.11 REVI EW QUESTIONS

1. Under NYSE rules, before a registered representative may take a second

job (moonlight), he/she must obtain written permission from the;

A. SEC

B. NASD

C. NYSE

D. Reps broker-dealer

2. Under NYSE rules, for firms that do not have a communications compliance

program, a registered representative must get approval of the Branch

Manger to;

I. Send options advertising to customers

II. Send a prospecting letter to customers

III. Mail a prospectus to a customer

IV. Conduct a seminar on options strategies

A. I and II only

B. I, II and III only

C. I, II and IV only

D. I, II, III and IV

3. A registered representative has written a research report about bank

stocks that she wishes to mail to customers. Which statement is true?

A. The report must be approved by a supervisory analyst dealing with bank stocks

B. The report must be approved by a Branch Manager

C. The report must be approved by the NASD

D. No prior approval is required

4. If there is activity in a customer’s account, a statement must be mailed;

A. That month

B. That quarter

C. Twice a year

D. Annually

5. Copies of order tickets must be kept for;

A. 1 year

B. 2 years

C. 3 years

D. 5 years

Module 71: NYSE Rules

Page 543 NASD Series 7

71.12 REVIEW ANSWERS

1. (D): If a rep chooses to take a second job, prior written approval must be

obtained from the employing broker-dealer.

2. (C): Under the NYSE rules, a firm that does not have a communications compliance

program requires manager approval for a prospecting letter, advertising or

even giving a speech. All of these fall under the definition of communications with

the public. There is no requirement for prospectus approval as a prospectus is a

requirement under the Securities Act of 1933.

3. (A): Believe it or not, under NYSE rules research reports must be either written

or approved by a Supervisory analyst as this person had to pass a qualification

exam specifically for that purpose. There is no requirement for NYSE approval,

NASD approval or Branch Manager approval.

4. (A): Generally, the minimum requirement for mailing statements is quarterly.

However, if there is activity in the customer’s account a statement must be mailed

for that month.

5. (C): This is a record retention rule. Most records must be kept for 3 years. This

is one of those records that would be kept for the 3 year period. Complaint letters,

blotters and registers must be kept for six years while copies of corporate minutes,

as an example, must be kept for life.

Which of the following are TRUE of defined contribution plans? I. Annual contribution amounts vary. II. Annual contribution amounts remain fixed. III. Employer contributions may be discontinued if the corporation has a bad year. IV. The retirement benefits will increase the longer an employee is with the corporation.
The word defined indicates a specified amount. In a defined contribution plan, the annual contribution amounts remain fixed, even if the corporation has a bad year; therefore, II is true and I and III are false. B is the only answer choice that works. Obviously, the longer the employee stays with the company, the more money goes into the retirement plan and the more money the investor will receive at retirement.
Sorry, that is not the correct answer. The correct answer is: B. II and IV only

A local municipality decides to issue bonds to build a public park. Which of the following characteristics of the issuer should be considered when analyzing this municipal GO bond issue? I. Rate covenants II. Property taxes III. Flow of funds IV. Budgetary practices
Sorry, that is not the correct answer. The correct answer is: B. II and IV only
If you remember that covenants (promises stated on the bond indenture) are for revenue bonds, not GO (general obligation) bonds, this question is really easy. Notice that rate covenants (Roman numeral I) appears in answers A, C, and D. Therefore, the only answer that works is B.

The margin department handles all credit that a broker-dealer extends to a customer. And the other departments? Well, the wire room (order department) is the department within a broker-dealer that transmits orders to the proper markets for execution and sends a confirmation to the registered rep. The purchase and sales department sends confirmations to customers and fixes any trading errors. And the cashier department accepts payments and issues checks to customers.

A tight money yield curve indicates which TWO of the following? I. Short-term bonds are at lower yields than long-term bonds II. Short-term bonds are at higher yields than long-term bonds III. Interest rates have recently increased IV. Interest rates have recently decreased
A tight money (inverted) yield curve suggests that bond yields are opposite of what you’d expect. Normally, short-term bonds have lower yields than long-term bonds. A tight money yield curve indicates that interest rates have recently increased and short-term bonds are therefore yielding more than long-term bonds.
An official notice of sale is published to indicate that a municipal offering is going to be made
A municipality publishes a notice of sale in The Bond Buyer if it’s taking bids on the issue. Therefore, a notice of sale is published for bonds that are sold on a competitive basis.

Albany, NY, is issuing $20,000,000 worth of callable bonds in future months. Which of the following is NOT TRUE about the call feature of these municipal bonds?
Sorry, that is not the correct answer. The correct answer is: A. The issuer added a call feature to make the bonds more marketable.
Answers A and B directly oppose each other, so the chances are extremely high that one of those is the correct answer. Because you’re looking for the answer that’s not true (or as people in some circles say, false), answer A has to the one you want. The call feature of a bond doesn’t make the bond more marketable; it makes it less marketable, because investors have no control over how long they can hold onto the bond. Therefore, the issuers have to pay a higher coupon rate to attract investors.

The NASDAQ short sale rule applies to which of the following?
Sorry, that is not the correct answer. The correct answer is: A. NNM securities
The short-sale rule applies to NNM (NASDAQ National Market) securities. This short sale rule prohibits entering a short sale (selling borrowed securities) at or below the current inside bid price whenever the current bid price is below the previous inside bid price. The idea behind this rule is to keep short sellers from continually driving down the price of a security.

A workable indication is a(n)
Sorry, that is not the correct answer. The correct answer is: C. likely bid price
A workable indication is the price at which a municipal securities dealer is willing to purchase securities from another municipal securities dealer. Therefore, a workable indication is a likely bid price.

An investor might be interested in purchasing a variable rate municipal bond over a bond with a fixed coupon rate because
There’s an inverse relationship between bond prices and interest rates. If interest rates increase, outstanding bond prices fall; if interest rates decrease, outstanding bond prices increase. However, if the bond is variable rate, the coupon rate adjusts to interest rate changes, so the market price of the bond will remain relatively stable. The word always is an absolute, which should have given you a hint that choice C is wrong.

An investor has held shares of DEF restricted stock for over one year. DEF has 3,000,000 shares outstanding. The most recently reported weekly trading volumes for DEF are as follows: 1 week ago: 25,000 2 weeks ago: 40,000 3 weeks ago: 30,000 4 weeks ago: 35,000 5 weeks ago: 40,000What is the maximum number of shares that the investor can sell under Rule 144?
Sorry, that is not the correct answer. The correct answer is: B. 32,500
The resistance level is the upper portion of the trading range, so the answer is C. The support, by the way, is the lower portion of the trading range and in this case would be 50.
Mikayla Moneysack would like to set up an UGMA account for her 12-year-old son. Mikayla would also like to be the custodian for the account. Which of the following governs the investments purchased for the account? I. The legal list II. The prudent man rule III. The NASD list of approved investments IV. The NYSE list of approved investments
Fiduciary accounts such as UGMA (Uniform Gift to Minors Act) accounts are required to follow the prudent man rule or legal list of the state in which the account is set up. The prudent man rule or legal list establishes a guideline of appropriate investments for accounts that involve trusts.

Which of the following BEST describes a third-market trade?
A first-market trade involves listed securities trading on an exchange. A second-market trade is OTC (over-the-counter) securities trading OTC. A third-market trade is listed securities trading OTC. A fourth-market trade is institutional trading without using the services of a broker-dealer.

Advertisements pertaining to previous stock recommendations MUST include which of the following?
You are correct. The answer is: C. All stock recommendations made by your firm in the last year
Customers have the right to see all recommendations of a similar kind for the last year. If a firm is claiming its expertise in making recommendations, customers are entitled to see how successful the firm’s recommendations really are — for instance, did the firm pick correctly 70 or 80 percent of the time or only 30 percent of the time?

Which of the following is TRUE regarding a Regulation D offering?
You are correct. The answer is: C. It is an offering of securities to no more than 35 unaccredited investors in a one-year period.

A Regulation D offering is also called a private placement. Regulation D offerings are exempt from SEC registration as long as the securities are sold to no more than 35 unaccredited (small) investors per year.

When analyzing the credit of a revenue bond issue, which of the following are important? I. Debt per capita II. Management III. Debt service coverage ratio IV. Debt to assessed valuation
If you remember that debt per capita (debt per person) is important to GO (general obligation) bonds and not revenue bonds, you can eliminate all answers except for B. Management is always important, as is the debt service coverage ratio. The debt service coverage ratio is how much money the municipality has available from the revenue-producing facility to be able to pay off the principal and interest on the bonds. The debt to assessed valuation is the amount of money that the municipality owes, as compared to the assessed values (estimated market values) of the properties in the municipality, which is important to the tax-backed GO bonds, not revenue bonds.

Short-term municipal notes include RANs (revenue anticipation notes), BANs (bond anticipation notes), TANs (tax anticipation notes), PNs (project notes), and CLNs (construction loan notes). AONs (all or none) may look like it belongs more than PNs (because PN has only two letters), but AONs are an order qualifier, not a municipal note.

All of the following are TRUE about GNMAs EXCEPT
A. They pay interest semiannually.
B. B. They are backed by the U.S. government.
C. C. The interest received by an investor is fully taxed.
D. D. They are considered safer than FHLMCs.
GNMAs (Ginnie Maes) pay interest monthly, not semiannually.

Corporations may issue which of the following debt securities? I. Income bonds II. Mortgage bonds III. Double barreled bonds IV. Moral obligation bonds
Corporations may issue income bonds and mortgage bonds. Income (adjustment) bonds are issued by corporations that are in the process of reorganization (usually to avoid bankruptcy); income bonds don’t receive interest or principal until the corporation can afford it. Mortgage bonds are issued by corporations and are secured by a pledge of property. Choices III and IV are municipal bonds. Double barreled bonds are a combination of revenue and general obligation bonds. If revenues aren’t enough, the bonds are backed by the full faith and credit (taxing power) of the municipality. Moral obligation bonds are local municipal bonds that are backed by the state in the event of default.

Companion (support) tranches absorb the prepayment risk associated with CMOs (collateralized mortgage obligations). All PAC (planned amortization class) and TAC (targeted amortization class) tranches are backed by companion tranches. PAC (planned amortization class) tranches have the most certain prepayment dates and are considered the safest of the CMOs. TAC (targeted amortization class) tranches are subject to more extension and prepayment risk than PAC tranches, so they receive a higher interest rate. As for the others, PO (principal only) tranches receive income from principal payments on the underlying mortgages. Z tranches do not receive payments until all other tranches are retired and are considered the most volatile of the CMO tranches.

This investor has a total of 2,000 votes (500 shares × 4 vacancies). Because this company uses cumulative voting, the investor can vote the shares in any way that he sees fit, even if he wants to vote them all for one candidate.

A Regulation D private placement offering is
You are correct. The answer is: C. an offering of securities to no more than 35 unaccredited investors in a 12-month period
Regulation D is a provision in the Securities Act of 1933 that exempts offerings sold to no more than 35 unaccredited (small) investors per year. Regulation D offerings can raise an unlimited amount of money but are limited to the number of unaccredited investors per year.

HLP Corp. is offering 5,000,000 new shares to the public. The shares are being sold to a syndicate for $11 and are being reoffered to the public at $12. The takedown for each share sold is $0.75. The concession is $0.30 a share, and the managing underwriter retains $0.25 in fees for each share sold by anybody. The selling group will assist in selling 1,000,000 of the 5,000,000 shares offered. If the selling group sells its entire allotment, how much does the syndicate make on shares sold by the selling group?
Sorry, that is not the correct answer. The correct answer is: C. $450,000
HLP Corp. is offering 5,000,000 new shares to the public. The shares are being sold to a syndicate for $11 and are being reoffered to the public at $12. The takedown for each share sold is $0.75. The concession is $0.30 a share, and the managing underwriter retains $0.25 in fees for each share sold by anybody. The selling group will assist in selling 1,000,000 of the 5,000,000 shares offered. If the selling group sells its entire allotment, how much does it make in profits?
You are correct. The answer is: C. $300,000
This problem provides loads of extraneous information. The selling group receives the concession. If the selling group sells all of its 1,000,000 shares, the total will be $300,000 ($0.30 × 1,000,000 shares).

The syndicate members receive the takedown of $0.75 per share for every share they sell, and the selling group receives the concession of $0.30 per share for each share they sell. Therefore, the profit the syndicate members make on shares sold by the selling group is $0.45 per share ($0.75 – $0.30) for a total of $450,000 ($0.45 × 1,000,000 shares).

Takedown = concession + additional take

The ex-dividend date (the first day a purchaser doesn’t receive a previously announced dividend) is two business days before the record date. If the record date is Tuesday, September 17, the ex-dividend date is Friday, September 13, because you don’t count weekends as business days.

A Rule 147 offering is an intrastate offering that is exempt from SEC registration, provided the issuer conducts business only in one state and sells securities only to residents of the same state. This provision also includes the 80 percent rule, which states that at least 80 percent of the issuer’s assets should be located within the state and at least 80 percent of the offering proceeds should be used within the state.

Choices C and D are opposites, so one of them is likely correct. Short-selling is selling a security that you don’t own (a borrowed security); you’re hoping that the price of the security will drop so you can purchase the securities in that market at a lower price and return the securities. Short-selling is a bearish strategy, because investors want the price of the security to decrease. All gains and losses from selling short are considered short-term, and the investor is taxed at his or her tax bracket.

All of the following are TRUE of IDR bonds EXCEPT I. The money raised is used to construct a facility for a private corporation. II. The bonds are issued and backed by a municipality. III. They are issued to construct a non-revenue-producing facility. IV. They are also known as a special tax bond.
Because this question is an except question, you’re looking for the false answers. Municipalities issue IDRs (industrial development revenue bonds) to fund the construction of a facility for the benefit of a private user (a corporation). Even though the IDR is a municipal bond, it’s backed by lease payments made by the corporation, and it’s therefore considered the riskiest municipal bond. Because you know that Roman numeral I is correct and you’re looking for incorrect choices, the only answer that works is C.

Place the following compensations paid out from a new issue in order from largest to smallest: I. Concession II. Reallowance III. Takedown IV. Manager’s fee
Sorry, that is not the correct answer. The correct answer is: C. III, I, II, IV
When a syndicate sells a new issue, the spread is broken down into the takedown and the manager’s fee. The takedown is the amount that syndicate members earn when selling shares or bonds. If the syndicate hires selling groups to help sell the shares, these groups receive the concession, which is paid out of the takedown. Firms outside of the syndicate or selling groups receive the reallowance when selling new shares, which is still paid out of the takedown but is not quite as high as the concession. The manager’s fee is the lowest, but the syndicate manager receives a fee on every share (or bond) sold by anyone.

Which of the following are possible responsibilities of a municipal bond counsel? I. Writing a qualified legal opinion II. Writing an unqualified legal opinion III. Making sure that the issue is legally binding on the issuer IV. Making sure that the issue meets all conditions to offer tax-free interest
You are correct. The answer is: D. I, II, III, and IV
What’s nice about this question is that Roman numerals I and II are in every answer, so you know they have to be correct (note that these answer choices aren’t mutually exclusive opposites). Municipalities hire a municipal bond counsel to write a legal opinion unless the issue is delivered ex-legal (without a legal opinion). The municipal bond counsel has to determine that the issue is legally binding on the issuer (III) and that it meets the tax laws so it can deliver federally tax-free interest (IV). Additionally, the municipal bond counsel has to determine whether the issue is qualified (there may be a legal uncertainty about the issue) or unqualified (without reservations) — I and II.

All of the following statements could legally appear in advertising or sales literature EXCEPT I. “The NAV of our aggressive growth fund will outperform the market.” II. “Our sales team is dedicated to working with you to help meet your investment goals.” III. “Our sales team is the best in the industry.” IV. “We guarantee that we will earn each customer at least 8 percent on investments each year.”
If you notice, Roman numeral III is in every answer choice, so you don’t even have to read that point; you know that a company can’t state that it’s the best. Roman numerals I and III aren’t legal because they’re guarantees, and a firm can’t guarantee that its aggressive growth fund will outperform the market, even if it has done so for the last ten years. Also, a firm can’t promise customers that it’ll earn them at least 8 percent on their investments each year. You can’t possibly look into a crystal ball and reliably predict what’ll happen in the future. Roman numeral II is okay, because it lists what every company should strive to do: keep investment goals in mind.

An investor holds a portfolio of securities that includes a significant number of preferred stocks. Which of the following is the BEST option strategy to protect against a decline in the value of the preferred stock?
Sorry, that is not the correct answer. The correct answer is: A. Buy interest rate calls
Preferred stock is similar to bonds in that bonds receive consistent interest payments and preferred stock receives consistent dividends. The interest payments on both bonds and dividends on preferred stock are somewhat tied to prevailing interest rates. Because there’s an inverse relationship between prices and rates, if interest rates drop, bond prices and preferred stock prices increase. However, if interest rates increase, bond prices and preferred stock prices drop. A savvy investor can protect his or her preferred stock prices by buying interest rate calls because even when the preferred stock prices drop, the investor can make money on the interest rate calls.

A Regulation A offering is
You are correct. The answer is: A. an offering of securities worth $5,000,000 or less in a 12-month period
Regulation A offerings are exempt from full registration requirements, provided the issuer doesn’t sell more than $5,000,000 worth of securities in a 12-month period.

You are about to open an account for a new customer. Which of the following information is required to open the account? I. The customer’s occupation and employer II. The customer’s legal name and address III. The customer’s educational background IV. The customer’s tax ID or Social Security number
You are correct. The answer is: D. I, II, and IV only
To open a new account for a customer, you need the customer’s legal name and address, the type of account that he or she is opening (cash, margin, corporate, and so on), the customer’s age, the customer’s tax ID number or Social Security number, and the customer’s occupation and employer. Customers’ educational backgrounds are irrelevant (even all those earth-shattering events noted in their permanent records from grade school).

Which of the following documents does a limited partnership file with the SEC prior to making a public offering?
A limited partnership has to file a certificate of limited partnership with the SEC (Securities and Exchange Commission) before making a public offering, so the answer is A. The agreement of limited partnership, which is received by limited and general partners, describes the roles and responsibilities of each. Investors fill out the subscription agreement prior to being accepted as limited partners; this document includes information such as the investor’s net worth and annual income, states that the investor understands the risks involved, and so on. The corporate charter (bylaws) is filed by a corporation, not a limited partnership.

Which of the following is NOT TRUE of stock index options?
You are correct. The answer is: B. If the option is exercised, the seller delivers shares of the underlying stock.
When looking at the answers, you notice that answers A and B oppose each other, so logically, one of them has to be right. The false answer is B, because index options are settled in cash and not in the underlying stock.

Mr. Smith, who has a 401k, is leaving his job. After Mr. Smith withdraws the money from the 401k, how long does he have to roll over money into an IRA?
After withdrawing money from a pension plan, individuals have 60 days to roll over the money into an IRA without being taxed.

You’re setting up a strategic asset allocation plan for one of your 60-year-old customers. Which of the following would be correct?
Sorry, that is not the correct answer. The correct answer is: A. You would have your customer invest 40 percent of her portfolio in stocks and 60 percent in bonds and cash.
As investors grow older, they don’t have the ability to take as much risk as they could’ve taken when they were younger. Therefore, you should shift more of the investments to fixed income securities and cash equivalents. The standard strategic asset allocation plan calls for investing 100-minus-the-investor’s-age percent into stocks and the balance into bonds and cash. This customer should invest 40 percent in stocks and the rest in bonds and cash.

Which of the following are characteristics of corporate debt securities? I. They are senior securities. II. They represent ownership of the issuing corporation. III. They are issued to raise working capital. IV. They represent a loan to the issuing corporation.
You are correct. The answer is: D. I, III, and IV only
Roman numeral I is in all the answer choices, so you don’t even have to read it; debt securities are senior securities, meaning that in the event of issuer bankruptcy, holders of debt securities will receive payment before preferred stockholders and common stockholders. Roman numerals II and IV are mutually exclusive, so you can immediately eliminate choice C — debt securities (bonds) are loans to the issuer, as compared to stocks, which represent ownership of the issuing corporation; IV is correct, so you can cross off A. Issuers issue bonds to raise working capital (the amount of money a corporation has available), so III is true; the answer is D

An investor sells ABC Corp. stock at a loss and buys ABC call options within 30 days. Which of the following is TRUE?
A. The loss deduction will be disallowed.
B. The loss deduction will be allowed.
C. The loss deduction can be used to offset capital gains.
D. A portion of the loss can be used depending on whether the holding period was short or long-term.
You are correct. The answer is: A. The loss deduction will be disallowed.
Answers A and B are opposites, so one of them is most likely true. This question is an example of the wash sale rule. The wash sale rule states that an investor can’t sell securities at a loss and then buy the same security or anything convertible into the same security within 30 days (prior or after) and be able to claim the loss. This investor bought a call option, which gives him or her the right to buy ABC stock at a fixed price, thereby violating the wash sale rule. Therefore, the loss is disallowed and would result in the cost basis being adjusted for the purchase.And in case you’re wondering, the cost basis is used to determine capital gains or losses. If an investor were to purchase a stock for $40 per share and sell it for $35 per share, he or she would have a $5 capital loss per share. If this investor violated the wash sale rule, the loss on the initial sale wouldn’t be allowed, but the cost basis of the new securities purchased would be increased by $5 per share.

Which of the following would LEAST likely purchase a general obligation bond?
A. A bank
B. B. An insurance company
C. C. A retirement plan
D. D. A mutual fund
Sorry, that is not the correct answer. The correct answer is: C. A retirement plan
General obligation bonds (or any municipal bonds) don’t make much sense for retirement plans because municipal bonds have the lowest yields of all bonds. Because retirement plans already receive a tax advantage, higher yielding bonds would make more sense for retirement plans.

You should consider which of the following before recommending a municipal security to a customer or potential customer? I. The customer’s state of residence II. The rating of the municipal security III. The customer’s tax bracket
You are correct. The answer is: D. I, II, and III
All the listed choices are important. You should know the customer’s state of residence because if he or she is buying a municipal bond issued from his or her own state, the interest received is triple tax-free (federal, state, and local) instead of just being tax-free on the federal level. The credit rating is important because some investors can take more risk than others. Additionally, the tax bracket is important because investors in higher tax brackets tend to find municipal bonds more valuable because they pay a larger percentage in taxes than investors in lower tax brackets when investing in corporate bonds.

Which TWO of the following are TRUE regarding Treasury STRIPS? I. The security’s principal and interest are paid at maturity. II. The security’s interest is paid semiannually, and the principal is paid at maturity. III. Investors must pay tax on interest earned annually. IV. Investors do not pay tax on interest earned until maturity.
You are correct. The answer is: A. I and III
Series 7 test designers are sly, and they often slip in two questions in one, as I’ve done here. Treasury STRIPS (T-STRIPS) are purchased at a discount and mature at par value. Therefore, investors don’t receive the principal and interest until maturity, so I is correct. However, the investor has to pay tax on the accretion (growth) each year, which means III is also true. The answer is A.

An active account is one in which trading was done during that month. Account statements should be sent out monthly for active accounts, quarterly (once every three months) for inactive accounts, and semiannually (once every six months) for mutual funds.

When a variable annuity is “annuitized,” the holder of the annuity contract receives
Sorry, that is not the correct answer. The correct answer is: A. a fixed number of annuity units based on the value of the accumulation units
For this problem, watch the wording. The question makes distinctions between numbers and values. When a variable annuity is annuitized during the payout phase, the investor receives a fixed number of annuity units based on the value of the accumulation units.

An investor owns 4 TUV Oct 50 puts. If TUV announces a 5-for-2 split, what is the investor’s new position after the split?
Sorry, that is not the correct answer. The correct answer is: D. 4 TUV October 20 puts, 250 shares each
When dealing with an uneven split, which is any split that is not for 1 (for example, a 5-for-2, 3-for-2, 5-for-3, and so on), you leave the number of option contracts the same and adjust the strike price and the number of shares per option. Start by adjusting the shares. Here you have a 5-for-2 split, so the investor has 5 shares for every 2 that he or she had before:
Which of the following transactions must settle on the third business day after the trade date? I. A customer buying closed-end fund shares through a broker-dealer II. A broker-dealer buying a T-note for its own account III. A customer selling a municipal bond using the services of a broker-dealer IV. A broker-dealer buying a mortgage bond from another broker-dealer
You are correct. The answer is: D. I, III, and IV only
The regular way settlement for U.S. government bonds (like the T-note) is one business day after the trade date. All the other choices listed settle in three business days after the trade date.

A floor broker asks a specialist to stop stock. Which of the following is/are TRUE about stopping stock? I. The specialist is guaranteeing a price. II. The specialist needs permission from an exchange official prior to stopping stock. III. This may only be done for public orders.
Sorry, that is not the correct answer. The correct answer is: C. I and III only
Stopping stock is a method a specialist uses to guarantee that a customer’s order will be executed at a specific price. Stopping stock may be done only for public orders. Because the specialist has his or her own inventory, a specialist doesn’t need permission to stop stock.

SIPC (Securities Investor Protection Corporation) covers investors in the event of broker-dealer bankruptcy. SIPC covers each investor up to $500,000, of which no more than $100,000 can be cash

ABC Corp. would like to sell additional shares of common stock to its existing shareholders through a rights offering. Shareholders who would like to subscribe to the rights offering must send the purchase amount with the rights certificate to the
An issuer (corporation) hires a rights agent, who’s responsible for maintaining records of the names of the rights certificate holders; that record-keeping is a lot easier when shareholders let the agent know what’s going on. (By the way, corporations also hire transfer agents, who are responsible for keeping a list of securities holders and for canceling old certificates and delivering new certificates for each trade. Sometimes a transfer agent also acts as a rights agent, so if rights agent weren’t one of the choices, you could look for transfer agent among your options.)

In which of the following documents is the allocation of orders outlined?
You are correct. The answer is: D. Syndicate agreement
The allocation of orders (the priority for customer orders to be filled) is found in the syndicate agreement (the agreement among underwriters). As for the rest of the documents, the official notice of sale is an invitation to underwriters from municipalities announcing that they’re accepting bids on a new issue. The indenture is a contract between a bond issuer and bondholders that states certain contract terms, such as whether the bond is callable, when the bond matures, the coupon rate, and so on. The official statement, a document prepared for municipal issues, is similar to a prospectus; the official statement includes items such as the offering terms, a description of the bonds, a description of the municipality, and so on.

Which of the following is a lagging economic indicator?
Sorry, that is not the correct answer. The correct answer is: C. Prime rate
A lagging economic indicator mirrors a leading indicator but reaches its peaks and troughs at a later date. The prime rate (the interest rate that banks charge its best customers for loans) is a lagging indicator. Unemployment figures and the Fed funds rate are leading indicators, and industrial production is a coincidental indicator. By the way, leading indicators may be used to help predict how the economy is going to perform. Coincidental indicators indicate the current state of the economy.
An ADR is a
Sorry, that is not the correct answer. The correct answer is: A. foreign security trading in the U.S.
Here’s a straight definition question. An ADR is an American Depositary Receipt. ADRs are receipts for foreign securities traded in the United States.

Which of the following documents must a general partner sign to officially accept a new limited partner?
Sorry, that is not the correct answer. The correct answer is: C. Subscription agreement
The general partner signs the subscription agreement when he or she officially accepts a new investor (limited partner); the subscription agreement is usually sent in with some form of payment. As for the other documents, the certificate of limited partnership must be filed with the SEC (Securities and Exchange Commission) prior to a public offering. The agreement of limited partnership describes the roles and responsibilities of limited and general partners. And a new account form is filled out when a registered rep takes on a new customer.

Place the following Dow Jones averages in order from largest to smallest: I. Transportation II. Utilities III. Industrials
Sorry, that is not the correct answer. The correct answer is: C. III, I, II
The Dow Jones composite is made up of 63 exchange-listed stocks and 2 OTC (over-the-counter) stocks. The largest and most often quoted of the Dow Jones averages is the DJIA (Dow Jones Industrial Average), which is made up of 30 stocks. The next-largest is the transportation average, with 20 stocks, and the smallest is the utilities, which is made up of 15 stocks. Note: If you put the names of the averages in alphabetical order, you automatically order them from largest to smallest.

Term bonds are quoted
A. on a nominal yield basis B. on a yield-to-maturity basis C. on a current yield basis D. according to a percentage of dollar price
Term bonds are also called dollar bonds because they’re quoted according to a percentage of dollar price. For example, a term bond that’s trading for $1,100 would be quoted as 110 (110 percent of $1,000 par).

All of the following are possible ways that a partnership may be terminated EXCEPT
Only limited partners have the right to vote, not general partners.
Sorry, that is not the correct answer. The correct answer is: C. the general partners vote to terminate the partnership

Which TWO of the following statements are TRUE? I. Mutual fund ownership represents ownership interest in the securities held by the fund. II. Mutual fund ownership represents ownership interest in the fund. III. Dollar cost averaging allows investors to pay a lower average cost per share than the average price per share when the market price is fluctuating. IV. Dollar cost averaging produces a higher average cost per share than average price per share when the market price is fluctuating.
Although mutual funds purchase several different securities, an investor who owns mutual fund shares owns interest in the fund, not the securities that the fund holds. Dollar cost averaging is purchasing the same dollar amount of a security periodically (for example, $500 per month). Dollar cost averaging produces a lower average cost per share for investors when the price of the security is fluctuating because the investor can purchase more shares when the price is low and fewer shares when the price is high.

Which of the following securities transactions settles regular way in three business days after the trade date? I. ABC common stock II. DEF mortgage bonds III. Suffolk, NY, GO bonds IV. Treasury notes
You are correct. The answer is: D. I, II, and III only
Equity securities, corporate bonds, municipal bonds, and U.S. government agency securities settle regular way (the standard completion of a transaction) in three business days after the trade date. Options, Treasury bills, Treasury notes, and Treasury bonds settle in one business day after the trade date. Cash transactions (naturally, trades completed with cash) settle the same business day as the trade date.

A trade confirmation must be sent to each customer
You are correct. The answer is: D. on or before the settlement date
A trade confirmation has to be sent to the customer on or before the completion of the transaction, which is the settlement date.

Under exchange rules, which of the following is considered sales literature?
A. Exchange-produced literature sent to potential customers
B. B. Seminar transcripts sent to potential customers
C. Brochures sent to potential customers
D. All of the above
You are correct. The answer is: D. All of the above
Any material sent to potential customers with the intent to induce a sale is considered sales literature.

Kawabunga Surfboard and Ironing Board Corp. would need shareholder approval to I. give its shareholders a cash dividend II. give its shareholders a stock dividend III. split its stock IV. reverse split its stock
Sorry, that is not the correct answer. The correct answer is: C. III and IV only
If a corporation wants to split its stock, it needs shareholder approval. However, shareholder approval is never necessary when a corporation wants to pay a dividend. The board of directors decides when a dividend is paid. You can imagine if investors got to vote on dividends, they might want one every week.

Ms. Moneyhill has an account with a broker-dealer that just filed for bankruptcy. Her account included $350,000 in securities and $200,000 in cash. Ms. Moneyhill is covered by the SIPC for a total of
You are correct. The answer is: B. $450,000
SIPC (Securities Investor Protection Corporation) is a nonprofit corporation created to protect clients of firms that are forced into bankruptcy. SIPC covers each investor for up to $500,000, of which no more than $100,000 can be cash. Ms. Moneyhill is completely covered for the $350,000 in securities, but for only $100,000 of the cash, for a total of $450,000.

Which of the following are defensive industries? I. Pharmaceuticals II. Clothing III. Food IV. Automotive
You are correct. The answer is: B. I, II, and III only
Pharmaceuticals, clothing, and food are all defensive industries because people have to buy these items no matter how the economy is performing. However, new car sales suffer if the economy is not strong (people will wait another year or two to purchase a new car).

Which of the following federal legislations requires the full and fair disclosure of all material information about new issues?
You are correct. The answer is: B. The Securities Act of 1933
The Securities Act of 1933 is the federal legislation that sets forth the rules for selling new issues, so the answer is B. The Securities Exchange Act of 1934 is the federal legislation that sets forth rules for outstanding securities. The Act of 1934 established the SEC, and it regulates the exchanges and the over-the-counter market. Regulation T covers the amount of credit that can be extended to customers for cash and margin accounts. The Glass-Steagall Act of 1933 prohibits commercial banks from underwriting securities.

During periods of stable interest rates, which of the following types of bonds would have the most volatile price changes?
You are correct. The answer is: A. Convertible bonds
Most bond prices should remain pretty stable during periods of stable interest rates. Convertible bonds are the exception to that rule because convertible bonds are convertible into common stock of the same issuer. Because stock prices are more volatile than bond prices, convertible bonds would be volatile also because they usually stay close to parity with the stock price. Parity is when the stock and convertible bond are trading equally — for example, when a convertible bond trading at $800 is convertible into $800 worth of stock.

You are attempting to help one of your customers diversify her portfolio. Which of the following are valid diversification strategies? I. Buying different types of instruments (equity, debt, packaged, and so on) II. Buying bonds with different maturities III. Buying securities from different industries IV. Buying bonds with different ratings
Sorry, that is not the correct answer. The correct answer is: D. I, II, III, and IV
All the choices listed are ways of diversifying a portfolio. When an investor is diversifying a portfolio, he or she is attempting to spread out the risk.

Marty would like to invest in securities that generate the greatest current income with only a moderate risk. Which of the following would BEST meet Marty’s goals?
Sorry, that is not the correct answer. The correct answer is: C. AA rated corporate bonds
The best answer is AA rated bonds, because the high (AA) rating indicates little credit risk. Additionally, bonds pay interest, which would nicely meet Marty’s goal of current income. By contrast, companies in bankruptcy (reorganization) issue income bonds, making them quite risky. Although convertible preferred stock pays a consistent dividend, it’s not considered as safe as AA rated bonds. Additionally, common stock of a new company without an established track record is considered a speculative (risky) investment.

48000 market val
19200 margin
28800 equity
(c-19200)
C*.25=equity
c-19200=equity
c=4c-76800
c=25600
Sorry, that is not the correct answer. The correct answer is: C. $25,600

An investor sells short 1,000 shares of DEF at $60. If ABC goes up to $65, by how much is the account restricted?
Sorry, that is not the correct answer. The correct answer is: D. $7,500

John would like to open a margin account at Twilight Broker-Dealer. Which of the following documents is optional for John to sign?
Sorry, that is not the correct answer. The correct answer is: B. Loan consent agreement
Although a lot of firms may require the loan consent agreement in real life, as far as the Series 7 exam is concerned, this form is optional. This leeway allows the brokerage firm to lend the customer’s securities to short sellers. Whenever a customer is opening a new account, a new account form is necessary. Additionally, a hypothecation agreement is mandatory for margin accounts; it gives authority to the broker-dealer to pledge customer’s margined securities as collateral.

Who are the owners of publicly held corporations? I. Common stockholders II. Mortgage bondholders III. Preferred stockholders IV. Guaranteed bondholders
Sorry, that is not the correct answer. The correct answer is: B. I and III
This question should give your mind a rest. Stockholders are owners of a corporation; bondholders are creditors.

All of the following are TRUE about IDRs EXCEPT
You are correct. The answer is: C. They need voter approval to be issued.
An IDR (industrial development revenue bond) is issued to build a facility for a private user or corporation, so choice B looks sound. The principal and interest on the bonds come from lease payments that the private user or corporation makes; therefore, the issuing municipality is not responsible for paying off the debt, so A is true. Additionally, IDRs are subject to the substantial user rule, which states that a corporation can’t buy its own IDRs and receive tax-free interest. IDRs do not need voter approval to be issued because they’re not backed by tax collections, which makes C the correct answer.

Which of the following are TRUE statements regarding civil penalties that may be imposed for insider trading violations? I. Civil penalties may be imposed only on registered persons. II. The civil penalty may be up to three times the profit gained or loss avoided on an illegal transaction. III. A broker-dealer may be held liable if one of its registered reps was not properly supervised while committing an insider trading violation. IV. The violation is defined as buying or selling securities while in possession of material non-public information.
You are correct. The answer is: B. II, III, and IV only
Civil penalties for insider trading may be imposed on anyone violating insider trading rules, whether registered or not. Therefore, if Roman numeral I is out, the only correct answer is B. All the other Roman numeral answer choices are true statements.

All of the following debt securities have a fixed maturity date EXCEPT A. T-STRIPSB. debenturesC. revenue bondsD. collateralized mortgage obligations
You are correct. The answer is: D. collateralized mortgage obligations
Collateralized mortgage obligations (CMOs) are mortgage-backed securities. Because mortgages are often paid off ahead of time when people refinance their loans or sell their homes, CMOs do not have a fixed maturity date.

HIJ Corp. declares a dividend of $0.80 per share. On the ex-dividend date, the specialist adjusts HIJ prices downward by $0.80 per share for which TWO orders? I. Buy stop II. Buy limit III. Sell stop IV. Sell limit
You are correct. The answer is: B. II and III
BLiSS (buy limit, sell stop) orders are reduced on the ex-dividend date to reflect the dividend; SLoBS (sell limit, buy stop) orders remain the same. To remember that SLoBS order don’t change, think of the phrase “once a slob, always a slob.”

All of the following are good delivery for a trade of 570 shares of stock EXCEPT
Sorry, that is not the correct answer. The correct answer is: D. three 100-share certificates and nine 30-share certificates
Always look at the shares first. The certificates should be in multiples of 100 shares (100, 200, 300, and so on), divisors of 100 shares (1, 2, 4, 5, 10, 20, 25, 50), or amounts that add up to 100 shares (80 + 20, 70 + 30, 60 + 30 + 10, and so on) to be considered good delivery. However, the odd lot portion (in this case 70) doesn’t have to fit. Answer D is incorrect because 30 is not a divisor of 100 and nothing matches it to bring the total up to 100.

The U.S. government directly backs which of the following issue debt obligations?
You are correct. The answer is: B. Government National Mortgage Association
Government National Mortgage Association (GNMA) securities are directly backed by the full faith and credit of the U.S. government. As for the rest, the Federal National Mortgage Association (FNMA) is a private company that issues securities on the NYSE. Although FNMA is privately owned, it operates under a congressional charter and has the implied backing of the U.S. government. Federal Farm Credit Banks provides loans to farmers and is a government-sponsored corporation, so it has the implied backing of the U.S. government. Federal Home Loan Mortgage Corporation (FHLMC, or Freddie Mac) is a publicly traded corporation that provides a secondary market for mortgages. Like FNMA and Federal Farm Credit Bank securities, Freddie Mac securities have the implied backing of the U.S. government.

John Jones owns property with a market value of $550,000 and an assessed value of $525,000. If the tax rate of the municipality is 16 mills, what is the ad valorem tax for John?
You are correct. The answer is: A. $8,400
If an answer looks weird, it probably is. It’s highly unlikely that someone who owns a house worth $550,000 is paying more than $80,000 in property taxes. Therefore, your two more likely answers are $8,400 and $8,800.Always use the assessed value when making your calculations, because that’s what the investor will be taxed on, not the market value. The property tax rate is always in mills, and 1 mill is equal to $0.001, or a tenth of a cent (remember, the two l’s in the word mills means you have two zeros after the decimal point). Multiply the assessed value by 16 and then by 0.001, to get the correct answer: Ad valorem tax = $525,000 × 16 × 0.001 = $8,400

Bonds issued by a corporation that are secured by stocks and/or bonds held by a trustee are called
You are correct. The answer is: D. collateral trust bonds
Collateral trust bonds are bonds secured by stocks and bonds that the issuer owns. In the event that the issuer misses interest or principal payments, the trustee sells off the securities to make the payments. Debentures are bonds that are backed not by collateral but by the good faith and credit of the issuer. The principal and interest on guaranteed bonds are backed by another entity, such as a parent company; if the issuer defaults on interest or principal payments, the other entity is required to make up the missing payment(s). Equipment trust certificates are secured by equipment owned by the issuer. This type of bond is usually issued by transportation companies. If the issuer defaults, the equipment is sold by the trustee to make up for the missing interest and/or principal payments.

Which of the following is TRUE of straight line depreciation when compared to accelerated depreciation? I. Straight line depreciation allows for greater deductions in the early years. II. Accelerated depreciation allows for greater deductions in the early years. III. Straight line depreciation allows for lower stated income in the early years. IV. Accelerated depreciation allows for lower stated income in the early years.+
You are correct. The answer is: D. II and IV

M1 includes all of the following EXCEPT
A. Checking accounts
B. B. Coins
C. C. Savings deposits
D. D. Currency in circulation
M1 includes easily spendable currency such as currency in circulation, coins, and checking accounts. Savings deposits (time deposits) are part of M2.

BBBB callable bonds were issued with five-year call protection from the issue date. The call protection would be especially valuable to investors if interest rates are
Sorry, that is not the correct answer. The correct answer is: A. falling
Call protection is the number of years an issuer would have to wait before calling in its callable bonds. Issuers would love to call their bonds in whenever interest rates drop because they could issue new bonds with a lower coupon rate. However, this question asks about value to investors. As an investor, you wouldn’t want the bond called if interest rates drop because your bond price would increase. Therefore, call protection is most valuable to investors when interest rates fall.

The MOST diversification can be found in which type of oil and gas DPP?
You are correct. The answer is: D. Combination program
Oil and gas combination programs offer diversification among exploratory, developmental, and income producing areas. Exploratory programs attempt to find and recover undiscovered oil or gas. Developmental programs drill in areas near where oil and gas reserves have been previously found. Income programs invest in wells that are already producing.

Income bonds are extremely risky investments because they’re issued by companies in bankruptcy (reorganization). Income bonds are issued at a deep discount from par and do not pay interest until the company becomes profitable (if ever).

A due diligence meeting is attended by I. the managing underwriter II. syndicate members III. the issuer IV. the NASD
C. I, II, and III only
The due diligence meeting is the meeting at the end of the cooling-off period in which a corporation’s officials and underwriters present information and hammer out details about a pending new issue of securities. The due diligence meeting is attended by the managing underwriter, syndicate members, and issuer. The SEC, NASD, NYSE, and so on don’t attend the due diligence meeting.

When does an option contract expire?
Sorry, that is not the correct answer. The correct answer is: D. 11:59 p.m. EST on the Saturday after the third Friday of the expiration month
Option contracts expire at 11:59 p.m. EST (10:59 p.m. CST) on the Saturday after the third Friday of the expiration month.

Hypothecation is not a prohibited activity; it’s the pledge of securities bought on margin that are used as collateral for the loan financing the purchase. Commingling is combining a customer’s fully paid securities with his or her margined securities or combining a customer’s securities with the broker-dealer’s securities. Interpositioning occurs when a broker-dealer or specialist unlawfully inserts himself or herself into a transaction between a buyer and seller in order to profit from the transaction. Backing away is the failure of a market maker to honor a firm quote.

Simplified arbitration is used for
Sorry, that is not the correct answer. The correct answer is: B. member-to-customer disputes not over $25,000
Simplified arbitration is used for member-to-customer disputes not over $25,000. Arbitration is an informal hearing regarding a dispute. The non-member (customer) decides whether the dispute goes to arbitration or through code of procedure (the NASD’s formal procedure for handling complaints). Arbitration decisions are binding and are not open to appeal.

This syndicate member is guilty of withholding. Withholding is the illegal practice of holding back securities for one’s own account instead of offering them to interested investors. As for the other choices, freeriding is when a customer purchases a security and never sends in payment but sells the security to pay for the trade. Marking to the market is adjusting the market value of the securities in a margin account to determine the equity. Discounting is something I threw in there to see if you’d pick it!

Which of the following departments is responsible for transferring orders to the proper market for execution?
You are correct. The answer is: D. The order department
The order department (wire room) is responsible for transmitting orders to the proper market for execution; after a trade is completed, the trade tickets are sent to the registered rep who initiated the trade and also to the purchase and sales department. Here’s what the other departments do: cashiering accepts payments and issues checks to customers, the margin department handles all credit that a broker-dealer extends to a customer, and purchase and sales sends confirmations to customers and fixes any trading errors.

One of your customers is expecting a large amount of passive income for the next few years. Which of the following investments would offset the largest amount of passive income?
A. Real estate income
B. B. Undeveloped land purchasing
C. C. Equipment leasing
D. D. Oil and gas exploratory
Passive income is derived from rental properties and limited partnerships. Only passive losses can offset passive income. The partnership that would provide the greatest losses over the next few years is the oil and gas exploratory program. An oil and gas exploratory program provides big write-offs initially for purchasing land and equipment, paying employees, drilling costs, and so on, without yielding any cash flow until oil is found and being pumped out of the ground, which could take several years.

Customer order tickets must include which of the following information? I. The account number of the customer II. The payment terms for the transaction III. The name of the broker who took the order IV. Whether the order was solicited or unsolicited
Sorry, that is not the correct answer. The correct answer is: B. I, III, and IV only
All order tickets have to include the account number of the customer, the name of the broker who took the order, and whether the order was solicited. However, the payment terms for the transaction aren’t necessary, because the industry regulates the terms and brokerage rules, which don’t change from transaction to transaction.

Antifraud provisions of the Securities Exchange Act of 1934 apply to which of the following securities? I. NYSE listed securities II. NASDAQ listed securities III. Municipal bonds IV. Mutual funds
You are correct. The answer is: D. I, II, III, and IV All securities (even exempt securities) are subject to the antifraud provisions of the Securities Exchange Act of 1934.

Which of the following does NOT affect the marketability of municipal bonds?
You are correct. The answer is: B. Accrued interest
Accrued interest is the amount of interest a buyer has to pay a seller when purchasing a bond in between coupon dates. Because the investor still earns the same amount of interest per day, accrued interest is not a factor when investors are looking to purchase municipal (or any other) bonds.

A municipality generates $40,000,000 in revenues from a facility. The municipality must pay bond interest of $2,000,000, principal of $4,000,000, and operating and maintenance expenses of $16,000,000. What is the debt service coverage ratio?
You are correct. The answer is: A. 4 to 1

Mr. Bullishinski is a U.S. citizen who owns stock issued by a French corporation. Mr. Bullishinski received dividends from the French corporation stock. Which of the following is TRUE of the tax treatment of the dividends received from this investment?
You are correct. The answer is: B. If taxes were withheld by France, Mr. Bullishinski would be able to claim a foreign tax credit in the U.S. for the amount of taxes withheld.
A U.S. citizen who owns stock or bonds issued in another country is liable for federal and state taxes on any interest and dividends received. However, if a foreign tax has been withheld on interest and dividends, the investor is eligible for an income tax credit on the amount withheld.

Who receives the “takedown” in a municipal bond underwriting?
You are correct. The answer is: B. Syndicate members

The certificate that includes the trade details, which your firm sends to a customer on completion of a trade, is called the
Each customer has to receive a written confirmation at or before the completion of each transaction. The confirmation includes details of the trade, including how many shares or bonds, trade price, whether the firm acted as a broker or a dealer, and so on.

Which of the following persons must register as an investment adviser under the Investment Advisers Act of 1940?
You are correct. The answer is: C. Persons who give investment advice for a fee
If you’re making money off all that great (or not-so-great) investment advice you’re offering, you need to sign up. An individual who, as part of his or her business, gives investment advice for a flat fee or a percentage of assets managed has to register as an investment adviser. Registered reps are not considered investment advisers because they receive payment in the form of commission, markups, or markdowns only when a customer buys or sells securities. Registered reps can also work as investment advisers when they pass the Series 65 or Series 66.Registration involves a little more than putting your name and address in the right places on the forms. Individuals who are just working as investment advisers need to pass the Series 65 and 63 exams. Whoever wants to work as a registered representative and an investment adviser has to pass either the Series 7 and 66 exams (a combination of the Series 65 and 63 exams) or the Series 7, 65, and 63 exams.

Under the Securities Exchange Act of 1934, which of the following is considered an “insider”? I. An officer of a company II. The spouse of a company’s president III. A bookkeeper working in a company’s accounting department IV. A 12 percent owner of a company
You are correct. The answer is: D. I, II, III, and IV
According to the Securities Exchange Act of 1934, an insider is anyone who may have inside information on a company that provides someone with an unfair advantage. This definition includes officers, directors, 10-percent stockholders, and so on. Certainly, spouses of officers and bookkeepers for a company would also have access to inside information.

One of your customers is interested in getting involved in options. You should inform your customer that he or she will have to meet margin requirements in which of the following positions?
You are correct. The answer is: B. A naked option contract
Because of the risk involved when selling naked (uncovered) options, they must always be executed in a margin account. However, when an investor has a long combination (owning a put and a call on the same stock with different expiration months and/or strike prices) or a long straddle (owning a put and a call on the same stock with the same expiration month and strike price), the most the investor can lose is the premium paid. And if an investor has a covered option contract, he or she either has the stock available to deliver if exercised or owns an option that will be in-the-money first, so there’s limited risk — no meeting margin requirements, well, required.

All of the following money market instruments do NOT pay interest EXCEPT
You are correct. The answer is: D. jumbo CDs
Most money market instruments are issued at a discount and mature at par value and do not receive interest payments along the way. Jumbo CDs are an exception to the rule because they do receive interest payments.

On the maturity date of Spazm corporate bonds, holders of the bonds
Sorry, that is not the correct answer. The correct answer is: B. receive par value from the issuer
When an investor holds a bond to maturity, the investor receives par value (usually $1,000) from the issuer. After a bond matures (whether corporate, U.S. government, or municipal), an investor stops earning interest.

Systematic risk (market risk) is the risk that securities can decline due to negative market conditions regardless of the performance of the corporations. As for the rest, credit risk is the risk that interest and principal on bonds are not paid on time (or at all). Inflation risk is the danger that a security owned doesn’t keep up with the rate of inflation. And liquidity risk is the risk that a security, such as DPP, is not easily traded.

Shelf registration allows the issuer to delay selling all or part of its stock for up to
Sorry, that is not the correct answer. The correct answer is: D. 2 years
With shelf registration, issuers can delay selling some or all their securities for up to two years. For example, assume that a corporation is authorized to issue 2,000,000 shares but can raise enough money to meet its needs by issuing only 800,000. In that case, it can delay issuing the remaining shares for up to two years and raise additional money as needed.

In a firm commitment underwriting, the underwriter keeps whatever shares it hasn’t been able to sell. Mini-max and all or none are types of best efforts underwritings. In a mini-max underwriting, a minimum number of the issue must be sold or the offering is canceled. In an all or none underwriting, all of the securities must be sold or the offering is canceled.

An investor purchases a 5 percent municipal bond yielding 6.2 percent. If the investor is in the 28 percent tax bracket, the investor would have to purchase a corporate bond yielding what to equal the after-tax yield on the municipal bond?
You are correct. The answer is: D. 8.6 percent

A trade of securities between a bank and an insurance company takes place on the
You are correct. The answer is: D. Fourth market
A fourth market trade is trading between institutions without using the services of a broker-dealer, so the answer is D. A first market trade is a trade of exchange-listed securities trading on an exchange, a second market trade is a trade of over-the-counter securities trading over-the-counter, and a third market trade is exchange-listed securities trading over-the-counter.

Which of the following indicates an increase in the U.S. balance of payments?
Sorry, that is not the correct answer. The correct answer is: C. An increase in U.S. exports
Balance of payments has to do with the flow of money between the U.S. and other countries. If the U.S. balance of payments increases, more money is coming into the U.S. However, answers A, B, and D are all sending money out of the U.S. Answer C is correct because if the U.S. is exporting more, foreign companies will be sending money into the U.S. to pay for those goods.

ABC Corp. would like to open a cash account at DEF Broker-Dealer Corp. In order for ABC Corp. to open the account, it must send a brokerage firm a copy of its
A. corporate charterB. corporate resolutionC. corporate bylawsD. hypothecation agreement
Sorry, that is not the correct answer. The correct answer is: B. corporate resolution
Actually, answers A and C mean the exact same thing, so you know the answer can’t be one of them. In order for a corporation to open a cash account, it has to send in a copy of its corporate resolution, which states who has trading authorization for the account; therefore, the answer is B. Now, if the corporation wants to open a margin account rather than a cash one, it also has to send in a copy of its corporate charter (bylaws), which would need to stipulate that the corporation may purchase securities on margin.

Which of the following is limited regarding a limited tax bond?
You are correct. The answer is: C. The rate of taxes that can be levied to pay off the debt
A limited tax bond is a GO (general obligation) municipal bond issued by a municipality whose taxing power is limited to a maximum tax rate.

A margin account contains the following: Long market value: $50,000 Short market value: $40,000 Debit balance: $20,000 Credit balance: $65,000What is the equity?
50+65-40-20
Which of the following factors may change a customer’s investment objectives? I. The customer’s age II. The customer’s investment experience III. The customer’s marital status IV. The customer’s inheriting a large amount of money
You are correct. The answer is: D. I, II, III, and IV

You are correct. The answer is: B. agreement of limited partnership
The rights and responsibilities of the limited and general partners is listed in the
The agreement of limited partnership describes the rights and responsibilities of both the limited and the general partners, so that’s your answer. The certificate of limited partnership has to be filed with the SEC (Securities and Exchange Commission) prior to making a public offering. Investors fill out the subscription agreement prior to their being accepted as limited partners; the subscription agreement includes information such as the investor’s net worth, his or her annual income, a statement that the investor understands the risks involved, and so on. A prospectus is a formal document that describes details about a new issue.

The Trust Indenture Act of 1939 applies to which of the following issues? I. $50,000,000 Future Biscuit Corporation mortgage bonds II. $20,000,000 Kold Stethoscope Corporation debentures III. $35,000,000 Fred’s Trucking and Shoehorn Corporation equipment trust certificates IV. $50,000,000 U.S. Treasury bonds
You are correct. The answer is: C. I, II, and III only
The Trust Indenture Act protects corporate bondholders. The act requires that all corporate bond issues over $5,000,000 be issued with an indenture (a formal written agreement), which protects bondholder’s rights. The Trust Indenture Act of 1939 does not apply to government issues.

Which of the following is FALSE?
A. Callable bonds usually have higher yields than non-callable bonds. B. Convertible bonds usually have lower yields than non-convertible bonds. C. Long-term bonds have more liquidity risk than short-term bonds. D. Long-term bonds have sharper price movements than short-term bonds.

Which of the following is FALSE?
Sorry, that is not the correct answer. The correct answer is: D. Long-term bonds have sharper price movements than short-term bonds.
Long-term bonds don’t have sharper price movements than short-term bonds. Long-term bonds change more in price than short-term bonds when interest rates fluctuate, but short term bonds change more quickly.

Which of the following is a coincidental economic indicator?
A. Money supplyB. Industrial productionC. Building permitsD. Prime rate
Coincidental economic indicators, like industrial production, tell you how the economy is performing right now and indicate the current state of the economy. Money supply and building permits are leading indicators (they give you an indication of how the economy is going to perform), and the prime rate is a lagging indicator (it confirms leading indicators).

The Fed funds rate is the rate that banks charge each other for loans (usually overnight loans) and is set by banks, not the Federal Reserve. Here’s how the Fed uses the other tools: The discount rate is the interest rate that the Fed charges member banks for loans; if the Fed raises the discount rate, the money supply is tightened, and if it lowers the discount rate, the money supply is eased. The reserve requirement is the percentage of deposits banks must hold each night; if the Fed increases the reserve requirement, the money supply tightens. The Fed can also control the money supply through open market operations (buying and selling U.S. government securities); the Fed can ease the money supply by buying U.S. government securities in the open market and tighten the supply by selling U.S. government securities.

A customer is short an OEX call option in his account. If this customer’s option contract is exercised, he is required to
Sorry, that is not the correct answer. The correct answer is: B. deliver cash equal to the in-the-money amount multiplied by the contract multiplier
The main difference between index options (OEX and SPX) is that exercises of index options are settled in cash instead of by delivery of the underlying security. When an index option is exercised, the seller delivers cash equal to the in-the-money amount multiplied by the contract multiplier (normally 100 shares).

All of the following are TRUE about the official statement of a revenue bond EXCEPT A. it includes information about the tax base of the municipalityB. it does not need to be filed with the SECC. a final version does not need to be preparedD. it must be sent to any potential purchaser
You are correct. The answer is: A. it includes information about the tax base of the municipality The official statement of a revenue bond doesn’t disclose the tax base of the municipal issuer because revenue bonds aren’t backed by taxes. Revenue bonds are backed by the revenues that the backing facility collects. In contrast, GO (general obligation) bonds receive backing from taxes.
Which of the following investments have tax advantages? I. IRAs II. Municipal bonds III. GNMAs IV. Hedge funds
Sorry, that is not the correct answer. The correct answer is: A. I and II only
Both IRAs (individual retirement accounts) and municipal bonds provide investors with tax advantages. When an individual invests in a traditional IRA, he or she can to write off the amount invested from his or her income taxes and not pay tax until the money is withdrawn. The interest payments received from municipal bonds are federally tax-free, which gives investors in higher tax brackets a great tax advantage. GNMAs (Ginnie Maes) are taxed on all levels even though they’re backed by the full faith and credit of the U.S. government.

A municipal bond backed by the proceeds of tax on certain items rather than ad valorem taxes is a
Sorry, that is not the correct answer. The correct answer is: D. special tax bond A municipality collects excise or regressive taxes, usually unrelated to the project financed by the issue, to back a special tax bond. Special taxes are usually on so-called luxury items such as alcohol, tobacco, gambling, gasoline, and so on (though strangely enough, not on pooper scoopers for well-to-do pet owners).

A “penny stock” is defined as an unlisted, non-NASDAQ security trading at:
Sorry, that is not the correct answer. The correct answer is: D $5.00 per share or less
The Securities and Exchange Commission (SEC) defines a penny stock as one that’s not listed on an exchange or NASDAQ and trades at $5.00 or less per share.

Mrs. Johnson makes a gift of securities to her 12-year-old son Zeb. Gift taxes are based on
Sorry, that is not the correct answer. The correct answer is: B. the value of the securities on the date of the gift
The gift taxes are based on the value of the securities on the date of the gift. The donor pays any gift tax due.

Abstracts of official statements must be kept on file for at least
You are correct. The answer is: C. 3 yearsAbstracts of municipal securities official statements are considered advertisements and have to be kept on file with the broker-dealer for at least three years.

Municipal bond trades settle regular way in You are correct. The answer is: B. three business days

All of the following enforce MSRB rules EXCEPT A. Comptroller of the CurrencyB. FGICC. NASDD. Federal Reserve Board
MSRB (Municipal Securities Rulemaking Board) rules are enforced by the SEC, NASD, FDIC, Federal Reserve Board, or Comptroller of the Currency. The FGIC is an insurance company for municipal issuers and does not enforce rules.

Which of the following would be considered promotional literature needing approval from a municipal securities principal? I. Abstract of an official statement II. Market letter III. Research report IV. Form letter
Sorry, that is not the correct answer. The correct answer is: D. I, II, III, and IV
An abstract of an official statement is an excerpt or adjustment to the original official statement. After a prospectus or official statement is adjusted, the document needs approval from a principal before it can be delivered to investors. A market letter is a newsletter that a brokerage firm uses to promote its services. A form letter is a generic letter sent to several customers or potential customers. All the choices listed are considered advertising or sales literature that needs approval from a principal.

Which of the following accounts may be opened without power of attorney?
You are correct. The answer is: B. An aunt opening an account for a minor
When one adult opens an account for another adult, a written power of attorney is required. However, when an adult opens an account for a minor (a Uniform Gifts to Minors Act/UGMA account), a written power of attorney isn’t necessary because the minor couldn’t legally sign it, anyway. None of the above, by the way, will almost never be the correct answer to a question on the Series 7 exam.

DEF Corp. owns securities from different companies. Which of the following investments would provide the BEST after-tax return for DEF if DEF is in the 34 percent tax bracket?
LMN Debentures yielding 7 percent
Suffolk, NY, GO bonds yielding 5 percent
C. T-bonds yielding 5.5 percent
D. WXY preferred stock with a 7 percent dividend
Sorry, that is not the correct answer. The correct answer is: D. WXY preferred stock with a 7 percent dividend
Seventy percent of the dividends paid from one corporation to another are exempt from taxes. When looking for the best after-tax investment for a corporation, look for the common stock or preferred stock that’s paying the highest dividend.

Which of the following situations would likely cause the value of the U.S. dollar to rise in value when compared to the Japanese yen? A. U.S. investors’ purchasing Japanese securitiesB. A decrease in the discount rateC. Japanese investors’ buying U.S. securitiesD. An increase in U.S. exports being sent to Japan
Sorry, that is not the correct answer. The correct answer is: A. U.S. investors’ purchasing Japanese securities
For the value of the U.S. dollar to rise when compared to the Japanese yen, the money supply in the U.S. has to tighten up when compared to Japan’s. When U.S. investors purchase Japanese securities, money leaves the U.S. (which tightens the U.S. money supply) and goes to Japan (which eases Japan’s money supply). In this scenario, the U.S. dollar would increase in value when compared to the Japanese yen. Answers B, C, and D, on the other hand, ease the U.S. money supply and make the U.S. dollar weaker.

Rule 144 covers the resale of I. restricted securities II. control securities III. limited partnerships IV. securities purchased through a Regulation A offering
You are correct. The answer is: A. I and II only
Rule 144 covers the resale of restricted securities (usually purchased through private placement) and control securities (securities held by persons who are in a position to influence the issuer, such as large shareholders, directors, officers, and so on).

Which of the following are characteristics of preferred stock? I. A fixed rate of return II. Priority claim to assets upon dissolution when compared to common stock III. A fixed maturity date IV. A priority claim to dividends over common stock of the same company
You can get this answer by choosing the one that doesn’t belong. Bonds, not stock, have a fixed maturity date, so Roman numeral III is not a characteristic of preferred stock. Therefore, the only answer that works is D.

Rule 346 says that registered reps have to notify and get permission in writing from their employer before engaging in another business. That rule covers moonlighting (choice A), having any financial interest in a privately held financial-services company (choice B), or being an officer or director of another company (choice C).

Which TWO of the following are zero-coupon government securities? I. Treasury receipts II. Treasury bonds III. Treasury notes IV. Treasury bills
Treasury receipts (Treasury STRIPS) and Treasury bills are zero-coupon securities because they don’t receive interest payments; they’re issued at a discount, and they mature at par value.

Which of the following documents explains the risks of investing in options and must be sent out to potential customers prior to approval of an options account?
Sorry, that is not the correct answer. The correct answer is: B. ODD
Potential customers have to receive the ODD (options risk disclosure document). The ODD explains the risk involved for options investors, such as the possibility of losing all money invested. The customer has to sign and return the OAA (options account agreement) 15 days after approval of the options account.

7)
Although organizations differ in their internal practices, which of the following correctly represents the typical flow of a customer’s order through a broker-dealer’s office?

I. Margin department
II. Wire Room
III. Cashier
IV. Purchases and Sales

(a) II, IV, I, III
(b) II, I, III, IV
(c) I, II, III, IV
(d) IV, II, I, III
Next
Explanation:
The order goes through the Wire Room for transmittal to the appropriate trading desk/market. The Purchases and Sales department creates the follow-up documents, including confirmations. All orders go through theMargin department to account for the funds. Finally the order ticket goes to the Cashier where securities and money are transferred.
In Progress : Question No. 8 of 602
8)
Under the NASD’s Conduct Rules, to which of the following does the segregation rule apply?
I. Securities a customer has purchased in a margin account with securities in lieu of cash.
II. Fully-paid securities purchased in a margin account.
III. Securities held by the broker-dealer in a margin account in excess of 140% of the client’s debit balance.
IV. Only those securities held in the client’s name.

(a) II, III
(b) IV
(c) II, III, IV
(d) I, II, III
Next
Explanation:

The Rule for segregation of fully paid and excess margin securities requires that a broker-dealer must completelysegregate, or completely separate, the securities described in choices II and III. The broker-dealer may not commingle, or mix client’s fully-paid and excess margin securities with those of the firm. Excess margin stocks are those held by the broker-dealer in excess of that stockrehypothecated by the broker-dealer.

9)
Quotes for stocks on Level 1 of the NASDAQ system are most accurately described by which of the following?

(a) Workout
(b) Subject
(c) Bona Fide
(d) Firm
Next
Explanation:
Level 1 of the NASDAQ system only gives the inside market; which is the highest bid and lowest ask prices from Level 2. It does not show the size of the quotes and is therefore not a firm quote, but subject.

In Progress : Question No. 13 of 602
13) What information may be found by a RR when looking at “Pink Sheets”?
(a) Daily quotations on corporate bonds
(b) Quotations on municipal bonds in the secondary market
(c) Quotes on penny stocks
(d) Quotations on non-NASDAQ OTC stocks
Next
Explanation:
Pink Sheets is a daily publication showing quotations on non-NASDAQ Over-The-Counter stocks. The Yellow Sheets are a similar publication for corporate bonds. Pink Sheet stock is not necessarily “penny stock,” it is simply stock of companies that are not quoted on the NASDAQ system. Pink Sheets are being supplanted by the OTC Bulletin Board (OTCBB), an electronic service.
he MSRB rules require a 90 day apprenticeship period for a new municipals representative. During this period, the apprentice rep may actually conduct business with other securities professionals, but not with the public.

In Progress : Question No. 16 of 602
16)
The underwriters for the MSM Corporation have filed a registration statement, under the Securities Act of 1933, for the firm’s IPO. The securities are currently in registration. Which of the following activities may the underwriters engage in at this time?

I. Take indications of interest
II. Send out “red herring” prospectuses
III. Provide a copy of an initial research report along with the preliminary prospectus
IV. Hold a due diligence meeting

(a) II, III
(b) I, II, III, IV
(c) II, IV
(d) I, II, IV
Next
Explanation:

When a registration statement has been filed and the securities are in registration, there is a 20-day cooling-off period. In general, there are only three activities permitted: taking indications of interest, sending out “red herring” prospectuses and holding a due diligence meeting.

There is generally no communication with the public (other than the preliminary prospectus, or red herring) during this period.

Quit
In Progress : Question No. 17 of 602
17) A customer of a NYSE firm has decided to open a margin account. As her first trade she buys 100 shares of LIB stock at $15 per share. With Regulation “T” at the standard 50%, what will be her minimum required deposit for this transaction?
(a) $2,000
(b) $1,500
(c) $3,500
(d) $750
Next
Explanation:
The initial equity in a new margin account, by NYSE and NASD rules is $2,000 unless the total transaction is less than $2,000 as it is in this case. The customer, for the opening transaction will deposit the total amount of the purchase.
In Progress : Question No. 18 of 602
18)
Under the rules of the CBOE, what is the maximum period of time for which a LEAPS contract may be written?

(a) 24 months
(b) 12 months
(c) 9 months
(d) 39 months
Next
Explanation:

Although LEAPS (Long-Term Equity AnticiPation Securities) contracts are most commonly traded with expiration dates of up to 24 months, the CBOE rules allow for contracts of up to 39 months.

In Progress : Question No. 19 of 602
19)
Under exchange rules, a GTC order on the specialist’s book must be renewed with the specialist at what intervals?

(a) Semi-annually, at the beginning of April and beginning of October
(b) On the first business day of each quarter
(c) Annually by the end of December
(d) Semi-annually, at the end of April and end of October
Next
Explanation:

The rule specifies the end of April and October. Don’t pick the first of April—that’s April Fool’s Day!
In Progress : Question No. 21 of 602
21)
All exchange-traded stock is, by definition, marginable. Which of the following decides, and maintains a list of, OTC stocks that are marginable?

(a) SEC
(b) FRB
(c) NASD
(d) NYSE
Next
Explanation:
Actually, this should be a fairly easy question if you recall that the FRB—the Federal Reserve Board—sets the margin requirements themselves. By extension, the FRB also decides which OTC stocks are marginable.
Quit
In Progress : Question No. 36 of 602
36)
Trades by broker-dealers in which of the following securities are considered to settle in clearinghouse funds?

(a) Municipal
(b) Commodities
(c) Options
(d) U.S. Government
Next
Explanation:
Municipal securities settle, regular way, T+3 (trade + 3 business days) inclearinghouse funds. Government securities settle T+1, in federal funds. By the way, commodities are not securities.

37)
Which of the following electronic/computer systems is associated with market order execution on the NYSE?

(a) SOES
(b) Super Dot
(c) OSS
(d) Super Montage
Next
Explanation:

The Super DOT (Designated Order Turnaround) is a NYSE automated system for execution of small and moderate-sized orders by bypassing the floor brokers and going directly to the specialist’s post for execution. The system is primarily used for market orders. SOES and Super Montage are associated with OTC markets and OSS, with the options exchange.
38)
A broker-dealer in Philadelphia is a market-maker in LIB stock, a NYSE listed stock. In a telephone transaction, the firm sells 5,000 shares of LIB to a broke-dealer in Dallas. This transaction would be known as:

(a) A NYSE trade
(b) A NASDAQ Intermarket Transaction
(c) A NASDAQ transaction
(d) A fourth-market transaction
Next
Explanation:
When listed stock is traded in transactions away from the exchange,(that is, over-the-counter—OTC) this is known as a NASDAQ Intermarket trade. These transactions were known as “third market” transactions until just a few years ago.

In Progress : Question No. 39 of 602
39)
An investor sells short 1,000 shares of FBN stock at 1.75. Under the rules what is the investor’s required deposit for this transaction?

(a) $2,000
(b) $2,500
(c) $3,750
(d) $1,750
Next
Explanation:

Under the so-called “cheap stock” rule, the investor who sells short stock that is priced $0 – $2.50 per share, the requirement is $2.50 per share. Selling short is a strategy that investors use when they expect the market to drop. There isn’t much room for a stock priced at $1.75 to drop. Also, the lower-priced stocks tend to exhibit great volatility, which translates to even greater risk to the investor.

40)
Under the provisions of Rule 144A, which of the following are true statements?

(a) Affiliates have no holding period for restricted stocks.
(b) QIBs may trade restricted securities without regard to quantity or holding periods.
(c) Non-affiliated persons may trade letter stocks freely after 1 year.
(d) Restricted stocks may only be traded after a two-year holding period.
Next
Explanation:

QIB stands for Qualified Institutional Buyer. Entities that meet the requirements for this type buyer may trade restricted stocks freely with no holding period or quantity restrictions. Rule 411A stipulates the requirements for these institutions.
In Progress : Question No. 41 of 602
41)
A trader for a broker-dealer in New York makes a trade over the telephone for 5,000 shares of Exxon stock with a broker-dealer inDallas. Which of the following is/are true regarding this transaction?
I. It is considered to be a third market transaction
II. It will be reported on Tape “B”
III. It is considered an exchange transaction since Exxon is a listed stock
IV. It will be reported on Tape “A”

(a) II, III
(b) I
(c) I, II
(d) I, IV
Next
Explanation:
Transactions in exchange-listed stocks that take place away from the exchange floor are technically called NASDAQ Intermarket trades. The earlier term, Third Market is, however, still in use and may appear on your exam. All trades in NYSE-listed stocks are reported on Tape “A,” regardless of where they occur.
47)
Under MSRB Rule G-34, which of the following is responsible for applying for the CUSIP numbers on a new issue of municipal bonds?

(a) The issuing authority
(b) The syndicate manager
(c) The trustee bank
(d) The Bond Counsel
Next
Explanation:
Rule G-34 requires that the syndicate manager make application for the issue’s CUSIP numbers.
49)
When control stock is sold for a profit before a six-month holding period has elapsed, this is known as which of the following?

(a) A short swing profit
(b) A short-term gain
(c) An unregistered sale of restricted stock
(d) A statutory underwriter sale
Next
Explanation:
Control stock is stock held by “control persons” who are corporate insiders. These persons must hold their company’s stock for six months to avoid the “short-swing” profit rule. If a control person sells the stock at a profit after holding it for less than six months, the profit must be “disgorged,” or go back to the corporation.

In Progress : Question No. 50 of 602
50) On which exchange(s) do foreign currency options contracts trade?
(a) CBOE
(b) AMEX
(c) NYSE
(d) PHLX
Next
Explanation:
Although years ago, foreign currency options were traded on other options exchanges, they are now traded exclusively on the Philadelphia Exchange, the PHLX.
In Progress : Question No. 53 of 602
53)
When a broker-dealer receives mutilated certificates from another member firm that are not accompanied by coupons, what is the process by which the transaction may be corrected if the delivery was initially accepted?

(a) Certification
(b) Rejuvenation
(c) Reclamation
(d) Rejection
Next
Explanation:
If a broker-dealer that has initially accepted delivery of certificates that are later found not to be good delivery, the process by which corrections may be made is reclamation. If the firm finds, upon delivery, that the certificates are not in good form, the broker-dealer will reject delivery.
55) A broker-dealer must keep which TWO of the following for three years:

I. All receipts and disbursements of cash and all other debits and credits.
II. All guarantees of accounts, powers of attorney and any other evidence of the granting of discretionary authority.
III. All repurchase and reverse repurchase agreements.
IV. All purchases, sales, receipts and deliveries of securities and commodities for each account, and all other debits and credits to each account.
(a) II and III
(b) I and III
(c) I and IV
(d) II and IV
Next
Explanation:
Transaction records such as receipts and disbursements of cash, and all purchases, sales, receipts and deliveries of securities, have six-year holding requirements.
In Progress : Question No. 56 of 602
56) Which of the following are characteristics of DVP settlement?

I. Normally institutional
II. Trade date plus 3 business days
III. Typically used between municipal dealers with customers
(a) II only
(b) I, III
(c) I only
(d) II, III
Next
Explanation:

DVP (Delivery Versus Payment) settlement is normally used in institutional trades. The actual payment to the delivering institution is usually cash.
In Progress : Question No. 59 of 602
59)
If the manager of a municipal underwriting syndicate determines that it is necessary to change the normal priority of order fills, under the rules of the MSRB it may do so:
I. If it is deemed to be in the best interests of the syndicate.
II. At least two business days before sales begin, all syndicate members are notified of the change.
III. The selling group will be paid concessions in excess of the total takedown.
IV. If all of the other members of the syndicate are notified no later than 5 business days before sales begin.

(a) I, III
(b) II, III
(c) I, II
(d) I, III, IV
Next
Explanation:

Under MSRB Rules, the normal order priority may be altered by the syndicate manager if it is in the best interests of the syndicate. The manager must, however, inform the members of the syndicate of the actual order fill priority no later than two business daysbefore sales begin.

60) Which of the following would not be included in a “when, as, and if issued confirmation”?

I. Extended principal
II. Accrued interest
III. Settlement date
IV. Total dollar amount

(a) III, IV
(b) I, IV
(c) II, III, IV
(d) I, II, III, IV
Next
Explanation:
A “when, as, and if issued confirmation” – or WI confirmation – is confirming a trade in a new issue that has been authorized but not yet issued. Because the settlement date is unknown, accrued interest cannot be calculated. If the accrued interest is unknown, the total dollar amount cannot be calculated.

62) Which of the following terms are associated with the purchase/redemption of open-end investment company shares?
(a) At the money
(b) Current business day close
(c) Forward pricing
(d) 90-day backdating
Next
Explanation:
Forward pricing is the SEC Rule that requires all transactions in open-end investment company shares to be based on the next calculation of the fund’s Net Asset Value (NAV).
In Progress : Question No. 69 of 602
69) Which of the following TWO statements are true of custodial accounts?

I. They must always be cash accounts.
II. Option trading and short selling are permitted in custodial accounts, provided the custodian provides written permission.
III. A registered rep needs written authorization from her supervisor and the client before exercising discretionary authority in a custodial account.
IV. A registered rep needs power of attorney before exercising discretionary authority in a custodial account.
(a) I and IV
(b) II and IV
(c) II and III
(d) I and III
Next
Explanation:
Custodial accounts must be cash accounts and written authorization is sufficient in order to exercise discretionary authority in a custodial account.

In Progress : Question No. 74 of 602
74) Which statement about the New York Stock Exchange is FALSE?
(a) It is a self-regulating organization.
(b) It requires written authorization from a customer before an employee of a member can exercise discretionary power over an account.
(c) It permits coded accounts.
(d) Employees of members cannot open accounts at competing broker-dealers.

In Progress : Question No. 81 of 602
81) John is an RR in Mississippi whose client, Sally, has placed an order to buy stock. John’s sales assistant takes the call and leaves John a note to call Sally when the order is executed. John notices that the area code and number are different from what he has on file and when he calls Sally he discovers that she has moved to Texas. Which of the following actions must John take?
(a) Mail the confirmation to Sally’s old Mississippi address so that it will be forwarded to her in Texas.
(b) If his broker-dealer is registered in Texas, John’s registration is also effective in that state.
(c) Cancel the order.
(d) Begin action to become registered in Texas under the Uniform Securities Act.
Next
Explanation:
Under the Uniform Securities Act (the “Blue Sky Laws”) a RR must be registered in each state in which he/she conducts business unless there is an exemption. In this case there are no applicable exemptions.
84)
A margin account has a long market value of $12,000, a debit balance of -$6,000 and a credit balance of $6,000. Which is/are TRUE about the this margin account if the share price of the stock purchased should appreciate 20% and the Reg T margin requirement is 50%?

I. Long market value is $14,400.
II. Debit balance is $7,200.
III. Reg T requirement is $6,000.
IV. Excess equity is $1,200.

(a) I and IV
(b) I, II and IV
(c) I only
(d) I, II, III and IV
Next
Explanation:

I is correct since the stock’s currently valued at $12,000 are increasing 20% or $2,400.
II is incorrect because the debit balance remains at $6,000.
III is incorrect since the Reg T requirement is $7,200.
IV is correct since: ($14,400/2) – $6,000 = $1,200.
In Progress : Question No. 89 of 602
89) Which of the following signatures are required on a client’s new account form?

I. The client’s
II. The client’s spouse
III. The registered representative
IV. The principal
(a) III, IV
(b) I, II
(c) I, III, IV
(d) I, II, III, IV

93) Which of the following is a reason for delays in transferring accounts?

I. The transfer form is incomplete.
II. The transfer includes a margin account.
III. The account owner is changed.
IV. The transfer involves a retirement account.
(a) I only
(b) I, II, III and IV
(c) I and II only
(d) I, II and III only
Next
Explanation:
NYSE Rule 412 requires that transfers be completed within three business days but these problems often arise.
In Progress : Question No. 99 of 602
99) A hypothecation agreement:
(a) pledges that securities in street name may be used as collateral for the broker-dealer’s obligations.
(b) applies to cash and margin accounts.
(c) cannot be signed until the account is activated.
(d) pledges that securities in a customer account may be used as collateral for funds loaned by the broker-dealer.
Next
Explanation:
The pledge that securities in street name may be used as collateral for the broker-dealer’s obligations is a description of rehypothecation.
In Progress : Question No. 101 of 602
101) All of the following are actions the broker-dealer should take upon hearing of the death of an account holder EXCEPT:
(a) Cancel all open orders
(b) Convert the account to an estate account
(c) Keep the account open until all until officially notified that all tax claims have been resolved.
(d) Ensure that no further orders are placed
Next
Explanation:
No changes should be made to the account’s type until the broker-dealer has received a death certificate.

104)
An RR has an elderly, single man who has been his client in an individual account for years. The RR learns that the client has passed away. Which of the following actions does the RR take?

I. Cancel all open orders
II. Mark the account “deceased”
III. Execute open orders at the best price.
IV. Await instructions from the executor of the estate.

(a) I, III, IV
(b) II, IV
(c) III, IV
(d) I, II, IV
Next
Explanation:
When the client in an individual account dies, only three actions are permitted: the cancelation of all open orders, marking the account “deceased” and awaiting instructions from the executor of the estate. The question did not specify any other instructions.
105) Which of the following is/are TRUE about custodial accounts opened under the Uniform Gift to Minors Act?
I. They establish a simple way for a minor to own securities and other types of property.
II. Terms are established by a trust document.
III. The donor irrevocably gifts the assets to the trust.
IV. There is no special tax treatment.

(a) I, II, III and IV
(b) II, III and IV
(c) IV only
(d) III and IV
Next
Explanation:
I is incorrect because other types of property are gifted under the UTMA.
II is incorrect because terms are set by statute, not by the trust document

106)
Which TWO of the following are the MOST important arrangements for ownership of a customer account:

I. Individual
II. Joint
III. Partnership
IV. Specific purpose

(a) II and IV
(b) I and II
(c) I and III
(d) III and IV
Next
107) Under the Telephone Consumer Act of 1991, a telemarketer must provide which of the following to a called prospect?

I. His/her name
II. Source from which the calling company obtained the called party’s phone number and name
III. Purpose of call
IV. Phone number and address of company making the solicitation call
(a) II, III, IV
(b) I, II, III, IV
(c) I, III
(d) I, III, IV
Next
Explanation:
All the choices except II are specific requirements of the Act. In addition, the law places time-of-day restrictions on cold calling. Telemarketers must also be trained in using and updating the company’s “do not call” list.
In Progress : Question No. 109 of 602
109) Which of the following statements is FALSE about custodial accounts opened under the Uniform Gift to Minors Act?
(a) The child has no right to gift the money back.
(b) The child must pay income tax on the account.
(c) The custodian may not draw cash from the fund.
(d) The money is the child’s property.
Next
Explanation:
The custodian can withdraw cash to spend on items of benefit to the child, provided they go beyond basic parental obligations.
Quit
112) What is the principal advantage for a client to designate an heir under the TOD Security Registration Act?
(a) The assets held in the account will be exempt from estate taxes
(b) Assets in the account are protected against all claims and pass to the heir unencumbered
(c) Assets in the account pass through to the designated beneficiary without going through the probate process
(d) It works much like a JTWROS account
Next
Explanation:
The Uniform Transfer on Death Security Act has been adopted by most states. It allows an investor to designate a beneficiary who will automatically receive the assets in the account at the death of the client without going through probate. The TOD account does not protect the assets against all claims and it does not shelter the account from estate taxes. The beneficiary has no ownership in the account until the death of the decedent.
In Progress : Question No. 113 of 602
113) Which of the following are FALSE regarding maintenance margin?

I. It is 50% of the current market value of most long securities in the account
II. It is $2.50 per share or 100% of current market value, whichever is greater, of each short stock in the account selling at less than $5.00 per share
III. It is $5.00 per share or 30% of the current market value, whichever is greater, of each stock short in the account selling at or above $5.00 per share.
IV. It is 5% of the principal amount or 30% of the account value, whichever is greater, of each short bond in the account.
(a) I, II and III only
(b) I and II only
(c) I only
(d) I, II, III and IV
Next
Explanation:
Most long stocks require a maintenance margin of 25% of the total market value of the securities in the margin account. Note that this level is a minimum and many brokerages have higher maintenance requirements of 30-40%.
114)
If an airline is using equipment trust certificates as a means of financing the purchase of new airplanes, the title to the planes is held by:

(a) the SEC.
(b) the trustee.
(c) the bondholders.
(d) the airline.
Next
Explanation:
In an equipment trust certificate, a form of bond, the collateral is the equipment itself. Title to the equipment is held by the trustee (required under the Trust Indenture Act of 1939) until the debt is paid off, much like a consumer automobile loan.

In Progress : Question No. 116 of 602
116)
Which of the following would be considered the most complete description of an options contract?

(a) Delta
(b) Class
(c) Series
(d) Strike
Next
Explanation:

A series of options contracts have the same issuer, expiration, strike price, and type, for example, all ABC Nov 60 calls, are a series. This is a complete description of the contract since the position (buying/selling), the quantity, and the premium are not parts of the contract.
In Progress : Question No. 121 of 602
121)
Which of the following would be traded flat?
I. Defaulted bonds
II. Zero coupon bonds
III. Original issue discount municipal bonds
IV. Income bonds

(a) I, II, IV
(b) I, II, III, IV
(c) II, III
(d) I, III
Next
Explanation:

The term “trading flat” means that the bonds are being traded without accrued interest. Of course, bonds that are in default—not paying interest—would trade flat. Zero coupon bonds pay no interest for an accrual to be paid. Income bonds only pay interest if the company has sufficient earnings. Since there is no assurance that the company will have any earnings, these too, trade flat.
Progress : Question No. 122 of 602
122)
If an economist’s basic philosophy is considered “supply side,” which of the following would best reflect his/her economic theories?

(a) Active government intervention using fiscal tools such as taxing and spending policies.
(b) The government has no reason to interfere in the markets, they are self-regulating entities.
(c) Control of the money supply means control of the economy.
(d) Lower taxes and less government intervention leads to a healthy, growing economy.
Next
Explanation:

Supply Side economics are associated with Arthur Laffer, the author of the mathematical model called the Laffer Curve. This theory proposed that cuts in taxes will allow the economy to grow.

Keynesian theory believes that active government intervention using fiscal tools such as taxing and spending policies is best. Monetaristtheory states that control of the money supply means control of the economy. Those who believe the government has no reason to interfere in the markets as they are self-regulating entities, follow the theories of the Austrian School.

In Progress : Question No. 123 of 602
123) In the separate account of a variable annuity, which of the following characteristics apply to annuity units, but not accumulation units?

I. Fixed number of units
II. Actuarial factors have been applied
III. Variable number of units
IV. Loss of control by investor
(a) I, II
(b) III, IV
(c) I, II, IV
(d) II, III, IV
Next
Explanation:
When the holder of a variable annuity contract annuitizes the contract, he/she is asking that the insurance company pay him/her for life. The company applies the actuarial factors for life expectancy and payout option to the value of the account and computes a fixednumber of units to be cashed in each month. At the point of annuitization, the annuitant relinquishes all control of the account.

In Progress : Question No. 124 of 602
124)
Which of the following would be considered the most useful tool for a RR analyzing a newly issued full faith and credit bond?
(a) Debt service coverage ratio
(b) Municipal debt statement
(c) Legal opinion
(d) Feasibility study
Next
Explanation:

A “full faith and credit” bond is a municipal general obligation bond. It is backed by the taxing power of the issuer and approved by a bond referendum. Analysis of the bond would necessarily include a study of the issuer’s debt profile as shown by the municipal debt statement to form a judgment on the issue. Choices “Feasibility study,” and “Debt service coverage ratio,” are tools used in the analysis of municipal revenue bonds. The legal opinion, while an important document—particularly for a source of information on the tax ramifications of the issue—is not a tool used in analysis.
In Progress : Question No. 126 of 602
126)
The NYSE Nine-Bond Rule requires that orders for nine bonds or less be sent to the floor of the exchange for one hour to seek a market. Exceptions to this rule include which of the following?
I. A transaction from a customer who directs that the order go into the OTC market
II. An order for ten or more bonds
III. Government or municipal bonds
IV. A better price for the customer exists in the OTC market

(a) I, II, IV
(b) I, II, III, IV
(c) II, III, IV
(d) II, IV
Next
Explanation:
The Nine-Bond Rule is as stated in the question and is designed to get customers a better price if it is available. The exceptions are all four of the above choices. Remember, any transaction that doesn’t happen on an exchange floor is considered OTC. Government and municipal bond trading is all OTC.
In Progress : Question No. 127 of 602
127)
The Securities Exchange Act of 1934 is the Federal Law that:
I. Required the registration of new securities offerings.
II. Created the SEC.
III. Identified insiders and prohibited trading on material non-public information.
IV. Classified Investment Companies.

(a) II, IV
(b) I, III
(c) II, III
(d) I, II, III, IV
Next
Explanation:

This Act is the companion law for the Securities Act of 1933. The Act of ’34 is sometimes called the “People Act,” because it required that the people involved in the securities industry be registered with the SEC. The Act was directed to secondary market activities, and was established for the protection of public investors. Among other things, it prohibited insider trading, and market manipulation.

In Progress : Question No. 127 of 602
127)
A client opens the following options positions when MML stock is trading at 49:

Long 1 MML Dec 50 call@4
Short 1 MML Dec 60 call @1

If the investor who established the strategy did not close the positions, what would happen at expiration if the market had closed on the final trading day at 51?

(a) The exchange would close both contracts at intrinsic value.
(b) The 60 call would not be exercised, but the 50 call would.
(c) Both contracts would expire worthless.
(d) Both contracts would be exercised
Next
Explanation:

When a customer has not given instructions to close the contract(s) the OCC (Options Clearing Corporation) will automatically exercise contracts that are .75 or more in-the-money for a customer. The automatic exercise for a member firm is .25 in-the-money.

In Progress : Question No. 127 of 602
127) A company has applied to the SEC for registration of its stock under the provisions of Regulation “A.” Which of these is true regarding the company’s filing?

I. The company will file a registration statement with the SEC
II. The document provided to customers is known as an “Offering Circular.”
III. The company will file an offering statement with the SEC
IV. The company is limited by regulation to issuing no more than $5 million in securities in a year.
(a) II, III
(b) I, II, IV
(c) I, III, IV
(d) II, III, IV
Next
Explanation:
Regulation “A” provides for a simplified form of registration for small offerings. The company will file an offering statement with the regional office of the SEC. The offering statement contains notification, an offering circular, and exhibits.

In Progress : Question No. 127 of 602
127) Which of the following statements are true regarding SIMPLE plans

I. A company may offer these as an alternative to its 401(k) plan.
II. Only companies with 100 or fewer employees may establish such a plan.
III. The SIMPLE plan is a salary reduction plan for employees.
IV. Business owners are allowed make matching contributions to the plan.
(a) III, IV
(b) I, IV
(c) II, III, IV
(d) I, II, III, IV
Next
Explanation:
Savings Incentive Match Plan for Employees (SIMPLE) allows for no other qualified plan within the company. These plans allow salary reduction contributions by employees and matching contributions by the employer. These plans, however, are only available to companies that have 100 or fewer employees.
Quit
The correct answer is b. Your answer was correct!
In Progress : Question No. 127 of 602
127) Under MSRB rules, which of the following municipal bonds would be quoted on a yield-to-call basis on the customer’s confirmation?
(a) City of Bonder 6s of 2014 trading @ 5.20
(b) Morgan County 5s of 2009 trading @ 6.15
(c) State of Illinois 4.50s of 2021 trading @ 4.55
(d) City of Nokomo 5.10s of 2020 trading @5.20
Next
Explanation:

MSRB rules require that the customer confirmation show the lowest potential yield. In a bond trading at apremium, this would, of course, be the yield-to-call. Only the city of Bonder 6s of 2014 trading @ 5.20 bond is trading at a premium, the others are trading at a discount.

127)
Long 1 ABC Aug 60 call
Short 1 ABC Mar 60 call

If the investor is to profit from the strategy above, the spread in premiums would have to do which of the following?

(a) Widen
(b) Narrow
(c) Remain constant
(d) Invert
Next
Explanation:

The investor has created a debit or bull call spread. This is true because the long contract (the August call) has a greater time value than the short contract (the February call) and premium is equal to the intrinsic value plus the time value. These contracts will always have the same intrinsic value, because they have the same strike price. Any time an investor creates a debit spread, he/she wants to be able to exercise the long contract and wants the spread in premiums to widen.
127)
A customer sells 10 Treasury bonds when the bonds are quoted at 96.15 – 96.18. What will the total proceeds to the investor be?

(a) $961.80
(b) $9,646.90
(c) $9,615.00
(d) $961.50
Next
Explanation:

Remember that U.S. Treasury Notes and Treasury Bonds are quoted in 32nds (or $.3125 of par $1,000). Remember that the customer sells at the BID—which this case is 96.15, or 96 and 15/32% of $1,000 par. Each bond will be sold at $960 + 4.69 = $964.69 × 10 bonds = $9,646.90.

127) Calculate an investment’s expected return [E(Ri)] based on the following data:

Risk-free rate of return (RF): 3%
Market rate of return (RM): 15%
Variance of RM (σ2M): 7
Covariance of the market
and the investment [Cov(RM,Ri)]: 11
(a) 21.84%
(b) 12.04%
(c) 18.22%
(d) 10.64%
Next
Explanation:
Step 1: Calculate the stock’s beta: 11 ÷ 7 = 1.57

Step 2: Incorporate beta into the capital asset pricing model:

ra = rf + ßa(rm -rf)
= 0.03 + 1.57(0.15 – 0.03)
= 0.2184 or 21.84%
127)
Which of the following are tools that are employed by the Federal Reserve in its efforts to control the money supply?

I. Moral suasion
II. Changing the discount rate
III. Changing the reserve requirement
IV. Changing the prime interest rate

(a) I, III
(b) I, II, III
(c) I, II, III, IV
(d) II, III, IV
Next
Explanation:
The tools of the Federal Reserve include all the above except changing the prime interest rate. The prime interest rate is set by banks. To be sure, the practices and policies of the Fed affect the prime, but the Fed does not set that rate. The Fed also employs the operations of the FOMC, and Regulation “T” in its efforts.
In Progress : Question No. 127 of 602
127)
Which of the following statements regarding the cost basis in a 401(k) plan is true?

(a) There is no cost basis in a 401(k).
(b) The cost basis consists of the total of the account’s value minus the tax-deferred growth.
(c) The cost basis is the amount contributed by the employee.
(d) The cost basis consists of the employer’s matching contributions.
Next
Explanation:
Any contributions made by an individual into a 401(k) plan are frompre-tax money. If the employer chooses to contribute to the plan, those contributions are deductible. Cost basis is the amount of money on which taxes have been paid. Since none of the funds in a 401(k) has been taxed, there is no cost basis; and upon withdrawal everything is taxed at the ordinary income rate.
Quit
In Progress : Question No. 127 of 602
127) Which of the following persons would be eligible to participate in a 403(b) plan?

I. The pastor of a church
II. A first-grade teacher in a public school
III. An electronics teacher for the U.S. Navy
IV. A professor at a state university.
(a) I, II, IV
(b) I, II, III
(c) I, II, III, IV
(d) II, III, IV
Next
Explanation:

403(b) plans are for employees of public school systems and employees of non-profit institutions such as churches (identified as 501 (c) (3) organizations). The U.S. Navy teacher is not a part of a public school system.
127) If a mutual funds investor were to be sold a dividend, what would be the effect?

I. The investor will see a decline in the price of his/her shares on the ex-dividend date.
II. The investor will realize a short-term gain.
III. The investor will experience adverse tax consequences because of the transaction.
IV. The fund will reimburse the investor’s losses, if any within 7 days.
(a) I, III, IV
(b) I, II, III, IV
(c) II, IV
(d) I, III
Next
Explanation:
The practice of “selling dividends” to a customer is prohibited. This occurs when brokers try to convince a customer that purchasing a particular investment such as stocks or mutual funds will be profitable because of an upcoming dividend. The dividend to be paid—in the near future—is a part of the fund’s NAV. Since the price of the shares drop on the ex-dividend date by the amount of the dividend there is no realized value. In fact, the customer is receiving back his/her own tax-paid investment and owes taxes on it yet again due to their receipt of the dividend.
Quit
In Progress : Question No. 127 of 602
127) When an individual who has made contributions to a Keogh plan for a number of years begins withdrawals after age 65, what are the tax implications?
(a) All withdrawals are taxed as ordinary income.
(b) All withdrawals are taxed as long-term gains
(c) Withdrawals prior to age 70 ½ are taxed as ordinary income and are subject to a 50% penalty.
(d) The portion of the account that represents growth is taxed as ordinary income, the cost basis is not taxed.
Next
Explanation:
There is no cost basis in a Keogh (HR-10) plan. All contributions are pre-tax, so all withdrawals are taxed as ordinary income. The early withdrawal and late withdrawal rules for Keoghs are similar to IRAs.
Quit
127)
In a municipal debt statement, what is added to the reporting entity’s net direct debt to produce the net overall debt?

(a) Sinking fund debt service
(b) Self-supporting debt
(c) Coterminous debt
(d) Ad valorem debt
Next
Explanation:

The major line items of a municipal debt statement are as follows:

Direct Debt
– Self-supporting Debt
Net Direct Debt
+Coterminous Debt
Net Overall Debt

Coterminous debt is synonymous with overlapping debt in the analysis of municipal debt. In the computation, the municipality takes into account the debt shared with other entities that share the same geographical area (counties, school districts, park districts, etc.) when calculating the net overall debt. Such debt must be taken into account when analyzing the burden of debt on taxable property of the municipality and its citizenry.
n Progress : Question No. 127 of 602
127) The term “breakpoint sales” in mutual funds refers to which of the following?

I. An unethical practice
II. Working with a client to qualify for a reduced sales charge through a letter of intent
III. Advising a client to invest an amount just below the point at which a lowered sales charge would be triggered
IV. The sale of mutual fund shares to a client in amount that would allow the client to receive a lower sales charge because of the rights of accumulation
(a) II, IV
(b) I, III
(c) II
(d) I, IV
Next
Explanation:
Breakpoint sales are sales to a client that deliberately (on the part of the RR) fall short of triggering a breakpoint, or lowered sales charge. Instead, the RR should work with the client to qualify for the reduced sales charge through LOI or other means.
Quit
In Progress : Question No. 127 of 602
127) When an issuer has a serial bond outstanding, and makes a partial callof bonds from the issue, the bonds for the call will normally be chosen by using which of the following methods:
(a) Inverse numerical sequence
(b) FIFO
(c) Random lottery
(d) Numerical sequence
Next
Explanation:
Inverse numerical sequence means that the bonds with the longest maturity dates will be called first, working backward to the shorter maturities. The longest bonds are called first in order to reduce the number of interest payments that the issuer would have to pay, thus saving money.
Quit
127)
The Interbank Market is most accurately described by which of the following?
I. Unregulated
II. Decentralized
III. Negotiated
IV. Trading is usually in $1 million increments

(a) I, IV
(b) I, II, III, IV
(c) II, III
(d) I, II, III
Next
Explanation:

The Interbank Market is the marketplace in which banks trade currencies. It is described as an unregulated and decentralized marketplace in which the exchange rates of currencies is set by a negotiated process. The normal increment is $1 million.
127)
An investor made his initial margin transaction by purchasing 1,000 shares MML @ 30. Within a month, the stock had increased in value to 38. At this point, if the investor wanted to buy 500 shares of WKW@20, what would be the minimum margin call for the transaction?

(a) $5,000
(b) $2,000
(c) $3,000
(d) $1,000
Next
Explanation:

When the investor purchased $30,000 in MML stock, she was required to deposit $15,000 and the debit balance—the loan from the broker-dealer—was $15,000. When the stock increased in value to $38,000, the equity went to $23,000 ($38,000 – $15,000 = $23,000). With Reg. “T” at 50% equity should be at $19,000 and the investor has $23,000. There is $4,000 in excess equity which creates SMA of $4,000. The Reg. “T” call on $10,000 is $5,000($10,000 × 50% = $5,000). Subtract the SMA from the Reg. “T” requirement ($5,000 – $4,000 = $1,000 deposit requirement.
In Progress : Question No. 127 of 602
127)
The Bond Buyer’s “Thirty-Day Visible Supply” is a statistical tool indicating which of the following?

(a) The ratio of general obligation bonds to revenue bonds scheduled in the next thirty days
(b) The total dollar amount of long-term municipal debt scheduled to come to the market in the next thirty days
(c) The total dollar amount of IDRs sold in the past thirty days
(d) The amount of municipal bonds that were issued and sold in the past thirty days
Next
Explanation:
Choice “B” is a succinct definition of the information provided by the Bond Buyer’s “Thirty-day Visible Supply.” This information, coupled with the Bond Buyer’s “Placement Ratio” is extremely valuable to municipal underwriters in determining the market for municipal securities.

127)
The XYZ Company declared its dividends for the year at the stockholders meeting in January. One of the dividends, for $50 per share is payable July 30th to stockholders of record on Thursday July 6th. What is the ex-dividend date for this payment?

(a) Wednesday July 29th
(b) Tuesday July 4th
(c) Monday July 3rd
(d) Friday June 31st
Next
Explanation:
By NASD Rule, the ex-dividend date for stock is 2 business daysbefore the record date. Since the record date is Thursday, July 6th, count back 2 business days to Monday, July 3rd. July 4th is not a business day.
127)
Which of the following is/are not classifications of investment companies under the Investment Company Act of 1940?
I. UITs
II. Hedge funds
III. Face Amount Certificate Companies
IV. Open-end funds

(a) I, III
(b) II
(c) I, II, III
(d) II, IV
Next
Explanation:
The key to this question is the word “classifications.” The Act has three formal classifications; Face Amount Certificate Companies, Unit Investment Trusts, and Management Companies. Open-end funds (mutual funds) are a subcategory of Management Companies. Hedge funds are not a form of Investment Company. If they are registered companies, they are typically registered as limited partnerships.
In Progress : Question No. 127 of 602
127)
Which of the following types of DPPs would be associated with the use of accelerated depreciation as an advantage?

(a) Oil and gas exploration
(b) Raw land
(c) Equipment leasing
(d) Real estate
Next
Explanation:

The only type of limited partnership that uses accelerated depreciation (actually, MACRS-Modified Accelerated Cost Recovery Systems) as an advantage is the equipment leasing partnership. Real estate partnerships may only use “straight-line” depreciation. Raw land may not be depreciated. Oil and gas partnerships use depletion rather than depreciation.

In Progress : Question No. 127 of 602
127)
The city of Limrock has issued an IRB to finance the construction of a new industrial park. The principal lessee in the industrial park will be Zoomer Automotive Industries. What will be the backing of the IRB?

(a) Both property taxes and lease payments
(b) The full faith and credit of the city of Limrock, specifically ad valorem taxes
(c) Lease payments from Zoomer
(d) The moral obligation of the state legislature for the state in which Limrock is located
Next
Explanation:

The term IRB stands for Industrial Revenue Bond (also known as IDR, Industrial Development Revenue bonds). The lease payments of the company occupying the industrial park will be the backing for the bond.
127)
When the Federal Reserve conducts its auctions for Treasury Bills, which bids are filled first?

(a) Competitive bids
(b) Non-competitive tenders
(c) Those determined by priority sequencing.
(d) Bids beyond parity
Next
Explanation:
The auction for T-Bills is split into two parts. The competitive bidders submit their bids to the Treasury at the best price they can get in what is known as a Dutch Auction. At the end of the day, non-competitive bids are filled at the average competitive price paid during that day’s auction. So, the competitive bids are filled first.
127)
If a registered representative is explaining the advantages and disadvantages of a Government Assisted Housing partnership to his clients, which of the following would she not describe as an advantage?

(a) High degree of tax shelter in the early years
(b) Low risk of audit
(c) Potential for appreciation
(d) Government-provided rental subsidies
Next
Explanation:
The question focuses on what is not an advantage. Government Assisted Housing partnerships have a very low chance of appreciation of the property compared to other real estate partnerships. The other items are considered advantages.
127)
In comparing a publicly-offered oil and gas drilling DPP with a private offering, all of the following are typically characteristics of a public program EXECPT:

(a) A public partnership is registered with the state or SEC
(b) A public partnership has higher expenses
(c) A public partnership typically carries lower risk
(d) A public partnership typically offers greater diversification
Next
Explanation:
Choices “B,” “C,” and “D” are typically cited as characteristics of public partnerships that may make them more suitable for many investors. The fact that a public partnership is registered does not, of course, mean any guarantees, but indicates that the regulatory authorities have at least examined the offering.

127)
A corporation, which does virtually all of its business in the state where it has its home offices, wishes to go with as little cost as possible and raise $10 million. Which of the following would be the most likely avenue for registration?

(a) Rule 147
(b) Standby
(c) Regulation “A”
(d) Registration by Notification
Next
Explanation:
Rule 147 is a rule that allows for intrastate offerings to be exempt from the requirements of full registration under the Act of ’33. A company located in the state must make the offering. 100% of the initial purchasers must be residents of the state, and there are strict rules on resale to non-residents. Registration by notification requires that the company has already been registered with the SEC. Regulation “A” is a limited registration with the SEC limited to a maximum of $5 million.
127)
Which of the following are used in the compilation of The Bond Buyer’s Revdex?
I. 25 revenue bonds
II. 20-year maturities
III. 20 revenue bonds
IV. 30-year maturities

(a) I
(b) I, II
(c) I, IV
(d) II, III
Next
Explanation:
The Bond Buyer’s indices include the Revenue Bond Index, familiarly called the “Revdex.” This index includes 25 revenue bonds with 30-year maturities.
In Progress : Question No. 127 of 602
127)
Which of the following is/are associated with foreign equity instruments?
I. Banker’s Acceptances
II. Eurodollar bonds
III. ADRs
IV. LIBOR

(a) III, IV
(b) III
(c) I, II
(d) IV
Next
Explanation:

American Depositary Receipts are issued to facilitate the trading by Americans in foreign-issued stocks. Banker’s Acceptances are debt instruments issued by banks and are associated with payment in the import-export business. Eurodollar bonds are foreign-issued bonds payable in U.S. dollars. LIBOR stands for London Interbank Offered Rate.
In Progress : Question No. 127 of 602
127) The public offering price of a mutual fund is based on which of the following?
(a) NAV + Load
(b) NAV + Concession + Sales Charge
(c) NAV
(d) NAV – Concession + Sales Charge
Next
Explanation:
In a mutual fund, everything begins with the calculation of the fund’s Net Asset Value (NAV). The sales charge, or load—if there is a sales charge—is added by a mathematical formula to compute the Public Offering Price (POP), or Asking price. Note that NAV + Load is incorrect as the question asks what the POP is based on, not how it is calculated. The POP is always based on the NAV, and the fund itself adds on any shareholder fees, such as loads, themselves. In your upcoming exam be sure to read the question carefully, seemingly simple questions are often worded differently to trick you!
127)
A customer’s order is to buy 500 shares XYZ @50 FOK. The floor broker for SMS Securities presents the order to the trading crowd at the specialist’s post. In the crowd, one floor broker offers to buy 100 at 50, and another offers to buy 300 at 50. What is the result?

(a) SMS buys 300 at 50, 100 at 50 and cancels the remainder of the order
(b) SMS buys 400 at 50 and places the remainder on the specialist’s book GTC at 50
(c) SMS places the entire order with the specialist as buy 500@ 50 GTC
(d) SMS cancels the order
Next
Explanation:

The order is Fill Or Kill (FOK). In an order of this type, the customer accepts no partial fills. Either the order is filled in its entirety at the price specified, or the order is killed

127) Which of the following are true regarding the letter of intent (LOI) by which a customer may qualify for a breakpoint on a mutual fund purchase?

I. The LOI covers purchases for a 13-month period.
II. It provides for a 90-day backdating privilege.
III. The breakpoint applies on all purchases during the period of the LOI.
IV. Only “new money” deposits are counted toward completing the LOI.
(a) I, II, IV
(b) II, III, IV
(c) I, II, III
(d) I, II, III, IV
Next
Explanation:
A LOI provides a mutual fund investor with a way of immediately qualifying for a lower sales charge, based on his/her intention to make the deposits over the next 13-months that reach the breakpoint. If the investor fails to make the deposits, and only “new money” counts, the fund will bill the investor for the difference between the breakpoint sales charge and the amount that would have been charged without the breakpoint.
127)
Which of the following indices of the Bond Buyer is price-based and based on quotations obtained from six municipal securities broker’s brokers?

(a) The Revdex
(b) The Bond Buyer Municipal Bond Index
(c) The 20 bond index
(d) The 11 bond index
Next
Explanation:

The Bond Buyer Municipal Bond Index is the only one of the indices that is price-based; all the others are based on yield. This index, sometimes called the 40 Bond Index represents an average of prices, adjusted to a 6.00% yield basis of 40 recently-issued municipal bonds. The information from which the index is compiled comes from 6 broker’s brokers selected by the Bond Buyer.
127) Which is NOT an element of M2 money supply?
(a) Dollar-denominated deposits outside the U.S.
(b) Money in savings accounts
(c) Cash in circulation
(d) Money in checking accounts
Next
Explanation:
Dollar-denominated deposits outside the U.S. is an element of the M3 money supply.
127) During a disastrous trading day on the NYSE, the DJIA is down by 21% (a Level 2 decline) at 2:30 in the afternoon. NYSE Rule 80B mandates which of the following trading halts?
(a) Close the market for the day
(b) Halt trading for 1 hour
(c) Halt trading for ½ hour
(d) Halt trading for 2 hours
Next
Explanation:
A Level 2 decline (20% -30%) after 2pm requires closing the market for the rest of the day. There are several levels of halts, depending on the severity of market decline. Rule 80B uses the DJIA (Dow Jones Industrial Average) as the standard for triggering trading halts.
All of the following would typically be considered advantages of a real estate limited partnership in an existing property except:

(a) Possible appreciation
(b) Possible immediate cash flow
(c) MACRS depreciation
(d) History of operating expenses
Next
Explanation:

Only straight-line depreciation is permitted in real estate limited partnerships. All of the other characteristics are considered advantages or potential advantages to investors.

Corporate bonds with which of the following ratings are considered to be investment grade?
I. A1
II. BB
III. AA
IV. Baa

(a) III
(b) I, II, III, IV
(c) I, III, IV
(d) I, III
Next
Explanation:

Moody’s and Standard & Poor’s rating systems differ in their “look.” S&P investment grade bonds are AAA, AA, A, BBB. Ratings of BB and below are “junk bonds” (or high-yield bonds). Intermediate grades are indicated by a + or a -. Moody’s investment grade ratings are: Aaa, Aa, A, Baa. Ba, B, etc are the junk bond ratings. Numerical modifiers such as Aa1 indicate intermediate grades.

Quit
The correct answer is c. Your answer was incorrect!
127)
By what time, at the latest, must a must a member firm send a notice of exercise for an options contract to the OCC?

(a) No later than 4:02 p.m. Eastern time on the last business day before expiration.
(b) No later than 3:02 p.m. Central time on the last business day before expiration.
(c) No later than 4:02 p.m. Central time on the last business day before expiration.
(d) No later than 4:30 p.m. Central time on the last business day before expiration.
Next
Explanation:
The CBOE Rule specifies that the notice of exercise must be sent to the OCC no later than 4:30 p.m. central time on the final trading day before expiration. This is actually 1.5 hours after the close of the markets. Trading on the floor of the exchange must halt at 4:02 p.m. (Chicago) Central on the final day before expiration.
Quit
127) Which of the following markets is most closely associated with INSTINET?
(a) The Fourth market
(b) The NASDAQ Intermarket
(c) The NASDAQ
(d) The exchange market
Next
Explanation:
The INSTINET (Institutional Networks Corporation) is an electronic, computer aided system that allows institutions such as mutual funds and pension funds to trade directly with one another, avoiding commissions. This institutional market is called the Fourth Market.
Quit
127)
In the computation of the Beta Coefficient on stocks which of the following is used to represent the overall market?

(a) The Dow-Jones Industrial Average
(b) The S&P 100 index
(c) The Wilshire 5,000
(d) The Russell 2,000 index
Next
Explanation:

A stock’s beta coefficient, or just beta, compares the volatility of the stock the overall market. The S&P 100 index is used to represent the overall market with a beta of 1.
127) Which of the following statements does not represent a disadvantage of owning a REIT?
(a) The tax sheltering effects of depreciation are not passed through to the unit holders.
(b) REITs have limited growth prospects.
(c) Since the underlying asset is still real estate, REITs are still illiquid relative to stocks.
(d) REITs may result in higher taxes to an individual relative to a similar risk investment that pays less income.
Next
Explanation:
Actually, REITs trade on major stock exchanges; therefore, they are just as liquid as the majority of the stocks listed on the exchanges. The reason why REITs have limited growth prospects is because they cannot retain more than 5% of their income. Furthermore, from an investor point of view, because dividends are taxed at a higher rate than capital gains, a REIT will result in higher taxes than a similar risk investment which pays a lower income, and thus consequently, a relatively higher amount of capital gains.
Quit
The correct answer is c. Your answer was correct
127)
The State Park Authority of a particular state wants float a municipal bond issue to develop a new state park. The debt service for the issue is to be paid from user fees. The Authority has made a presentation to the state legislature and has its backing for the issue if fees collected at any time are insufficient. This type of bond is:

(a) A PHA
(b) An IDR
(c) A moral obligation bond
(d) A double-barreled bond
Next
Explanation:

When the state legislature grants the Authority its backing, it appropriates no funds. Therefore, this bond has the authorization but not the appropriation to go forward with the offering. The legislature has only taken a moral obligation of intent to support the issue if it becomes necessary.
In Progress : Question No. 127 of 602
127) What is the most common par value of municipal bearer bonds?
(a) $50,000
(b) $5,000
(c) $1,000
(d) $10,000
Next
Explanation:
Although bearer bonds of any type have not been issued for some years, questions of this nature persist in the exam. Just note this fact.
127)
A registered representative has been reading the Bond Buyer and has noticed that a notice of defeasance has been posted for a bond issue that forms a significant part of a client’s portfolio. What effect does defeasance have on his client’s bonds?

I. The issuer of the bonds has, in effect, defaulted on the issue.
II. The bonds will be called in the near future.
III. The investor’s bonds that have been defeased will be AAA rated.
IV. An investor holding these bonds may put them to the issuer during the defeasance period.

(a) II, III
(b) III, IV
(c) I, II, IV
(d) I, IV
Next
Explanation:

The process of defeasance means that the bonds in question have been pre-refunded, that is, the issuer has raised the money (by issuing new bonds with a lower coupon rate) to call the outstanding bonds. The money is normally invested in U.S. Government securities and placed on deposit with a trustee bank (the escrow trustee). The notice informs investors that the funds to call the bonds are available and that the bonds will be called in the relatively near future. Because there is no doubt about the payment of interest and principal, the defeased bonds go to AAA.
In Progress : Question No. 127 of 602
127)
An investor bought a corporate bond at 105 with a coupon of 6% and settled on May 28th. The bond’s interest payment dates are January and July 15. In addition to the price of the bond, how much will the investor pay the former owner in accrued interest?

(a) $34.65
(b) $18.33
(c) $22.21
(d) $18.57
Next
Explanation:

When computing accrued interest on corporate or municipal bonds, use 30-day months and 360-day years. Subtract the last interest payment date from the settlement date to get the number of days of accrued interest.

Settlement: Month Day
5 28
Last interest pay date: 1 15
4 13

4 months @ 30 days each = 120 days + 13 days = 133 days of accrued interest

$60/year interest payments ÷ 360 days = $.167 × 133 = $22.21

Quit
127)
A company’s stock is being distributed under the provisions of Regulation D. Which of the following statements apply to this offering?
I. It may be sold to an unlimited number of institutional investors
II. It will be known as “legend stock.”
III. The stock may be sold to 35 or fewer non-accredited investors
IV. There is a mandatory holding period of one year for non-accredited investors

(a) II, III, IV
(b) I, III, IV
(c) I, II, III, IV
(d) II, III
Next
Explanation:
Regulation “D” stock is private placement stock, and is an unregistered issue. Institutional investors (often called QIBs, or Qualified Institutional Buyers) may purchase the stock without limitation. Non-accredited investors are limited to thirty-five or fewer. These investors must hold the stock, fully-paid, for one year before resale. There is a legend printed or stamped on the stock identifying the stock as restricted stock.
In Progress : Question No. 127 of 602
127)
Which of the following systems is associated with trading in the NASDAQ markets?

(a) Super DOT
(b) INSTINET
(c) OSS
(d) SuperMontage

127) ABC shares are currently trading at $27.00, and they are expected to pay a dividend of $0.50 in 6 month’s time. The 6- month put option on ABC shares with an exercise price of $25.00 is currently trading at $1.50. If the risk-free rate is 8%, what should be the value of a 6-month call option on ABC shares with an exercise price of $25.00?
(a) $3.96
(b) $2.87
(c) $2.00
(d) $2.82
Next
Explanation:
According to Put-Call Parity:

(Stock Price) + (Put) – (Call) = PV(X) + PV(D)

27.00 + 1.50 – c = [(25)/(1.08)0.5] + [(0.50)/(1.08)0.5]

28.50 – c = 24.06 + 0.48
c = $3.96

n Progress : Question No. 127 of 602
127)
In the process of preparing a legal opinion for a new municipal issue, which of the following would most likely result in the issuance of a qualified opinion by the bond counsel?

(a) A determination by the bond counsel that the syndicate letter specifies a priority for the filling of orders that is outside the generally-accepted norm.
(b) The attorney preparing the legal opinion determines that the bonds in question are probably subject to the AMT.
(c) The bond counsel determines that the quality of title to land on which a proposed facility is to be built is less than clear.
(d) The discovery that the municipal issue may not be totally tax-exempt.
127)
When an individual chooses to annuitize his/her account, which of the following payout options, if all other factors are equal, will pay the least payment per period?

(a) Unit refund
(b) Joint and survivor
(c) Life annuity
(d) Period certain
Next
Explanation:
When the annuitant chooses the and joint and survivor payout option, the insurance company is taking a mortality risk on more than one person, and will be required to continue payments so long as any person named is alive. Because of this, this option will pay the least per payment.
127)
A bond is described in its indenture as a “Refunding Mortgage Bond.” Which of the following statements about the bond are true?
I. The collateral for the bond is real estate
II. The proceeds of the bond are pledged to purchase new real estate
III. The bonds are being issued so that the issuer may pay off its mortgage
IV. The proceeds from the bond issue will be used to retire other debt obligations

(a) I, IV
(b) I, II
(c) II, III
(d) I
Next
Explanation:

The process of refunding is very similar to refinancing a home mortgage. The company is borrowing new money (the refunding bond) at a lower interest rate to pay off old, higher-interest-rate debts. The term “mortgage bond,” of course indicates that the collateral for the bond is real estate.
127) There are several indexes that track the market in various sectors. Which of the following is the broadest index?
(a) NYSE Composite
(b) Wilshire
(c) Russell 2,000
(d) Dow Jones Industrial Average
Next
Explanation:
The broadest index is the Wilshire 5000. More than 7,000 stocks are used in adjusting the index. The Russell 2000 is a well-known index of smaller companies. The Dow Jones Industrial Average is an index of 30 blue chip stocks. The NYSE Composite measures only stocks listed on the NYSE-3000+ used in the index
127)
If a corporation has a large number of outstanding convertible bonds, the company may use which of the following actions to lead bondholders to convert to the common stock of the company?

I. Call the bond at a price below parity
II. Change the conversion price
III. Raise dividends on the common stock above the bond’s interest rate
IV. Lower the coupon rate on the bonds

(a) I, IV
(b) I, III
(c) I, II, III
(d) I, II, III, IV
127)
A RR has a client who has a varied and sophisticated portfolio, which contains, among many other instruments, a large number of Eurodollar bonds issued in foreign countries by various corporate entities. Regarding these bonds, which of the following risks will this client not be exposed to?

(a) Interest rate risk
(b) Loss of purchasing power
(c) Currency exchange rate risk
(d) Reinvestment risk
Next
Explanation:

A Eurodollar bond is issued in a foreign country, but both interest and principal are payable in U.S. dollars. Since the client will be receiving his/her payments in U.S dollars, there is no currency exchange rate risk.
127) Under which of the following circumstances would a due bill be used?
(a) When an investor purchases a stock after the ex-dividend date, but prior to the dividend actually being paid.
(b) When an investor has purchased a stock shortly before the ex-dividend date.
(c) When a customer, in a regular-way trade, has failed to make payment for the purchase within 5 business days.
(d) In a buyer’s/seller’s option trade.
Next
Explanation:

The investor purchasing stock after the ex-dividend date but prior to the payment date would sign a due bill attesting to the fact that the former owner (the seller of the stock) isdue to be paid the amount of the dividend.
127)
The ex-dividend date for mutual fund distributions is:

(a) the date set by the fund
(b) two business days prior to the record date
(c) three business days after the trade date
(d) the fifth business date after the trade
Next
Explanation:
Unlike corporate dividends, in mutual funds, the fund itself—either the board of directors or the fund’s principal underwriter will set the ex-dividend date.
127)
When an investor has purchased “legend stock” from a Regulation D private placement, which of the following are true statements?
I. The stock must be held, fully-paid for 1 year before resale
II. After 1 year, there are no restrictions on sales for non-affiliates
III. After 1 year, sales of the stock are limited by SEC Rule 144
IV. After 2 years, there are no restrictions for a non-affiliate

(a) I, III
(b) I, II
(c) III, IV
(d) I, III, IV
Next
Explanation:

“Legend stock,” and “letter stock,” are just some of the names applied to unregistered, restricted stock issued under Regulation D. The investor, unless an institution, must hold the stock fully-paid for 1 year before sales. After the first year, the investor is limited in the quantity of stock that may be sold by Rule 144. After 2 years, a non-affiliate may sell without restrictions.
127)
From what source does a mutual fund pay dividends?

(a) Gains on stocks and bonds in the portfolio
(b) Net investment income
(c) Realized profits from the sale of securities held in the fund’s portfolio
(d) Net income after all senior securities have been paid
Next
Explanation:

A mutual fund pays dividends from its Net Investment Income, which is: dividends received from stocks in the portfolio, plus interest received from debt instruments, minus the fund’s expenses. Choice “A” describes the source of capital gains distributions. As a reminder, regarding choice “B” there are no senior securities in a mutual fund.
127)
An investor owns 600 shares of XYZ stock. XYZ has initiated a rights offering in anticipation to a new primary offering in the near future. The terms of the offering call for five rights and $30/share to purchase each share of the new stock during the rights offering period. If the investor wished to purchase a total of 1,000 shares of stock during the period, how many rights would he/she have to buy and exercise?

(a) 5,000
(b) 880
(c) 4,400
(d) 1,000
Next
Explanation:

When an investor owns stock of a company which initiates a rights offering, the investor will receive one right for each share of stock he/she owns. This investor owns 600 shares and will receive 600 rights. The terms of the offering call for the exercise of 5 rights to buy 1 new share. Since the investor wants a total of 1,000 shares, he/she will need 5,000 rights and would have to buy 4,400 additional rights (5,000 -; 600 = 4,400).
In Progress : Question No. 127 of 602
127) Which of the following statements regarding existing properties real estate limited partnerships is not true?
(a) A greater certainty of operating costs.
(b) Tax credits may be available for certain type projects.
(c) The possibility of capital appreciation.
(d) An advantage of this type of limited partnership is the use of accelerated depreciation.
Next
Explanation:

The statement: “An advantage of this type of limited partnership is the use of accelerated depreciation.” does not apply to real estate partnerships, which may only use straight-line depreciation. All of the other choices are considered advantages. Tax credits may be available if the limited partnership has purchased and is rehabilitating designated historical properties. Records regarding operating costs should be available, and there is a possibility of capital appreciation, though lower than that on new construction.
127)
Which of the following would be most likely issuer(s) of equipment trust certificates?
I. FastAsCanBe Airlines
II. The Goodenuff Aircraft Manufacturing Company
III. South to North Railroad Lines
IV. Haul A Lot Boxcar Manufacturing Company

(a) I, II, III, IV
(b) II, III
(c) II, IV
(d) I, III
Next
Explanation:
Equipment trust certificates are a form of corporate bond issued primarily by transportation companies. These bonds use the equipment purchased by the bond issue as collateral for the bonds

127)
In a short margin account, an investor sells 1,000 shares of FBN @ 1.75. What is the minimum requirement for this transaction?

(a) $525
(b) $2,000
(c) $875
(d) $2,500
Next
Explanation:
One of the miscellaneous rules specific to short margin accounts, is sometimes referred to the cheap stock rule. If an investor shorts stock from $0 -$2.50, the requirement is $2.50 per share; $2.50 – $5.00, the requirement is 100% per share; above $5.00 the requirement is 30% of CMV (current market value) or $5.00, whichever is greater.
Quit
127)
Municipal offerings are frequently oversubscribed. There is more demand for the bonds than there are bonds available. Which of the following is the correct sequence in which orders will be filled?

I. Member
II. Pre-sale
III. Syndicate
IV. Designated

(a) I, II, III, IV
(b) II, III, IV, I
(c) III, IV, II, I
(d) II, I, II, IV
Next
Explanation:

The standard progression of filling orders in a municipal offering puts pre-sale orders first; followed by syndicate or group orders, then designated orders and finally, at the last, those orders from members of the syndicate.
n Progress : Question No. 127 of 602
127) Which of the following statements with respect to passive real estate investments is (are) true?
I. REITs are most beneficial to institutions that prefer to diversify their real estate exposure.
II. REIT managers may only use the capital raised through the issuance of the trust units in order to buy properties.
III. Instead of acquiring properties, mortgage REITs simply make out real estate loans.
IV. In the U.S., REITs do not have to pay taxes on their profits if they pay out at least 75% of their income to shareholders by way of dividends.
(a) III and IV
(b) III and II
(c) I only
(d) I and IV
Next
Explanation:
Choice I is incorrect because REITs were primarily constructed to suit the needs of small investors who wanted to have real estate exposure but did not have the financial resources to buy properties outright. Choice II is incorrect because REIT managers may also finance their acquisitions through the issuance of debt.
127) A registered representative has an investor with 10 convertible debentures issued by the GHJ Company the bonds convert into 25 shares of GHG common stock. The RR learns that GHJ intends to call the bonds at par within a month and, checking the price of GHJ common, learns that it is currently trading at $42. Which of the following recommendations should the RR make?
(a) Sell the bonds in the market at par
(b) Tender the bonds for conversion and sell the stock short-exempt
(c) Buy 25 shares of stock and wait for the call
(d) Answer the call
Next
Explanation:
The bonds convert to 25 shares, which means the conversion price is $40/share ($1,000 par / 25 shares = $40/share). If the investor tenders the bonds for conversion and immediately sells the stock short, he will realize a profit over the par call. The short sale is known as “short exempt” because it is done simultaneously with conversion of the bond.
127) An investor bought a municipal bond at the primary offering from one of the underwriting syndicate. The bond was bought at 90 to yield 4.20@ in 20 years. After holding the bond for 10 years, the investor sold the bond at 95. What are the tax implications?
(a) The investor’s cost basis is not adjusted because this is a municipal bond, but he pays taxes on a $50 capital gain.
(b) The investor was taxed year on the accretion and has a $50 capital gain.
(c) The investor paid taxes on the accretion each year, but has no capital gain.
(d) The investor’s accretion was tax-free and he has no capital gain.
Next
Explanation:
The bond was bought in the primary market and is an OID (Original Issue Discount) bond. The appreciation each year—the accretion—is treated as municipal interest, and is tax-free. The bond must be accreted, however, to determine the loss or gain upon sale. Each year, this bond was accreted by $5. ($100 discount divided by 20 years = $5/year). After holding the bond for ten years, the adjustment to the basis was $50, which was added to the original basis to produce the adjusted basis ($50 + $900 = $950). Since the bond was sold for $950, there is no gain or loss.
127)
A corporation has filed a registration statement with the SEC, but the effective date has not yet been established. During this period of time while the offering is “in registration,” in which of the following activities may the underwriters engage?

I. Accept indications of interest from possible investors
II. Publish a research report for possible investors
III. Publish and distribute a preliminary prospectus
IV. Hold a due diligence meeting

(a) I, II, III, IV
(b) II, IV
(c) III, IV
(d) I, III, IV
Next
Explanation:
The period of time between the filing of the registration statement and the release by the SEC for sale (with a final prospectus) is known as the “cooling-off” period, which is typically 20 days. During this time the activities of the underwriters (and the company itself) are strictly limited. No communications with the public, other than the preliminary prospectus—known as a “red herring”—are permitted. A research report would not be allowed.
127)
When an investor is watching a proposed corporate merger and buys the stock of the company that is the takeover target while selling the acquiring company’s stock short, he/she is engaging in

(a) naked stock transfers
(b) risk arbitrage
(c) conversion arbitrage
(d) covered stock transactions
Next
Explanation:
A highly sophisticated investment strategy, risk arbitrage involves the buying of a company that is the likely takeover target in a merger; while simultaneously selling the stock of the company making the acquisition short. If the investor is right, he/she will make a profit on both directions. If wrong, he/she loses in both transactions.
n Progress : Question No. 127 of 602
127)
An investor purchased a municipal bond in the market when it was quoted a 120 and had 20 years remaining to maturity. If, after holding the bond for 10 years, the investor sells it at 110, what are the tax consequences?

(a) Depreciation of $100
(b) Accretion of $100
(c) No gain, no loss
(d) A loss of $100
Next
Explanation:

All premium bonds must be amortized. When the bond was purchased at $1,200 (120), it had 20 years remaining to maturity. The amortization is ($200 premium ÷ 20 years = $10/year). After ten years the basis in the bond is amortized by $100 (10 years × $10year = $100) and is now $1,100 ($1,200 – $100 = $1,100). Since the investor sold the bond at the amount of the amortized basis, there is no gain or loss for tax purposes.
Rank the following municipal bonds in order from safest to riskiest.
I. Revenue bonds
II. Moral obligation bonds
III. Public housing authority bonds
IV. Industrial development revenue bonds
(A) I, II, III, IV
(B) III, II, I, IV
(C) II, III, IV, I
(D) II, IV, III, I
The correct answer is Choice (B). If you remember that public housing authority bonds are considered the safest of the municipal bonds because they’re backed by U.S. government subsidies, this question’s easy. Public housing authority bonds would be the safest; moral obligation bonds are also considered very safe because the state government has a moral obligation to help pay off the debt if needed, follow.
In Progress : Question No. 127 of 602
127) Which of the following most closely resembles accumulation units of a variable annuity?
(a) Open-end investment company shares
(b) Closed-end investment company shares
(c) CMOs
(d) REITs
Next
Explanation:
Accumulation units of a variable annuity are very much like mutual fund shares except for some of the charges and lack of control over distributions. They have voting rights and valuation schedules that are the same as mutual fund shares. REITs and CMOs are mortgage-based investments. Closed-end company shares are virtually the same as the stocks of other companies.
127)
A registered representative is looking at two attractive issues of municipal as possible recommended additions to a customer’s portfolio. Which of these bonds would have the higher yield?

(a) A municipal issue subject to the AMT
(b) Bonds originally sold at a premium
(c) A municipal issue not subject to the AMT
(d) Bonds originally sold at a discount
Next
Explanation:
The AMT, Alternative Minimum Tax, laws specify certain investment instruments as “tax preference items.” These are also called “add-backs” because, if a taxpayer crosses certain thresholds of income and deductions, these are added back to the taxes due. One of the tax preference items is interest on certain municipal bonds; these are considered to be subject to the AMT and therefore would carry a higher yield because of their greater exposure to taxes.
Quit
127)
Which of following types of orders may never be given to a specialist?

(a) Stop loss
(b) Above the market limit
(c) Buy stop limit
(d) Not Held
Next
Explanation:

A “not held” instruction (abbreviated NH) is a market order to buy or sell securities in which the customer has given the floor broker the choice of time and price for execution. The floor broker is not held responsible for a specific price or time, but cannot leave the order with the specialist.
n investor tells his registered representative that he is interested in learning more about a single-state municipal bond mutual fund created for his home state. He says that his motivation is to receive totally tax-free distributions from the fund. The RR should advise the customer that:

(a) He can only qualify to invest in such a fund if the fund’s bond portfolio does not contain bonds subject to the AMT.
(b) The mutual fund’s distributions will exempt from state taxes but may be taxed at the federal level.
(c) Since he resides in the state where the bonds in the fund’s portfolio are issued, all distributions will be tax-exempt at both the state and federal levels.
(d) Not all distributions from the fund will be totally tax free.
Next
Explanation:
State-specific municipal bond mutual funds do indeed providedividends that are free from both state and federal taxes. Any distribution of capital gains by the fund will be taxable at both levels. Mutual fund dividends come from Net Investment Income,which is dividends received by the fund + interest received by the fund minus fund expenses. Capital gains distributions from a mutual fund come from securities held by the fund for a year-and-a-day and sold for a profit.
127) Use the following information about a customer’s margin account to answer the question.

ABC stock – 200 shares. CMV $40/share
PDQ stock – 100 shares. CMV $30/share
LIB stock – 100 shares. CMV $50/ share
Debit balance — $6,400

If this customer used any SMA in the account to its maximum by purchasing stock, the DR balance in the account would be:
(a) $3,200
(b) $8,000
(c) $9,600
(d) $12,800
Next
Explanation:
Any time an investor uses SMA, he/she is borrowing money from the broker-dealer and increasing the DR balance. SMA has a purchasing power equaling the SMA x 2 ($1,600 x 2 = $3,200). The broker-dealer will loan the client an additional $3,200 to purchase stock. Add this to the existing DR balance of $6,400 + $3,200 = $9,600.
n Progress : Question No. 127 of 602
127)
An investor has just celebrated her 71st birthday. She has been contributing to her Roth IRA for more than 20 years and has planned to continue her contributions. When she asks her RR for advice regarding taxation on the account, the Rep should advise her that:

(a) She is subject to taxation and penalties for failure to begin withdrawals in accordance with her life expectancy.
(b) The penalty for failing to withdraw at age 70 ½ is 10%.
(c) The penalty for failing to withdraw at age 70 ½ is 50% of the insufficient withdrawal amount.
(d) There are no taxes or penalties due and she may continue to contribute to the account.
Next
Explanation:

In a Roth IRA, unlike a traditional IRA, once the participant has reached age 59 ½ and has held the account for 5 years, there are no penalties for a tax-free withdrawal. Additionally, the Roth IRA permits the participant to continue contributions after 70 ½ and imposes no penalties for “late withdrawal.”
13)
Which of the following would a registered representative use as a source of information for corporate bonds?

(a) Yellow Sheets
(b) INSTINET
(c) NASDAQ Level 1
(d) Pink Sheets
Next
Explanation:

The Yellow Sheets, published by the National Quotations Bureau, provide pricing and market maker information on corporate bonds. The NASDAQ system is a quotation system for OTC stocks. Pink Sheets provide information on non-NASDAQ OTC stock. INSTINET is a trading system used by institutional traders in the Fourth Market.
In Progress : Question No. 16 of 197
16) What is the holding period requirement for dividends paid on common stock to receive favorable tax treatment as “qualified dividends”?
(a) At least 90 days prior to the ex-dividend date
(b) A minimum of 30 days before or after the ex-dividend date
(c) At least 61 days during the 121-day period that begins 60 days before the ex-dividend date
(d) A minimum of 180 days prior to the ex-dividend date
Next
Explanation:
The Jobs and Growth Tax Relief Reconciliation Act of 2003 reduced the top federal income tax rate on “qualified dividends” received by individual taxpayers to 15%, the same as the top tax rate on long-term capital gains. Generally, to meet the holding period requirement, a shareholder must have held the stock for more than 60 days during the 121-day period that begins 60 days before the ex-dividend date.
21)
If delivery in a stock trade is specified as a “seller’s option,” what is normally the latest that the seller can deliver the stock?

(a) 3 business days
(b) 4 calendar days
(c) 60 calendar days
(d) 35 business days
Next
Explanation:

In this type of agreement, the seller is given the option of delivering the stock to the buyer on the date the seller’s option expires; or earlier with a 1 business day notice. The seller’s option typically runs from 4 business days to a maximum of 60 calendar days.
25)
If an options contract is described as having “European-style” exercise, this means which of the following?

(a) The option may only be on a particular foreign currency.
(b) The investor who is long the contract may only exercise the option on the last trading day before expiration.
(c) The party who has bought the contract is obligated to exercise it during the final week prior to expiration.
(d) The holder of the contract may only close the position within a certain time frame.
Next
Explanation:
European-style exercise in an options contract does not necessarily refer to currency options. It does restrict exercise—but not closing—to the last business day before the contract expires.
26)
A married couple has a young daughter for whom they want to begin a college savings plan. Their registered representative has recommended a 529 plan as the vehicle which will allow them to maximize their contributions. What is the maximum contribution that they may make as a couple in any given year?

(a) $2,000
(b) $55,000
(c) $11,000
(d) $110,000
Next
Explanation:
The rules regarding 529 plans allow contributions for couples to be $22,000 per year, but allow 5 years contributions to be made at one time. The couple may contribute $110,000 in one year but may make no more contributions for 5 years.
27)
The daily publication known as “Yellow Sheets” detail quotes for which of the following:

(a) Municipal bonds
(b) Government bonds
(c) ADRs
(d) Corporate bonds
Next
Explanation:

The “Yellow Sheets” provide BID and ASK quotations by market makers on OTC corporate bonds only. These publications are named for their color and are actually published byPink Sheets, LLC.
34)
If a stock held by a customer in a margin account pays a cash dividend, which of the following statement(s) is/are true?
I. The customer may receive the dividend in cash
II. The firm will require that the dividend be left in the account.
III. The customer may apply the dividend against the debit balance.
IV. If the dividend is left in the account and applied to the debit balance, the SMA will increase.

(a) I, III, IV
(b) II
(c) III, IV
(d) III
Next
Explanation:

The customer has the option of taking the cash dividend or leaving it in the account to be applied to reduce the debit balance. If it is applied against the debit balance, it will increase the SMA in the account on a dollar-for-dollar basis.
37)
Which of the following instruments may be callable?
I. HH Bonds
II. T – Bills
III. Treasury Bonds
IV. Preferred stock

(a) I, III, IV
(b) I, II, III, IV
(c) III, IV
(d) III
Next
Explanation:

Treasury bonds are callable, usually over the last 3-5 years before maturity. One of the characteristics that preferred stocks share with bonds is callability. HH bonds and T-Bills are not callable instruments.
If an investor is bearish on XYZ stock, which of the following positions would he/she be most likely to take?
(l) Short the 50 call and long the 55 call
(l) Short the 50 call and long the 60 call
(g) Short the 60 call and long the 50 call
(g) short the 55 call Long the 45 call and

In Progress : Question No. 52 of 197
52) Which of the following economists is considered to be a monetarist?
(a) Arthur Laffer
(b) Ludwig Von Mises
(c) Milton Friedman
(d) John Maynard Keynes
Next
Explanation:
Milton Friedman, of the Chicago School of Economics, believed that the money supply was the key to the economy. He developed his theories toward the end of the Great Depression. Arthur Laffer is of the “supply-side” school of economics. Von Mises was the father of the Austrian School of Economics – his theories embraced the philosophy that the government had no place in the markets. Just opposite was Keynes who taught that the government should manage the economy through its programs.
55) In the protective covenants of a municipal revenue bond’s indenture, the bond is described as having a “closed-end indenture.” This would mean which of the following to an investor?
(a) The closed-end indenture puts a statutory ceiling on the amount of municipal debt that is payable from ad valorem taxes.
(b) The issuer may only issue additional bonds if application of the “additional bonds test” demonstrates that the new issue will not result in risk to the original bondholders.
(c) The issuer may only issue additional bonds that will have a senior lien to the bond’s collateral.
(d) The bond’s issuer may not issue additional bonds that will have an equal claim to the revenues, with a project-completion clause.
Next
Explanation:
When a municipal bond’s indenture has a protective covenant describing a closed-end indenture, the issuer of the bonds is prohibited from issuing additional bonds with equal claim to the revenues of the facility. The project completion clause is a hedge for the issuer in case it is necessary to issue additional, equal, bonds to complete the project in case of cost overruns.
57) An investor bought a new full faith and credit bond issued by the city of Limrock. The bond had a twenty-year maturity, a coupon 4.35 and was purchased for $900. Ten years later, the investor sold the bond for $950. What are the tax implications for this investor? I. The investor has paid taxes on a total accretion of $50 II. The investor will have a $50 capital gain III. The investor has accreted the bond, but has paid no taxes on the accretion IV. The investor will realize no gain; no loss
(a) I, IV
(b) II, III
(c) III, IV
(d) I, II
Next
Explanation:
There are two problems in this question. The first is to recognize the fact that this bond is an Original Issue Discount Bond (OID). All discount bonds, including OID municipals, must be accreted, but the tax rules allow the accretion to be treated as municipal interest—tax exempt. The amount of accretion is $50 ($100 discount ÷20years = $5/year × 10years = $50). The original basis of the bond, $900 is accreted by $50 to $950, so when the bond is sold for $950 there is no capital gain or loss.
59)
A class of options would include which of the following?
I. Underlying Stock
II. Strike price
III. Type of contract
IV. Expiration month

(a) I, II, IV
(b) II, IV
(c) I, III
(d) I, II, III, IV
Next
Explanation:
A class of options contracts includes all the contracts that have the same underlying stock and type. For example, all the XYZ calls—regardless of expiration or strike price constitute a class. A seriesof contracts has all four of the choices above.
61) Which of the following measurement tools used in the fundamental analysis of a company is the most tangible measure of the value of a share of common stock?
(a) Book value
(b) Price/earnings ratio
(c) EBIT
(d) Earnings per share
Next
Explanation:
The formula for the book value (BV) per share of common stock of a corporation comes from the balance sheet: Total Assets – Intangible Assets – Current and Long-Term Liabilities -Preferred Stock at Par. This is the theoretical liquidating value of the company (the total net assets). Dividing this by the number of common shares gives the BV per share.
n Progress : Question No. 66 of 197
66) The NASD Rule dealing with the purchase of IPOs, by persons associated with member firms, in general terms states which of the following?
(a) A person associated with a member firm may not purchase any IPO.
(b) Such persons are restricted persons and may not buy new issues when they have become “hot.”
(c) Restricted persons may only purchase “hot” issues if they are a less-than 25% beneficial owner in the account.
(d) Only the family members of associated persons may buy an IPO, “hot” or not, not the person himself/herself.
Next
Explanation:
NASD Rule 2790 did away with the old “free-riding and withholding rule.” Under the new rule, any officer, director, general partner, associated person or employee of any NASD member firm is a restricted person and may not purchase an IPO. This restriction also includes a prohibition against any member firm selling an IPO to any account that includes a restricted person or his/her immediate family.
Quit
68)
A customer has entered the following order: Buy 200 shares XYZ @45 Stop Limit.
After the order was entered, the following trades in XYZ are reported:

44.44.46.46.47.45.46
Which transaction elected the order?

(a) The trade at 46
(b) The second trade at 44
(c) The trade at 47
(d) The trade at 45
Next
Explanation:
A buy stop limit is typically entered to automatically execute in a rising market with a limit on the price paid by the investor. The order “stop” is elected or triggered when the market trades at or through the stop price. When the stock trades at 46, it has gonethrough the stop price and the order is now a live limit order.
69)
The Trust Indenture Act of 1939 requires:

(a) a legal contract between the issuer of corporate bonds and the trustee
(b) a legal contract guaranteeing minimum payments to preferred stockholders
(c) a legal contract between Corporations and their stockholders
(d) a legal contact between issuers of bonds and their bondholders
Next
Explanation:

The Trust Indenture Act of 1939 requires that corporate issuers of bonds establish a trust for the protection of bondholders. The trustee, a commercial bank, is the fiduciary for the bondholders. The contract, or indenture, is between the corporate issuer and the trustee bank.
A mutual fund’s prospectus describes the fund as a “regulated” investment company and states that its primary objective is to provide income to its shareholders through regular distributions from its balanced portfolio. Which of the following describes a “regulated” investment company?

I. The mutual fund must distribute at least 95% of interest income to qualify as a regulated investment company.
II. The fund may also be described as operating under the “pipeline” or “conduit” tax theory.
III. The fund must distribute at least 90% of its net investment income and capital gains to qualify as a regulated investment company.
IV. The mutual fund must only distribute 90% of its dividend income.

(a) I, IV
(b) I, II
(c) II, IV
(d) II, III
Next
Explanation:
The term “regulated investment company” is synonymous with “pipeline” or “conduit” tax theory. Such a fund must distribute at least 90% of its net investment income (dividends from stock + interest from bonds -; fund expenses) as dividends and at least 90% of its capital gains to avoid being taxed on this income at the fund level. All distributions are taxable to the shareholders.
73) Which of the following statements are true regarding a mutual fund that charges 12b-1 fees?

I. The 12b-1 fee plan must have been initially approved by a majority vote of the outstanding shares of the fund and a majority vote of the fund’s board of directors.
II. 12b-1 funds are no-load funds.
III. The board of directors of the fund must review the 12b-1 plan at least quarterly and issue a written report of expenditures from the fees collected.
IV. The 12b-1 plan may be terminated by a majority vote of the outstanding shares or by a majority vote of the non-affiliated members of the fund’s board of directors.
(a) I, II, III, IV
(b) I, II, III
(c) I, III, IV
(d) III, IV
Next
Explanation:
12b-1 funds are not truly “no load” since they assess a fee, quarterly, based on the assets of the fund. They are sometimes called “level load” or Class “C” shares. 12b-1 fee money may only be used for the purpose of distributing shares to new investors, thus the requirement for a quarterly review of expenditures. Termination does not require a majority vote of the whole board of directors, just a majority of the non-affiliated (non-interested or outside) directors or a majority vote of the outstanding shares.
In Progress : Question No. 76 of 197
76)
In a new municipal offering, which of the following is a function of the bond counsel but not a function of the syndicate counsel?

(a) Writing the official statement
(b) Writing the reoffering scale
(c) Writing the syndicate letter
(d) Writing the legal opinion
Next
Explanation:

The function of the bond counsel, or independent bond attorney, for a municipal offering is to write the legal opinion. Since municipal bonds are not registered with the SEC, the bond counsel establishes the legality for issuance and, very importantly, the tax implications for the bond.
In Progress : Question No. 80 of 197
80) What does the Placement Ratio in the “Bond Buyer” show for investors?
(a) A ratio of revenue bonds to IDRs coming to market for the past 30 days.
(b) The percentage of the past week’s new municipal bonds that have been placed by the underwriters.
(c) The ratio between GO and Revenue bonds placed by issuers in the past 30 days.
(d) The number of new bond offerings that have been placed with underwriters.
Next
Explanation:
The “Bond Buyer” is considered the best source of information regarding new municipal securities. One of the pieces of information that is invaluable to municipal underwriters is the “Placement Ratio.” The placement ratio is the percentage of the past week’s new municipal bonds that have been placed by the underwriters, and is compiled at the close of business every Thursday and appears in the Friday edition of this publication.
81)
In which of the following bond issues would an analyst be concerned with ad valorem taxes?

(a) A bond issued by a pollution control agency of the state
(b) A bond issued by the Industrial Development Commission of Richford County
(c) A bond issued by the Wheatsis County School District
(d) A bond issued by the Airport Authority of Rockledge County
Next
Explanation:

Bonds issued by authorities, commissions and agencies are normally revenue bonds. G.O. bonds are typically based on the collection of ad valorem taxes (property taxes) and one of the prime examples is a school construction bond.
85)
Which of the following insure municipal bonds for the timely payment of interest and principal?

I. MBIA
II. FGIC
III. AM Best
IV. AMBAC

(a) II, III, IV
(b) I, II, IV
(c) I, IV
(d) I, II, III
Next
Explanation:
The three companies which are the principal insurers of municipal bonds are AMBAC -; American Municipal Bond Assurance Corporation; FGIC -; Financial Guarantee Insurance Company and MBIA -; Municipal Bond Insurance Association. A.M. Best is a company that rates insurance companies for safety. By the way, the companies do not insure against market risk.
87) In which of the following types of DPPs is accelerated depreciation considered an advantage?
(a) Real estate
(b) Cattle feeding
(c) Raw land
(d) Equipment leasing
Next
Explanation:
A major advantage for equipment leasing programs is that they may use accelerated depreciation—taking larger write-offs in the early years against any income. Raw land and cattle feeding DPPs cannot use depreciation. Real estate partnerships may use depreciation, but only straight-line.
90)
Which of the following are part(s) of the Federal Farm Credit System?
I. FHLBs
II. FICBs
III. COOPs
IV. FLBs

(a) II, IV
(b) II, III, IV
(c) I, IV
(d) I, II, III, IV
Next
Explanation:
Federal Intermediate Credit Banks (FICBs) make loans to farmers for equipment, livestock, etc. Federal Land Banks (FLBs) loan mortgage money to farmers for land. Banks for Cooperatives (COOPs) make loans to farm cooperatives. FHLBs are Federal Home Loan Banks and are not a part of the farm system.
96) Under the Act of 1933, securities from which of the following issuers are exempt from registration

I. U.S. Government
II. National Banks
III. Municipalities
IV. Public Utilities
(a) I, III, IV
(b) I, II, IV
(c) I, III
(d) I, II, III, IV
Next
Explanation:
Of the choices given, only U.S Government and municipal issuers are exempt from registration under the Act of 1933. These issuers are exempt from virtually all regulation except the anti-fraud laws. Public utilities and banks enjoy an exemption, in most cases, from registration with the states under the Uniform Securities Act, but not the Act of 1933.
97)
Under MSRB Rule G-28, if a municipals RR wishes to open a securities trading account with another broker-dealer, what are the obligations of firm opening the account?
I. Written permission by the employing member firm
II. Duplicates sent automatically to the employing firm.
III. Written notification sent to the employing firm.
IV. Duplicates sent to the employing firm upon request

(a) IV
(b) I, IV
(c) I
(d) II, III
Next
Explanation:

MSRB Rule G-28 relates to Transactions with Associated Persons of Other Municipals Securities Professionals. The Rule requires that the firm opening an account for the RR of another firm provide the employing firm with written notification and, if the account is opened, send automatic duplicates.
In Progress : Question No. 100 of 197
100)
Under the rules of the MSRB, which of the following are considered to be advertising?
I. Abstracts of official statements
II. Preliminary official statements
III. Market letters to existing customers
IV. Summaries of official statements

(a) I, II, III
(b) I, II, III, IV
(c) II, IV
(d) I, III, IV
Next
Explanation:

Advertisements are defined as any communications with the public other than official statements and preliminary official statements. The other documents specifically fall within the definition.
In Progress : Question No. 101 of 197
101)
In a mutual fund, what is the single greatest item of expense to the fund?

(a) Investment advisory fees
(b) Advertising and sales literature
(c) Fees paid to the transfer agent
(d) Expenses of the fund’s board of directors
Next
Explanation:
Fees paid to the Investment Advisor are calculated as a percentage of the assets managed. By far, the fees paid to the Advisor constitute a fund’s greatest expense. Advertising and sales literature are not expenses of the fund, they are expenses of the fund’s Underwriter.
102) An individual has recently passed her Series 7 exam. The firm with which she is registered wants her to be able to offer her clients wrap accounts. Which of the following are necessary qualifications for the RR?
(a) Passing the Series 66 exam
(b) Passing the Series 4 exam
(c) Passing the Series 63 “Blue Sky” exam
(d) Passing the Series 24 exam
Next
Explanation:
If the RR is to offer wrap accounts in which the client’s assets are managed by one or more money managers and all transaction and management fees are “wrapped-up” into a single fee (calculated as a percentage of the assets managed), the RR must pass the Series 66 exam. The Series 66 combines the Series 63, the state securities exam (Blue Sky) and the Series 65 (the Investment Adviser Representative) exams. The Series 63 alone does not qualify an RR to sell investment advice. The Series 24 is the exam for General Securities Principals and the Series 4 is the exam for Options Principals.
1)
When a customer is unable to make payment for securities in a regular way trade within the time allotted under Regulation “T,” an extension of time may be granted by which of the following?
I. The exchange where the stock is listed
II. The SEC
III. The NASD
IV. The Federal Reserve

(a) I, IV
(b) II, III, IV
(c) I, II, III, IV
(d) I, III, IV
Next
Explanation:
When a customer must obtain an extension of time past the 5 business days allotted under Reg. “T,” the extension may be obtained all of the choices above except the SEC, which does not become involved in the extension of credit. If the stock is a listed stock, normally the extension would be granted by the exchange. If the stock is an OTC stock, the extension application would go to the NASD. Since Reg. “T” is a Federal Reserve Board Regulation, the Fed could also grant the extension.
2) Which of the following are characteristics/functions of a specialist on a stock exchange, but are not functions of the order book official on the CBOE?

I. Maintains an orderly market
II. Takes customer and member firm orders
III. Trades as a market maker
IV. Not employed by the exchange
(a) I, II, III, IV
(b) I, II
(c) II, III, IV
(d) I, III, IV
Next
Explanation:
Except for the function of maintaining an orderly market, the Order Book Official (OBO) on the CBOE has functions and characteristics that are quite different. The OBO takes only customer orders from floor brokers; may not trade for its own account and is an employee of the options exchange.
4)
In a new stock offering, which of the following approves the release of a “red herring”?

(a) The underwriting firm
(b) The selling group
(c) The SEC
(d) The NASD
Next
Explanation:

The SEC neither approves nor disapproves of any prospectus or preliminary prospectus—the red herring. The NASD similarly does not approve or disapprove and does not review the registration statement. The first document to be released by the underwriting syndicate for use with the public—during the “20-day cooling-off” period is the red herring, so called because portions of the print on the cover sheet are in red.
5) What regulatory standards are applied to the use of options educational material to be used with the public prior to the delivery of the Options Disclosure Document?
(a) The material must be filed with the NASD’s Advertising Regulation Department prior to first use or publication.
(b) Options advertising and educational material must be filed with the Advertising Regulation Department within 10 days of first use.
(c) The SEC must approve options advertising and educational material at least 10 days prior to first use.
(d) Such materials must be filed with the Advertising Regulation Department at least 10 days prior to first use or publication and must not be used prior to approval by the department.
Next
Explanation:
The NASD’s FAQ on advertising states that, “Options advertisements, educational material and sales literature should comply with NASD Conduct Rule 2220 … Advertisements and all educational material for options used prior to delivery of the Options Disclosure Document must be submitted for review at least 10 days prior to use. The options material must not be used prior to final approval by the Department.” The word “approval” may distract you at first, but this is the exception to the general rule.
Which of the following enforce the MSRB rules for broker-dealers?
I. The MSRB Enforcement Authority
II. The SEC
III. The FDIC
IV. The NASD

(a) I, II
(b) I, II, III
(c) II, IV
(d) II, III, IV
Next
Explanation:

The acronym MSRB stands for Municipal Securities Rulemaking Board. That organization makes rules, but doesn’t enforce them. For broker-dealers, the rules are enforced by the NASD and the SEC. For bank dealers, the rules are enforced by the Federal Reserve, the FDIC or the Comptroller of the Currency.
7)
The securities offerings of which of the following are part of the FFCS?
I. FLBs
II. FICBs
III. FHLMC
IV. COOPs

(a) III, IV
(b) I, II, III
(c) I, II, III, IV
(d) I, II, IV
Next
Explanation:

FFCS stands for Federal Farm Credit System. Those securities that are a part of the consolidated system are Federal Land Banks (FLBs), Federal Intermediate Credit Banks (FICBs) and Banks for Cooperatives (COOPs). FHLMC is the Federal Home Loan Mortgage Corporation.
8)
A corporation has decided to raise additional working capital by offering stock that was authorized in its charter but not issued. Which of the following statements are true about this offering?
I. It is an additional IPO.
II. A prospectus must be available for 40 days.
III. It is a primary offering
IV. A prospectus must be available for 90 days

(a) II, III
(b) I, IV
(c) I, II
(d) II, IV
Next
Explanation:
An IPO, Initial Public Offering, only happens once. When a company decides to raise additional capital it may make an additional or subsequent, primary offering. The stock it is selling has never been owned by anyone before. A prospectus for such an offering must be made available by the underwriting syndicate for 40 days.
In Progress : Question No. 9 of 87
9) When a prospective municipal issuer publishes an “official notice of sale” in the Bond Buyer, what is the purpose of this notice?
(a) A notice that a municipal issuer has sold all the bonds in a new issue.
(b) A notice to the general investing public, that the issuer will be selling a new issue of bonds within the next 30 days.
(c) An invitation to prospective underwriters to submit sealed bids for an upcoming bond offering.
(d) An invitation to the citizens and taxpayers that the issuer will sell municipal bonds to investment banking firms.
Next
Explanation:
An Official Notice of Sale published in the Bond Buyer invites prospective underwriters to submit sealed bids for an upcoming issue. The notice will give the exact time and place where the bids must be submitted and will give the name of the person to whom the bids must be submitted, as well as other information.
10) Which of the following statements regarding the OCC is/are true?
(a) It sets the options contract size, expirations, strike price intervals and premiums.
(b) It is the principal trading place for equity options.
(c) It is the issuer and guarantor of listed options contracts.
(d) It is the marketplace where one would find the OBO.
Next
Explanation:
The Options Clearing Corporation, (OCC) is said to; “get in the middle of options transactions” that have been made on the various exchanges where options are traded. It is considered to be the issuer of contracts because, until OCC records a contract as being opened, there is no contract. The financial integrity of the (OCC), which is owned by the options trading exchanges, guarantees the contracts. This is possible because of the financial requirements levied on the broker-dealers who participate in the options market.
12) Russ is an active RR in North Carolina and is registered only in that state. One day, when he returns from lunch, his sales assistant has left a note saying that Mary, one of his best clients, has placed a market order to buy 300 shares of XYZ and asked that he call her when he returns to tell her the execution price. When he starts to dial the number, he recognizes that the area code is not a North Carolina number. When he reaches Mary and asks about the telephone number, she tells him that she moved to Tennessee about six weeks ago. What actions should the RR take regarding his client and her account?
(a) Cancel the order and place it with an office of the broker/dealer in Tennessee.
(b) Take the Tennessee state securities exam.
(c) Take action to become registered in Tennessee.
(d) Tell Mary that the order cannot be executed.
Next
Explanation:
One of the major reasons for the Uniform Securities Act (the Blue Sky Laws or USA) was to provide uniformity in registration of representatives. If an RR is registered in one state, he or she can become registered in another by sending evidence of registration to the new state. The RR’s broker/dealer must, however, be registered in the new state.
Quit
You have answered 8 out of 87 questions correctly for a score of 9.2%.
The correct answer is c. Your answer was correct!
13) Under the NASD rules, which of the following is considered to be advertising?
(a) Reprints from newspapers or magazines
(b) A broker/dealer’s web site
(c) The script for a seminar
(d) A group email sent to 16 clients
Next
Explanation:
Advertising, as defined by NASD Rules, is mass public communications. The broker/dealer cannot control the audience. In all of the instances above, the firm is in control of the audience for which the communication is intended, but because the broker/dealer cannot control who visits the website, this is considered advertising.
19) A registered representative is completing account documentation for a new customer. During the fact-finding process, he learns that the person opening the account is affiliated with a NYSE firm. Which of the following actions must the representative opening the account take?

I. Notify the employing firm that one of its employees is opening the account
II. Send duplicates of transaction documents to the employing firm automatically
III. Obtain written permission from the employing firm before opening the account
IV. Send duplicates of transaction documents to the employing firm upon request
(a) I
(b) I, II
(c) I, IV
(d) II, III
22)
A RR overhears two individuals from his firm’s investment banking department discussing a “green-shoe” agreement. The term means that:

(a) the company has solidified a contract with the Green Shoe Corporation.
(b) the underwriting contract for a company was a best-efforts agreement.
(c) the syndicate has a clause in the underwriting agreement that calls for the company to authorize the issuance of additional shares in case of high public demand.
(d) the indications of interest for an upcoming stock offering have been slower than expected and the syndicate may be in a losing proposition.
Next
Explanation:
Although the term green shoe allegedly originated with a clause in an underwriting agreement with the Green Shoe company, the term now means that the syndicate has a clause in the underwriting agreement that calls for the company to authorize the issuance of additional shares in case of high public demand.
23)
Which of the following are exempt from the requirements of the Trust Indenture Act of 1939?
I. A U.S. Treasury Bond
II. A bond issued by the Toll Road Authority of Glenpark County
III. A convertible subordinated debenture issued by a railroad company.
IV. A Full Faith and Credit bond issued by the city of Sandlow

(a) I, III, IV
(b) I, II, IV
(c) I, II, III, IV
(d) I, IV
Next
Explanation:
The Trust Indenture Act of 1939 applies to corporate bond issues. Bonds issued by the U.S. Government and by municipal issuers are exempt from the requirements of this law. In fact, except for the anti-fraud laws, U.S. Government and municipal issuers are exempt from virtually all Federal regulation.
25) Which of the following is not covered by the NASD’s Code of Arbitration?
(a) A situation in which two registered representatives have agreed to share commissions and one has allegedly reneged on the deal
(b) A situation in which a customer has written a letter to a broker-dealer complaining that a registered representative has made trades in her account without authorization.
(c) A disagreement between a registered representative and a member firm regarding the amount of commissions to be paid for a transaction
(d) A dispute between a customer, who has signed an arbitration agreement, and a member firm regarding the lack of profitability of an investment
Next
Explanation:
The Code of Arbitration is designed to settle disputes arising within the investment community and with customers who have signed a pre-dispute arbitration agreement. A situation where a customer institutes a complaint in writing, alleging unauthorized transactions, to a broker-dealerwould most likely be handled by the NASD’s Code of Procedure through the Department of Enforcement.
26) After the receipt of a written complaint from a customer who has lost money in his account and before the matter reaches any level of hearing, the broker-dealer settles with the customer and pays him $20,000. After settlement, which of the following are required of the broker-dealer?
(a) Give notice to the Federal Reserve Board
(b) Apply to the SEC for special arbitration procedures
(c) Inform the appropriate SRO
(d) Dismiss the registered representative who handled the customer’s account
Next
Explanation:
Although the firm might fire the RR, it is not a requirement. Informing the SRO—Self Regulatory Organization—such as the NYSE or NASD is a requirement. The SEC does not get involved in arbitration and the FRB has no function in these proceedings.
Under Regulation “T” (50%) a customer is required to make payment, in a cash account, for securities purchased. If the client is unable to make suitable payment within that period of time, which of the following may grant an extension?
I. The exchange on which the stock is traded.
II. The NASD
III. The SEC
IV. The Federal Reserve

(a) I, II, III, IV
(b) I, II
(c) II, III
(d) I, II, IV
Next
Explanation:
Choice III, the SEC is the only wrong choice. If a stock is listed, the extension may be granted by the exchange. If the stock is OTC, the NASD may grant the extension. In any case, since Reg. “T” is a Federal Reserve Board Rule, the Federal Reserve banks have the authority to grant extensions of time.
28)
A broker-dealer firm is planning a series of options seminars to educate its customers and prospects about the use of listed options contracts for hedging and income strategies. The firm has produced a small booklet describing the use of options contracts to protect stock positions. What are the regulatory requirements for use of this material with the public?

I. It must be approved by the CBOE prior to use.
II. The booklet must be approved by a CROP of the firm.
III. A copy of the Options Disclosure Document must be provided to each attendee along with the booklet.
IV. The booklet must be approved by an ROP.

(a) I, IV
(b) I, III
(c) II, III
(d) III, IV
Next
Explanation:
The CROP (Compliance Registered Options Principal) must approve options communications with the public. The Options Disclosure Document serves the same purpose as a prospectus and must be provided to each person attending.
30) The Uniform Practice Code calls for settlement of trades in Government securities between member firms:

I. in clearinghouse funds
II. in three business days
III. in federal funds
IV. next business day

(a) IV
(b) III, IV
(c) II, III
(d) I, IV
Next
Explanation:

Member-to-member settlement in U.S. Government securities settle next business day (Trade + 1 business day). III would be correct if the trade settlement had no “float”, since these settlements are termed federal funds settlement. Federal funds are also used by the Federal Reserve to pay for its purchase of Government securities.
33)
Which of the following advertisement and sales literature material must be filed with the NASD’s Advertising Regulation Department within 10 days of first use or publication?

I. Electronic form letters concerning investment companies prepared for delivery to 25 or more prospective retail customers
II. Generic ads
III. Mutual fund family corporate image sales communications
IV. Reprints of magazine articles to accompany sales literature

(a) I, IV
(b) I, II, III, IV
(c) I, III, IV
(d) I, III
Next
Explanation:
All advertising and sales literature concerning registered investment companies must be filed with the NASD’s Advertising Regulation Department within 10 days of first use. Choices II and III are specifically included in filing requirements. If the investment company has never filed material with the Department (a new company) all advertising and sales literature must be filed at least 10 days prior to first use for one year. All material must be approved, dated and signed by a principal prior to submission.
34)
The Investment Advisors Act of 1940 and the Uniform Securities Act were both modified by recent legislation, the National Securities Markets Improvement Act of 1996. Under this law, which of the following investment advisors may be registered with the state only under the USA?

I. An advisor with $60 million in assets under management.
II. An advisor with $28 million in assets under management.
III. An advisor with $23 million in assets under management.
IV. The investment advisor of a mutual fund

(a) I, IV
(b) II, IV
(c) II, III, IV
(d) II, III
Next
Explanation:
The National Securities Markets Improvement Act (NSMIA) split the registration requirements for investment advisors. Advisors with less than $25 million under management register with the appropriate state(s) under the USA. Those advisors with assets under management between $25 million and $30 million may choose whether to register with the state or the SEC. If an advisor has more than $30 million under management the advisor -called a federal covered advisor – registers only with the SEC. Another example of a federal-covered advisor is the portfolio manager of a mutual fund.
35)
When a NYSE member broker-dealer is publishing a research report on a company, which of the following must approve the copy before use with the public.

(a) A Series 24 Registered Principal
(b) A Supervisory Analyst
(c) An Allied Member of the Exchange
(d) A Managing Trader
Next
Explanation:

Under the rules of the NYSE, only an individual who has passed a special qualification examination as a Supervisory Analyst can approve a research report for use with the public.
37)
When a registered representative has been sanctioned by the NASDR’s Office of hearing officers, what is the chain of appeals for the sanctions?

(a) The NASD’s Board of Governors, then the SEC
(b) The SEC and then the NAC
(c) The appropriate Federal court, then the SEC
(d) The NAC and then the SEC
Next
Explanation:
The first level, after sanctions by the Office of Hearing Officers (OHO) is the National Adjudicatory Council, and finally—within the investment community—the SEC. After that, the party making the appeal would have to go to Federal Court.
39)
Under the MSRB rules, which of the following is defined as a “customer”?

(a) A broker-dealer
(b) A bank dealer
(c) A bank
(d) All of the above
Next
Explanation:

The MSRB defines a customer as any person other than a broker-dealer, a bank dealer, and an issuer involved in issuing its own municipal securities. A bank is not a bank dealer.
In Progress : Question No. 40 of 87
40) The rules of the MSRB are enforced for bank dealers by which of the following regulatory authorities?

I. SEC
II. FRB
III. FDIC
IV. Comptroller of the Currency
(a) I, II, III, IV
(b) I, II, III
(c) II, III
(d) II, III, IV
Next
Explanation:
The MSRB stands for the Municipal Securities Rulemaking Board. It is the SRO for the municipals business, but as the title indicates; it makes rules but doesn’t enforce them. For bank dealers, the enforcement authorities are the FRB, the FDIC and the Comptroller of the Currency. For broker-dealers, the enforcement of MSRB rules falls under the NASD and the SEC.
42) Which of the following types of underwriting commitments by an Investment Banker could not be made in an IPO?
(a) Standby
(b) Firm Commitment
(c) All-or-None
(d) Mini-max
Next
Explanation:
A standby underwriting is a form of firm commitment underwriting used in a preemptive rights offering. The underwriter is “standing by” until after the rights offering has expired and will purchase, for distribution, any unsubscribed shares. Since preemptive rights are only offered to existing shareholders, this could not be done in an Initial Public Offering.
44)
Under the Securities Act of 1933, a prospectus is required for the sale of new securities. Which of the following are the periods of time for which a prospectus must be available for public customers?
I. 20 days for an IPO
II. 40 days for a subsequent primary offerings
III. 90 days for an IPO
IV. 35 days for subsequent primary offerings

(a) II, III
(b) I, IV
(c) IV
(d) I
Next
Explanation:
The rules require that the prospectus for an IPO be available for 90 days from the effective date of the registration. When a company has an additional—or subsequent—primary offering, the prospectus must be available for 40 days.
45)
Which of the following would violate the anti-reciprocal rule?

(a) Recommending that clients purchase the broker-dealer’s proprietary mutual funds
(b) Recommending that a client purchase a stock that the broker-dealer holds in inventory
(c) An investment company choosing a broker-dealer with which it has a selling agreement for its portfolio transactions
(d) Recommending that clients purchase the shares of a particular mutual fund in return for portfolio transaction business from that investment company
Next
Explanation:

The anti-reciprocal rule specifically prohibits recommending the purchase of mutual funds based on the broker-dealer’s portfolio transaction business with that fund. Recommendations for the customer must be based solely on suitability for the customer. It is not necessary for an investment company to choose a broker-dealer with which it has no selling agreement, but the volume of sales cannot be the factor in that choice.
46)
Under the MSRB Rules, which of the following are not considered advertising?

I. Abstract of the Official Statement
II. Preliminary Official Statement
III. Summary of the Official Statement
IV. The Official Statement

(a) I, III
(b) I, II
(c) I, II, III, IV
(d) II, IV
47)
During a municipal securities representative’s apprenticeship period, which of the following are permitted?

I. Making transactions with other municipal securities professionals
II. Working with other broker/dealers on a salary basis
III. Making commission-based transactions for new customers
IV. Discussing municipal securities with existing customers

(a) I, II, III, IV
(b) I, II, III
(c) III, IV
(d) I, II
Next
Explanation:
MSRB Rule G-3 stipulates that during the 90-day apprenticeship period, a new municipal securities representative may make transactions, but only with other municipal broker-dealers or professionals. They may be paid only by salary during this period.
Which of the following sets the margin requirements for the purchase of municipal bonds?

(a) NASD/NYSE
(b) Federal Reserve
(c) MSRB
(d) SEC
Next
Explanation:

Municipal securities, along with U.S. Government securities are exempt securities. As such, Federal Reserve Board Regulation “T” does not govern them. Since investors may choose to buy these bonds on margin, the Self-Regulatory Organizations, which govern the business practices of broker-dealers, set the margin requirements for municipal securities. The MSRB makes rules, but does not enforce them and the SEC does not become involved setting margin rules.
49)
Which of the following would be considered advertising?

I. An announcement in the local newspapers about employee promotions within a broker/dealer’s offices
II. A newspaper article announcing a broker/dealer’s new 24-hour toll-free hotline for customers
III. A tombstone ad
IV. A TV announcement describing a new mutual fund

(a) I, II
(b) I, II, III, IV
(c) II, III
(d) III, IV
Next
Explanation:
Advertising is public communications that solicit for securities business. An announcement in the local newspaper about a broker/dealer’s promotions and a newspaper article announcing a broker/dealer’s new 24-hour toll-free hotline are public-relations notices, not solicitations. Although a tombstone ad does not have to meet prospectus requirements, it is an advertisement. The TV spot is definitely considered advertising under the rules.
54)
A broker-dealer is conducting a series of seminars in major cities to inform the public and attract new customers. What are the records that must be kept on file for each seminar?

I. The names of the persons attending
II. The date and location of each seminar
III. A copy of the script of each presentation
IV. The names of the RRs making the presentations

(a) II, III, IV
(b) II, III
(c) I, III, IV
(d) I, II, III, IV
Next
Explanation:
The rules require all the items listed except choice I. Obviously, the firm will keep a list of attendees for prospecting. Additionally, a principal of the firm must approve the script for the seminar.
n Progress : Question No. 57 of 87
57) What is the purpose of SEC Rule 145?
(a) Sets forth the requirements for registration of small issues
(b) Provides that securities acquired in recapitalization, merger or consolidation are exempt from registration
(c) Permits shelf registration
(d) Specifies the requirements for the public sale of unregistered, Regulation D stock
Next
Explanation:
EC Rule 145 provides that persons who acquire stock as a result of reclassification, merger, consolidation or transfer of assets may sell the securities without registration.
58) Which of the following is a prohibited practice under the SEC Rules?
(a) Failure, by a client, to complete a letter of intent
(b) Buying stock just prior to the ex-dividend date
(c) Short tendering of stock
(d) Converting a bond and shorting the stock without an uptick
Next
Explanation:
A tender offer is an offer to buy the shares of a company back from the public, often as a means of taking control of the target company in an acquisition. Selling stock short involves borrowing stock; selling it and buying it back to replace the borrowed stock. Since a tender offer could easily involve a takeover there may not be stock available for replacement. SEC Rule 10b-4 prohibits short tendering. Failure to complete a letter of intent may cost the customer, but it isn’t prohibited. When a person converts a bond and sells the stock simultaneously, the sale of the borrowed stock will be covered by the stock acquired by conversion and is short-exempt. Buying stock just prior to the ex-dividend date is usually considered an unwise practice, but not a prohibited practice—convincing a customer to do that, however, is prohibited.
59) A General Securities RR can sell all the following except:
(a) Derivatives
(b) Commodities futures contracts
(c) Municipal bond mutual funds
(d) Penny stock
Next
Explanation:
A General Securities RR is a Series 7 registered representative and can sell all securities. Commodities futures contracts are not legally considered securities. To sell commodities, the RR would have to take other examinations such as the Series 3.
60) The simplified Code of Arbitration of the NASD, when public customers are involved in the process, applies to which of the following?

I. Complaints by public customers regarding alleged misconduct by member firms or Registered Representatives.
II. Disputes regarding amounts not exceeding $25,000.
III. Complaints by member firms against other member firms alleging such matters as dishonest conduct by associated persons.
IV. Monetary disputes involving public customers when the customer has signed a pre-dispute agreement for the process.
(a) II, IV
(b) I, II, III, IV
(c) I, III
(d) I, III, IV
Next
Explanation:
Simplified Arbitration is for the settlement of monetary disputes between member firms, associated persons and public customers for amounts in dispute not exceeding $25,000. If a public customer is involved, the customer must have signed a pre-dispute arbitration agreement before any proceedings may take place.

Options I and III are incorrect as both deal with disputes that are not monetary. Note that these options would be correct if the question referred to the Code of Arbitration rather than the simplified version.
In a analyzing a customer’s financial profile for suitability of investment recommendations, which of the following is included?

I. Balance sheet
II. Income statement
III. Home ownership
IV. Life and disability insurance

(a) I, II, III, IV
(b) I, II, III
(c) II, IV
(d) I, II
Next
Explanation:
All of these, plus nonfinancial considerations such as family and risk tolerance, must be considered when evaluating a customer in the information-gathering process.
64)
An individual made a one-time deposit of $25,000 into a variable annuity when she was 45 years old. The annuity has grown and the total value of the account ten years later is $40,000. The investor, who is in the 28% tax bracket, calls her registered representative and tells him that she has decided to withdraw $10,000 from the account and allow the remainder to continue its tax-deferred growth. The RR should advise her that:

(a) she will have a $2,800 tax liability
(b) she may withdraw an amount equal to the original deposit with no tax consequences.
(c) she withdrawal will be treated as FIFO
(d) she will have a tax liability of $3,800
Next
Explanation:
In an annuity, withdrawals are LIFO (Last In First Out). The “last in” is the growth. The investor must take out the growth and pay the appropriate taxes before she may withdraw the deposits (cost basis). If a withdrawal is made, without annuitizing the account—except for death or disability—the withdrawal is taxed at the ordinary income rate and penalized 10% if the investor is younger than 59 ½ . This investor is younger than 59 ½ and is in the 28% tax bracket, so (28% + 10% = 38% × $10,000 = $3,800).
A DPP must avoid certain corporate characteristics if it wishes to retain its tax advantages. The IRS has deemed that which of the following are “avoidable” characteristics of a corporation?
I. Full flow through of income and expenses
II. Limited liability
III. Free transferability of ownership
IV. Continuity of life

(a) I, II, III
(b) I, III, IV
(c) II, IV
(d) II, III, IV
Next
Explanation:

Choices II, III and IV are three of the four “avoidable” characteristics. The business must avoid two of these to be taxed as a DPP. The fourth, and hardest to avoid, is centralized management. Usually, the limited partnership will avoid continuity of life, because there is a fixed term to the business; and the free transferability of ownership because interest in the limited partnership is considered to be an illiquid investment. Giving up the limited liability for an investor is a very difficult proposition, and normally isn’t an option.
73) When an RR is evaluating a new customer, which of the following is not included in the client’s income statement?
(a) Current income
(b) Current expenses
(c) Liquid net worth
(d) Employment of family members
Next
Explanation:
Employment of family members is considered a part of the “nonfinancial investment considerations” in the Series 7 outline. The concept is that the income of other family members may not be available to the RR’s client
75)
Which of the following are characteristics associated with Keogh plans?
I. They are intended for corporations with less than 200 employees.
II. The plan holder must include employees who are 21 years old.
III. These plans are for self-employed persons
IV. Employees with one year of service or more must be included in the plan

(a) III
(b) I, II, IV
(c) II, IV
(d) II, III, IV
Next
Explanation:

Keogh Plans, also known as H.R. 10 Plans, are for self-employed persons. Only income from self-employment activities may be used as contributions. The rules require that employees who have attained age 21 or older and who have one year of service be included in the plan at the same percentage rate as the plan holder (the boss).
79) An RR has been interviewing a new customer. The customer is a lady who is interested in investing to pay for her grandchildren’s education. One grandchild will be entering college in three years and the other in six years. Which of the following recommendations will be most suitable?
(a) An aggressive income fund with a portfolio of high-yield bonds
(b) A portfolio of investment-grade bonds with maturities that begin in three years and continue for nine years
(c) A conservative capital mutual fund with the objective of capital appreciation
(d) A portfolio of adjustment bonds with ten-year maturities
Next
Explanation:
A portfolio of investment-grade bonds with maturities that begin in three years and continue for nine years is a laddered bond portfolio. The advantages of such a portfolio are lower risk and ongoing liquidity. The interest from the bonds provides income and the staggered maturity dates provide funds from the maturing bonds. A part of the risk reduction is that it is unlikely that all bonds will be called simultaneously. Adjustment bond is another term for an income bond, which only pays interest if the company has sufficient earnings.
80)
A grandfather is the custodian for his 13-year old grandson in a UTMA account. This past year, the account had earnings of $2,000. What are the tax implications of these earnings?

(a) The entire $2,000 is taxable to the child at the child’s tax bracket
(b) $1,500 is taxable at the child’s bracket, $500 is taxable to the child at the parent’s top marginal bracket.
(c) $1,500 is taxable at the child’s bracket, $500 is taxable at the grandfather’s bracket.
(d) $2,000 is taxable to the child at the parent’s top bracket
I. Municipal bonds
II. Unlisted stocks
III. Third market
IV. Primary offerings
(a) I, III
(b) II, IV
(c) I, II, III, IV
(d) II, III
Next
Explanation:
Any transaction that doesn’t take place on an exchange is an OTC (Over The Counter) trade. Of course municipal bond and unlisted stock trades are OTC, but the trades in the “Third Market” are OTC trades in listed stocks. Primary offerings come directly from the issuer (through an underwriter) to the public, and thus, are OTC.
84)
An investor bought 1,000 shares of PDQ stock at $10/share. Later, when the stock was trading at $15/share, he gave the stock to his favorite niece. She held the stock for 14 months and sold the stock at $18/share. What are the tax implications in this series of transactions?

(a) The uncle has a write-off of $10,000 and the niece has a taxable gain of $3,000
(b) The niece owes long-term gains tax on $8,000 and her uncle pays gift tax on $4,000.
(c) The uncle has taxable gain of $5,000 at the ordinary rate and the niece has a long-term gain of $3,000
(d) The niece will be required to pay gift taxes, as the donee, on $5,000 and her uncle will pay long-term gains taxes on $5,000
Next
Explanation:

A donor must, under the Gift Tax Rules, pay taxes on everything above $11,000 given in a year. When he gave the stock to his niece, the value was $15,000, so he pays gift tax on $4,000. When the niece sells the stock, after holding it for 14 months, she pays long-term gains taxes using the original cost basison the stock, which was $10,000. She pays taxes at the long-term rate on $8,000.
87) Which of the following would typically be considered the most appropriate investment recommendation for a retired person who seeks capital preservation and regular income?
(a) Money market mutual fund
(b) Income bonds
(c) Emerging markets mutual fund
(d) GNMA mutual fund
Next
Explanation:
The investor described needs both capital preservation and regular income. The GNMA fund provides these. Although the fund itself is not guaranteed, the GNMA certificates held by the fund are government guaranteed. Because GNMAs are “pass-through” securities, income from the pool of mortgages results in monthly distributions. Income bonds are high-risk because they only pay interest if the fund has adequate income. Emerging markets funds are aggressive growth funds, primarily in overseas markets and normally pay no income. A money market mutual fund is good for preservation of capital and liquidity, but does not provide the regular income required.
Public communications: definition and approval process

1. Advertisement: advertising in any electronic or other public media, including any website, newspaper, magazine or other periodical, radio, television, telephone or tape recording, videotape display, signs or billboards, motion pictures or telephone directories (other than routine listings).
2. Sales Literature: literature distributed or made generally available to customers or the public, including circulars, research reports, market letters, performance reports or summaries, form letters, telemarketing scripts, seminar texts, reprints (that are not independently prepared reprints) or excerpts of any other advertisement, sales literature or published article, and press releases concerning a member’s products or services.
3. Correspondence: written letter or electronic mail message distributed by a member to existing retail customers and fewer than 25 prospective retail customers within any 30 calendar-day period.
4. Institutional Sales Material: material distributed or made available only to institutional investors.
5. Public Appearance: seminar, forum (including interactive electronic forums), radio or television interview, or other public appearance or public speaking activity.
6. Independently Prepared Reprint: unaltered reprint or excerpt of any article issued by a publisher, provided the publisher is unaffiliated with and uncompensated by the circulating firm or investment company or any underwriter or issuer of a security mentioned in the reprint, and neither the member using the reprint nor any underwriter or issuer of a security mentioned in the reprint or excerpt has commissioned the reprinted or excerpted article.
There are separate standards for public communications about options, municipal bonds, mutual funds and other securities.

By admin